beck fed incometax outline

139
Chap 1: Orientation.................................................6 I. A Look Forward..................................................6 A. Understanding Income Tax.....................................6 B. Two Types of Tax Practice....................................6 II. A Glimpse Backwards............................................7 A. Law Prior to 1939............................................7 B. Codification.................................................7 C. The Income Tax and the Constitution..........................7 III. The Tax Practitioner's Tools..................................8 A. Legislative Materials........................................8 B. Administrative Materials.....................................8 C. Judicial Materials...........................................8 IV. Tax Policy Considerations......................................8 Identification of Income Subject to Taxation ........................... 8 Chap 2: Gross Income § 61...........................................8 I. Introduction....................................................8 A. Gross Income defined § 61..................................9 B. Equivocal Receipt of Financial Benefit. Regs 1.61-1, -2(a) (1), -2(d)(1), -14(a)............................................9 II. Imputed Income................................................10 A. Income without receipt of cash or property Regs 1.61-2(a)(1), -2(d)(1)........................................................10 B. Bartering...................................................10 C. From Closely Held Corporations..............................10 Chap 3: The Exclusion of Gifts and Inheritances....................11 I. What to Include, What to Exclude...............................11 A. §§ 71 – 90 Items specifically included in gross income......11 B. §§ 101 – 139 Items specifically excluded from gross income..11 II. Gifts § 102...................................................12 A. Income Tax Meaning of Gift § 102(a) Regs 1.102-1(a), (b)..12 B. Gifts to Employees § 102(c); 274(b) see 74(c); 132(e); 274(j)..........................................................12 III. Bequests, Devises, & Inheritances ..§ 102(b) Regs 1.102-1(a), (b) 12 A. Definition of inheritance...................................12 Chap 4: Employee Benefits..........................................13 I. Exclusions for Fringe Benefits ... § 132 see 61(a)(1); 79; 83; 112; 120; 125. 13 Regs 1.61-1(a), -21(a)(1) & (2), (b)(1) & (2), 1.132-2(a)(2) & (5) ..................................................................13 A. No-additional cost service § 132(a)(1) & (b)..............14 B. Qualified employee discount § 132(a)(2) & (c).............14 C. Working condition fringe § 132(a)(3) & (d)................15 D. De minimis fringe § 132(a)(4) & (e)......................15

Upload: evan-joseph

Post on 03-Mar-2015

252 views

Category:

Documents


15 download

TRANSCRIPT

Page 1: Beck Fed IncomeTax Outline

Chap 1: Orientation...................................................................................................................................6I. A Look Forward..................................................................................................................................6

A. Understanding Income Tax.......................................................................................................6B. Two Types of Tax Practice.......................................................................................................6

II. A Glimpse Backwards........................................................................................................................7A. Law Prior to 1939.....................................................................................................................7B. Codification..............................................................................................................................7C. The Income Tax and the Constitution......................................................................................7

III. The Tax Practitioner's Tools.............................................................................................................8A. Legislative Materials................................................................................................................8B. Administrative Materials..........................................................................................................8C. Judicial Materials......................................................................................................................8

IV. Tax Policy Considerations................................................................................................................8

Identification of Income Subject to Taxation ............................................................................................... 8

Chap 2: Gross Income § 61.......................................................................................................................8I. Introduction..........................................................................................................................................8

A. Gross Income defined § 61.....................................................................................................9B. Equivocal Receipt of Financial Benefit. Regs 1.61-1, -2(a)(1), -2(d)(1), -14(a).....................9

II. Imputed Income................................................................................................................................10A. Income without receipt of cash or property Regs 1.61-2(a)(1), -2(d)(1)...............................10B. Bartering.................................................................................................................................10C. From Closely Held Corporations............................................................................................10

Chap 3: The Exclusion of Gifts and Inheritances.................................................................................11I. What to Include, What to Exclude.....................................................................................................11

A. §§ 71 – 90 Items specifically included in gross income.........................................................11B. §§ 101 – 139 Items specifically excluded from gross income................................................11

II. Gifts § 102........................................................................................................................................12A. Income Tax Meaning of Gift § 102(a) Regs 1.102-1(a), (b)................................................12B. Gifts to Employees § 102(c); 274(b) see 74(c); 132(e); 274(j)...........................................12

III. Bequests, Devises, & Inheritances § 102(b) Regs 1.102-1(a), (b).............................................12A. Definition of inheritance.........................................................................................................12

Chap 4: Employee Benefits.....................................................................................................................13I. Exclusions for Fringe Benefits § 132 see 61(a)(1); 79; 83; 112; 120; 125.................................13Regs 1.61-1(a), -21(a)(1) & (2), (b)(1) & (2), 1.132-2(a)(2) & (5)......................................................13

A. No-additional cost service § 132(a)(1) & (b).......................................................................14B. Qualified employee discount § 132(a)(2) & (c)...................................................................14C. Working condition fringe § 132(a)(3) & (d).........................................................................15D. De minimis fringe § 132(a)(4) & (e)...................................................................................15E. Qualified transportation fringe § 132(a)(5) & (f)..................................................................15

II. Exclusions for Meals & Lodging §§ 107, 119(a) see § 119(d) Reg 1.119-1.............................16

Chap 5: Awards.......................................................................................................................................17I. Prizes § 74 see § 102(c); 132(a)(4), (e); 274(j). Reg 1.74-1......................................................17

A. Non-taxable prizes..................................................................................................................17II. Employee Achievement Awards § 74(c)....................................................................................18III. Scholarships & Fellowships §§ 117; 127(a), (b)(1), (c)(1) Reg 1.117-6(b), (c)(1)-(4), (d)(1)-(3) 18

A. Scholarship.............................................................................................................................18B. Reduction in Tuition...............................................................................................................19

Chap 6: Gain from Dealings in Property...............................................................................................20I. Factors in the Determination of Gain §§ 1001(a), (b) first sentence, (c); 1011(a) Reg 1.1001-1(a)...............................................................................................................................................................20

Page 2: Beck Fed IncomeTax Outline

II. Determination of Basis §§ 109; 1011(a); 1012; 1016(a)(1); 1019 Reg 1.61-2(d)(2)(i); 1.1012-1(a) 21

A. Cost as Basis...........................................................................................................................21B. Property Acquired by Gift § 1015(a) see §§ 1015(d)(1)(A), (4) & (6) Reg 1.1015-1(a).....22C. Property Acquired Between Spouses or Incident to Divorce § 1041 Reg 1.1041-1T(a) & (d) 23D. Property Acquired From Decedent § 1014(a), (b)(1) & (6), (e) Reg 1.1014-3(a)................24

II. Amount Realized § 1001(b) Reg 1.1001-1(a), -2(a), (b), (c) Examples (1) & (2)..................24

Chap 7: Life Insurance Proceeds and Annuities...................................................................................26I. Life Insurance § 101(a), (c), (d), & (g) Reg 1.101-1(a)(1), (b)(1), -4(a)(1)(i), (b)(1), (c)............26

A. Non-taxable proceeds.............................................................................................................26B. Interest payments....................................................................................................................27C. Payments over time.................................................................................................................27

II. Annuities (the new rule) § 72(a), (b), (c) Reg 1.72-4(a), -9(table V)........................................28

Chap 8: Discharge of Indebtedness........................................................................................................29I. Included in income §§ 61(a)(12); 102(a); 108(a), (b)(1)-(3), (d)(1)-(5), (e)(1) & (5); 1017(a), (b)(1), (2), (3)(A) & (B) Reg 1.61-12(a); 1.1001-2(a), 2(c) Ex. 8...........................................................29II. Exceptions to 61(a)(12)...................................................................................................................30

Chap 9: Damages And Related Receipts...............................................................................................32I. Damages for Business Loss...............................................................................................................32II. Damages & Recoveries for Personal Injury.....................................................................................33

A. Damages § 104(a) Reg 1.104-1(a), (c), (d).........................................................................33B. Insurance and Workers' Compensation §§ 105(a)-(c) & (e); 106(a) Reg 1.105-1(a); 1.106-1

33

Chap 10: Separation And Divorce.........................................................................................................36I. Alimony and Separate Maintenance Payments.................................................................................36

A. Direct Payments §§ 71; 215(a) & (b); 7701(a)(17) Reg 1.71-1T(a) & (b)...........................36B. Indirect payments § 71(b)(1(A) Reg 1.71-1T(b)(q 6 & 7)..................................................38

II. Property Settlements § 1041; see 1015(e) Reg 1.1014-1T(b)...................................................39III. Other Tax Aspects of Divorce........................................................................................................40

A. Child Support § 71(b)(1)(D); (c) Reg 1.71-1T(c)...............................................................40

Chap 11: Other Exclusions From Gross Income..................................................................................41I. Gains From the Sale of Principle Home § 121............................................................................41Reg 1.121-1(a)-(e), -1(f) ex. 1,6,10 & 11, -2(a), -3(a), (b) ex 1...........................................................41

Identification of the Proper Taxpayer ........................................................................................................ 42

Chap 12: Assignment Of Income............................................................................................................42I. The Progressive Tax System & Income Splitting §§ 1(a) – (e), (h); 6013(a) see §§ 1(g); 63; 66; 73...............................................................................................................................................................42II. Income From Services......................................................................................................................43

A. Assignment of Income............................................................................................................43B. Renunciation of income..........................................................................................................43C. Waiver of Executor Fee..........................................................................................................43D. Faculty....................................................................................................................................43

III. Income From Property....................................................................................................................43A. General Rule...........................................................................................................................44B. Exceptions...............................................................................................................................44C. Income Earned But Not Realized...........................................................................................44

Deductions in Computing Taxable Income ................................................................................................ 44

Chap 14: Business Deductions................................................................................................................44I. Introduction §§ 1; 63....................................................................................................................44

Page 3: Beck Fed IncomeTax Outline

A. Deductions..............................................................................................................................44B. Definition................................................................................................................................45C. Trade or Business Requirement..............................................................................................45

II. The Anatomy of the Business Deduction.........................................................................................45A. “Ordinary and Necessary” § 162(a) Reg 1.162-1(a)...........................................................45B. “Expenses” §§ 162(a); 263(a) Reg 1.162-4; 1.263(a)-2....................................................46C. “Carrying On” Business § 162(a); 195; 262 Reg 1.195-1(a)..............................................46

III. Specific Business Deductions.........................................................................................................48A. Reasonable Salaries § 162(a)(1) see §§ 162(m); 280G Reg 1.162-7, -8, -9.........................48B. Travel “Away from Home” §§ 162(a)(2); 274(n)(1) see § 274(c), (h) & (m)(1) & (2) Reg 1.162-2..............................................................................................................................................49C. Necessary Rental and Similar Payments § 162(a)(3) Reg 1.162-11(a)................................51D. Expenses for Education §§162(a); 262; 274(m)(2) Reg 1.162-5(a), (b)(1), (2)(i), 3(i), (c), (d), (e)(i)...........................................................................................................................................51

IV. Miscellaneous Business Deductions...............................................................................................52A. Introduction §§ 162(a); 274(a), (d), (e), (k), (l), (n) Reg 1.162-20(a)(2); 1.274-2(a)(1), (c), (d) 52B. Business Losses §§ 165(c)(1); 280B.....................................................................................54

V. Depreciation §§ 167; 168............................................................................................................54A. Introduction.............................................................................................................................54B. Special Depreciation Rules on Personal Property §§ 168(k); 179; 280F(a) & (b); 197.........55C. Special Rules on Realty §§ 168; 42; 46; 47...........................................................................56

Chap 15: Deductions For Profit-Making, Nonbusiness Activites........................................................57I. Expenses for production of income §§ 212; 274(h)(7) Reg 1.212-1(g), (k), (l), (m); 1.262-1(b)(7)..........................................................................................................................................................57

A. § 212 or § 162.........................................................................................................................57B. Limitations & Restrictions......................................................................................................57C. Personal expenses are not deductible § 262..........................................................................57

II. Expenses arising out of Transactions entered into for profit §§ 121(a), (d)(6); 165(a), (b), (c)(2); 167(a)(2); 168(a); 212 see §§ 195; 280A Reg 1.165-9(b); 1.167(g)-1; 1.212-1(h)............................58

Chap 16: Deductions Not Limited To Business or Profit-Seeking Activities.....................................60I. Interest § 163.............................................................................................................................60

A. Personal Interest § 163(h).......................................................................................................60B. Qualified Educational Loan § 221..........................................................................................60C. Loans to purchase tax-exempt property § 265(a)(2)..............................................................60D. Investment interest § 163(d)...................................................................................................61

III. Taxes §§ 164(a), (b), (c), (d)(1); 275; 1001(b)(2) Reg 1.164-3(a)-(d)..................................61A. Property taxes.........................................................................................................................62

Chap 17: Restrictions on Deductions.....................................................................................................62I. Introduction........................................................................................................................................62II. Activities not engaged in for profit § 183(a)-(d).........................................................................63

A. Presumption of profit motive..................................................................................................63B. Deductions if not for profit (Hobby) § 183(b).......................................................................63

III. Restrictions on Deductions of Homes § 280A(a), (b), (c)(1), (3), & (5), (d)(1), (e), (f), (g).....63A. Rental of Home.......................................................................................................................63B. Limits under § 280A...............................................................................................................63C. Home Office § 280A(c)..........................................................................................................64

IV. Illegality or Impropriety § 162(c), (f), (g)...................................................................................65A. Criminal Defense....................................................................................................................65

Chap 18: Deductions For Individuals Only...........................................................................................66I. Adjusted gross income.......................................................................................................................66II. Moving expenses §§ 62(a)(15); 82; 132(a)(6) & (g); 217 Reg 1.217-2(b)(2), (3), (4), & (8)....67

A. Distance & Time.....................................................................................................................67B. What is included.....................................................................................................................67

Page 4: Beck Fed IncomeTax Outline

III. Medical Care §§ 213(a), (b), (d)(1)-(4) & (9); 263(a)(1) Reg 1.213-1(a)(1), (e)(1)-(4)(i)(a)....68A. Inclusions................................................................................................................................68B. Addition to home....................................................................................................................68C. Food and lodging....................................................................................................................68D. Long term care........................................................................................................................69E. Business expenses...................................................................................................................69

IV. Exemptions §§ 151, 152, 7703 Reg 1.151-1(b) & (c)(2).........................................................69A. Personal exemption § 151......................................................................................................69B. Dependents §152 Reg 1.152-1, -4T......................................................................................70C. Standard Deduction § 63, 67, 68, 7703 Reg 1.67-1T(a).......................................................72

The Year of Inclusion or Deduction ............................................................................................................ 73

Chap 19: Fundamental Timing Principles............................................................................................73I. The Cash Method of Accounting: Cash Receipts and Disbursements..............................................73

A. Receipts § 446, 451 Reg 1.446-1(c)(1)(i); 1.451-1(a), -2.....................................................73B. Disbursements § 461(a) & (g) Reg 1.461-1(a)(1)...............................................................74

The Characterization of Income and Deductions ...................................................................................... 75

Chap 21: Capital Gains And Losses § 1(H), 1222.................................................................................75I. Introduction........................................................................................................................................75

A. Capital Gain............................................................................................................................75II. Calculating Capital Gains § 1222.....................................................................................................76

A. Net from the inside out...........................................................................................................76III. Capital Losses § 1211, 1212...........................................................................................................77

A. Limitation...............................................................................................................................77B. Carry forward..........................................................................................................................77

IV. Definition of Capital Asset § 1221.................................................................................................77A. Exceptions...............................................................................................................................77B. Income Property......................................................................................................................78

V. Sale or Exchange Requirement........................................................................................................79A. Introduction.............................................................................................................................79B. Correlation with prior transactions.........................................................................................79

VI. The Holding Period.........................................................................................................................80A. Acquisition date......................................................................................................................80B. Trade Date..............................................................................................................................80

Page 5: Beck Fed IncomeTax Outline

Introduction to Income TaxFederal Income Tax Individual - Prof Beck- [email protected]

Individual Tax FormulaGross income

- § 62 deductions = Adjusted Gross Income - § 63 Personal Exemptions & either Standard Deduction or Itemized Deductions = Taxable Income

Taxable Income x Tax Rate = Gross Tax Liability

Gross Tax Liability - Tax Credits = Net Tax Liability or Tax Refund

From the chart:[(Ordinary income – “above the line” deductions – personal exemptions – the greater of the standard deduction or itemized deductions) x taxpayer’s ordinary rate] + [net capital gain x taxpayer’s capital gains rate] – [credits] = tax liability

derivation of the basic formula:the general formula is: tax liability = taxable income x taxpayer’s rate – credits§ 63: taxable income = adjusted gross income – personal exemptions – (the greater of the standard deduction or itemized deductions)§ 62: adjusted gross income = gross income – above the line deductions§ 61: gross income = all includable incomeSo, gross income includes both ordinary income and capital gains for determining adjusted gross income, but since they are taxed at different rates the final formula separates them.

Distinguishing income tax from other taxes:- estate tax: paid on property over $ 1,00,000 – tax on the wealthy – being phased

out by 2010, unless phase out repealed – originally intended to prevent huge wealth from being retained and passed on – raises very little revenue because the wealthy would rather give their money to charities

- gift tax: paid on property given before death – similar to estate tax- inheritance taxes: state level – the heir is taxed on the property received - sales tax: federal sales tax is called excise tax (gasoline, liquor, firearms etc; tax is

in the price) – state sales tax is a percentage imposed at the point of sale – paid by vendor on a periodic basis – customer is done with the tax once the purchase is made but does not take account of financial status of customer (poorer pay higher percentage of income in sales tax than the rich)

- Value Added Tax – at each stage of production and resale, a tax is added – but refunds are issued to businesses buying for resale – each person in the chain polices the validity of the taxability of the purchase (self-policing tax) -

Page 6: Beck Fed IncomeTax Outline

withholding taxes are a good example of a self policing tax – VAT falls more heavily on the poor, like sales tax

o Some items are exempt from sales tax or VAT tax - Property tax: every state and local – no federal property tax because clause in

Constitution says a direct tax must be apportioned (each states share of the federal tax must be proportional to their population) – this results in a different tax rate from state to state and Constitution requires uniformity across the country – state and local taxes are assessed on value of property; same rate within the taxing authority – property taxes tend to be very politicized (especially in their relation to funding of public education)

- Personal property taxes: avoided by not reporting the property or holding the property out of state

- Hotel and tourism taxes: federal, state and local- Social security tax – 15.3% of all pay (7.65% by employee and 7.65% by

employer) – capped at $102,000 for 2008 – for all employees but not independent contractors – easy tax to administer; no deductions, no exemptions – and no individual rights to a benefit (the essential aspect of a tax) – this is a flat tax

Marginal tax rate: tax on the last dollar of income – brackets of percentage of taxUp to 10,000 pays 0% 10,001 to 20,000 pays 15% 20,001 to 30,000 pays 28% each additional dollar pays 28cents Effective rate is the sum of all the tax bracket effecting the tax payerExample:10,000 @ 0% no tax on first 10,00020,000 @ 50% - half of next 10,000total tax is 5,000Effective tax rate is 25%

Chap 1: OrientationI. A Look Forward

A. Understanding Income Tax1. More than just a numbers game, the study of tax involves both, clear

reasoning and careful attention to policy considerationsa. Why study policy? It fosters an understanding of tax that permits lawyers

to predict how unsettled matters will be resolved.2. Scope of Tax Law: has some effect on virtually every substantive area of the

law.3. The Code

a. The study of income tax begins with the Internal Revenue Code.b. The Code is often cryptic and difficult to understand.

B. Two Types of Tax Practice1. Reactive: Tax practice can involve an application of tax principles to

completed transactions.2. Planning: Tax practice can also involve the structuring of proposed

transactions according to tax principles.

Page 7: Beck Fed IncomeTax Outline

II. A Glimpse BackwardsA. Law Prior to 1939

1. The first internal-revenue tax law, enacted in 1791, imposed a tax on alcohol, while subsequent legislation imposed taxes on carriages, tobacco, sugar and other items.a. These taxes were abolished only a decade later – Pollock case

2. Due to the War of 1812, taxes were again imposed in 1813, but were abolished five years later.

3. first income tax imposed by Congress was enacted in 1862, and it taxed virtually everything.

B. Codification1. Tax laws enacted after 1862 were eventually codified in 1939. 2. The Internal Revenue Code of 1954

a. In 1954, the first major revision of the Internal Revenue Code was passed by Congress.

b. Subsequent, less drastic revisions occurred in 1969, 1971, 1975, 1976, 1977, 1978, 1981, 1982, and 1984.

3. The Internal Revenue Code of 1986a. In an attempt to simplify the tax code, in 1986, Congress replaced the

1954 Code with what it believed to be broad-based, simple, fair, and revenue neutral legislation.

b. The 1986 Code ended up being more complicated than the 1954 version.c. Save for the years 1994 and 1995, the 1986 Code has been amended every

year since its enactment.C. The Income Tax and the Constitution

1. Power to Taxa. Article 1, Section 8, clause 1 of the Constitution vests Congress with the

"power to lay and collect taxes, duties, imposts and excises."2. Two Limits on Congressional Taxing Power

a. Article 1, Section 2, clause 3 and Section 9, clause 4 require that direct taxes be apportioned among the several states.(1) In other words, the monies raised from a direct tax must be divided

equally amongst the states in proportion to their populations.(2) A "direct" tax is one that is imposed on the person who will ultimately

pay it.b. Article 1, Section 8, Clause 1 requires indirect taxes to be uniform

throughout the states.(1) An indirect tax is imposed on a person who will shift the burden onto

another (e.g., a sales tax).(2) Only geographic uniformity is required by the Constitution.

3. The Sixteenth Amendment provides that taxes shall not be subject to the rule of apportionment.

4. Constitutional Attack: The taxing power has been interpreted so broadly that most taxing statutes are not subject to constitutional attack.

Page 8: Beck Fed IncomeTax Outline

III. The Tax Practitioner's ToolsA. Legislative Materials

1. The major source of tax law is the Internal Revenue Code. 2. As in other areas of the law, bills, debates, hearings, and committee reports

are often used to decipher the meaning of the Code.B. Administrative Materials

1. The Treasury Department is vested with the authority to promulgate regulations for the enforcement of the Code. (Administrative agency)a. Pursuant to this power, the Treasury Department has enacted a series of

regulations purporting to interpret the various provisions of the Code. (keyed to the code – § 61 interpreted in Regulation § 1.61) Regulations have force of law as long as they are “a reasonable interpretation of the statute.”

b. IRS of the Treasury Dept also issues revenue rulings that answer specific questions posed by a taxpayer. (can be given force of law by a judge or denied such status by a judge) (1) A ruling may not be relied upon by anyone other than the taxpayer to

whom it was issued, but it does give indication as to how the IRS will treat a specific situation.

(2) Private Letter Rulings are issued to one taxpayer and apply only to that one taxpayer’s specific situation.

C. Judicial Materials1. As usual, courts serve as the final arbiters of what the Code says.2. Both Article I and Article III courts are involved in the interpretive process.

a. The Tax Court is a forum provided for taxpayer to challenge those deficiencies determined by the Internal Revenue Commissioner. (only court if you want to resist)

b. Taxpayers may also resort to Federal District Court in order to recover sums they believe were improperly collected.

c. Refunds may also be sought in the Court of Federal Claims.d. Appeals

(1) The Federal Circuit Courts of Appeals reviews decision of tax court and district court.

(2) The Court of Appeals for the Federal Circuit reviews decisions from the Court of Federal Claims.

(3) The Supreme Court reviews cases emanating from any court, be it an Article I or Article III court. (but they don’t do many)

IV. Tax Policy ConsiderationsFederal income tax is not neutral, for any activity that is taxed is discouraged, while any activity that goes untaxed is to some extent encouraged.Taxation is a method used to indirectly "tinker" with the economy.

Identification of Income Subject to TaxationChap 2: Gross Income § 61I. Introduction

Page 9: Beck Fed IncomeTax Outline

A. Gross Income defined § 611. § 61 defines gross income as all income from whatever source derived

B. Equivocal Receipt of Financial Benefit. Regs 1.61-1, -2(a)(1), -2(d)(1), -14(a)

1. gross income includes income from all sources, unless the taxpayer can point to an express exemption. Cesarini v. United States.a. treasure trove is incomeb. claim of right doctrine: if money or property is apparently yours and yours

to dispose of or spend as you will, even if there is a remote contingency that might negate that, you will be taxed on it

2. So long as there is an economic benefit to the taxpayer, the income need not have come into his possession. Old Colony Trust v. Commissioner.a. The tax was the personal obligation of the employee and the employer

conferred a benefit to the employee by paying his debt to the IRS. (discharge of debt is considered gross income § 61(a)(12))

3. Gross income is an accession to wealth, clearly realized, and over which the taxpayer has complete dominion. (all gains are taxable income unless they are specifically excluded) Commissioner v. Glenshaw Glass Co.a. Punitive damages awards are taxable as gross income.b. Beck argues that the old definition from Macomber could apply (the gain

derived from capital, from labor, or from both combined) – the treble damages are the investment of the time and labor required for the lawsuit to be pursued.

4. Economic benefits are gross income, regardless of their source. Charley v Com missioner a. Travel credits (for the difference in value between a first class ticket and a

coach ticket) converted into cash in a personal travel account established by an employer constitute gross income to the employee.

b. Payments for tax purposes are characterized based on the payor’s motive and IRS can re-characterize a payment once it establishes what the payor was trying to do.(1) Frequent flier miles will not be taxed by the IRS (recent ruling) There

should be little doubt that frequent flyer miles earned on business trips paid for by an employer constitute gross income, but the problem comes with their valuation. Technically, the fair market value of flyer miles at the time of redemption is the "amount realized," whether characterized as a dealing in property or compensation income. However, the nature of plane tickets makes it difficult to determine the fair market value because the same ticket can wildly vary in price. The IRS therefore refrains from taxing these miles, unless they are traded in for cash, a point at which valuation becomes very easy. Had Dr. Charley not cashed in his miles it is most likely that he would not have been taxed on their use.

3. Illegal activity: Gains from illegal activity are gross income. James v. United States.

Page 10: Beck Fed IncomeTax Outline

II. Imputed IncomeA. Income without receipt of cash or property Regs 1.61-2(a)(1), -2(d)(1)

Imputed income is any product of your own exertions that you yourself consume (e.g., the fair market value of the tomatoes you grew, and then ate).1. Taxpayer does not have income from the use of his own real property, nor

does a homeowner have income from the use of his own house. Imputed income is not "income" under the 16th Amendment, and thus is generally not taxable. Helvering v. Independent Life Ins. Co.

2. Haig- Simons definition of income: Consumption + savings = incomesome countries charge a flat tax on consumption – hard to measure consumption – easier to measure it at the source of the income

3. Double tax on savings: once when earned and again on return (the interest) – consumption is only taxed once, when earned but not when used

B. Bartering 1. Exchange of services: In a transaction where services, not money, are

exchanged, the fair market value of the services received is includable as gross income. Revenue Ruling 79-24.

2. If a service is exchanged for goods, then the fair market value of the goods is includable as gross income. Revenue Ruling 79-24.

C. From Closely Held Corporations 1. Financial benefits received from one's wholly-owned corporation are not

imputed income (it can be considered either compensation or dividends), and they must be included as gross income. Dean v. Commissioner.

Problems pg 65:1) if Cesarini piano had instead been found to be worth $500,000, would the result to the taxpayer be different?

no, when he re-sells the piano, he would be taxed on the gain (selling price – original purchase price)2) free raffle ticket wins watch – any gross income?

yes, the fair market value of the property gained3) employee gets incentives to stay: $20,000 in company stock and $10,000 new car for spouse – income?

$35,000 – payment to wife does not allow you to evade taxation – IRS looks to employer’s motive to determine the payment’s character; it was intended as payment to employee - tax cost basis

4) insurance adjuster gets kickback on referrals – income?kickbacks are income, even if they are illegal – James v U.S.

5) O rents house to T for summer for $4000a) what income to O if T pays $1000 and does $3000 in improvements

$4000 income (payment in cash or property or services doesn’t matter, what matters is that the payment was intended to be rent)

b) what if all labor is done by T at cost to him on only $500labors are still valued at $3,000 as benefit to O

c) what tax to T$2500 income for labor in order to get 2500 worth of summer cottage – it’s a barter of services for services – tax barter situations by converting everything into cash

Hypo: free Ikea bus – is that income?Law student invited for interview – law firm pays for travel – is there income? Employer gives employee an officeProbably not income; these all benefit the one providing the bus, travel or office Beck test: what would happen if you asked for the cash instead?

Page 11: Beck Fed IncomeTax Outline

Chap 3: The Exclusion of Gifts and InheritancesI. What to Include, What to Exclude

Gross income includes any financial benefit unless it is:1. The mere return of capital2. A loan that the person agrees to repay3. Excluded from income by a specific statutory provision

A. §§ 71 – 90 Items specifically included in gross income71. Alimony and separate maintenance payments72. Annuities; certain proceeds of endowment and life insurance contracts73. Services of child74. Prizes and awards77. Commodity credit loans79. Group term life insurance purchased for employees82. Reimbursement for expenses of moving 83. Property transferred in connection with performance of services84. Transfer of appreciated property to political organization85. Unemployment compensation 86. Social security and tier 1 railroad retirement benefits

B. §§ 101 – 139 Items specifically excluded from gross income101. Certain death benefits 102. Gifts and inheritances103. Interest on state and local bonds104. Compensation for injuries or sickness105. Amounts received under accident and health plans106. Contributions by employer to accident and health plans107. Rental value of parsonages108. Income from discharge of indebtedness109. Improvements by lessee on lessor’s property 110. Qualified lessee construction allowances for short-term leases111. Recovery of tax benefit items112. Certain combat zone compensation of members of the Armed Forces115. Income of states, municipalities etc.117. Qualified scholarships118. Contributions to the capital of a corporation119. Meals or lodging furnished for the convenience of the employer121. Exclusion of gain from sale of principal residence123. Amounts received under insurance contracts for certain living expenses125. Cafeteria plans126. Certain cost-sharing payments127. Educational assistance programs129. Dependent care assistance programs132. Certain fringe benefits135. Income from US savings bonds used to pay higher education tuition and fees137. Adoption assistance programs

Page 12: Beck Fed IncomeTax Outline

139. Disaster relief payments

II. Gifts § 1021. Excludes from gross income any property received as a gift, bequest, devise,

or inheritance. Section 102(a).2. Does not exclude from income any income produced by the property that is

given. Section 102(b)(1).3. Does not exclude the gift of an income from a property. Section 102(b)(2).

A. Income Tax Meaning of Gift § 102(a) Regs 1.102-1(a), (b)1. A gift proceeds from a detached and disinterested generosity out of affection,

respect, admiration, charity, or like impulses. C ommissioner v. Duberstein a. best in context of friends and family - out of place in a commercial setting,

especially where there is a quid pro quob. may be a gift under property law (intent to make a gift, delivery and

acceptance) and not be a gift under tax law.2. Stanton portion of the case: The critical consideration in determining whether

a transfer is a gift is the intent or motive of the donor. a. The trier of fact makes this determination on a case by case basis

3. Origin of the claim test: general doctrine in tax law in which you consider the most relevant reason for the transfer – look for the motive for the transfer

B. Gifts to Employees§ 102(c); 274(b) see 74(c); 132(e); 274(j)1. Initially § 102 did not address the question of whether transfers from

employer to employee that were called gifts were excludable from income under the Section.

2. This was addressed by the addition of § 102(c) prohibits gifts to employeesEmployee gifts: (1) In general.-Subsection (a) shall not exclude from gross income any amount transferred by or for an employer to, or for the benefit of, an employee.

3. There are exceptions to this for de minimus fringes. § 132(e).4. Some employee achievement awards are excluded from gross income under §

74(c) and § 274(j).5. § 274 (b)(1) ceiling does not apply

III. Bequests, Devises, & Inheritances § 102(b) Regs 1.102-1(a), (b)Inheritance: non-specific residuary from a will or intestacy Bequest: a gift of personal property Devise: gift of real property

A. Definition of inheritance Just because the taxpayer labels a payment a bequest, and even if it is a bequest under state law, doesn’t mean that it is a bequest under tax law – have to look at the fact and circumstances and discover the true economic substance of the transaction 1. When an heir receives a payment as the result of settling their contest of a

will, they receive that money because of their status as an heir and thus it is to

Page 13: Beck Fed IncomeTax Outline

be considered an inheritance for purposes of § 102(a). What he got, he got because he was an heir and so such money should be treated as an inheritance. Ly eth v. Hoey a. State inheritance law governed the payment of state inheritance tax but

federal law governs what gets taxed as income. Congress apparently meant for inheritance to be understood broadly in that although they have tinkered with the language they have not made any move to limit the definition.

2. A bequest made in a will for the purpose of satisfying an agreed compensation for services rendered is not excluded from income under § 102(a). Wol der v. Commissionera. Surrogates court made the lawyer defend the debt to show that it was

really owedb. IRS can look through the form of a transaction and re-characterize it

according to its true economic substance

Chap 4: Employee BenefitsI. Exclusions for Fringe Benefits § 132 see 61(a)(1); 79; 83; 112; 120; 125.

Regs 1.61-1(a), -21(a)(1) & (2), (b)(1) & (2), 1.132-2(a)(2) & (5)

1. Fringe benefits are incidental benefits in an employment context given in kind (a form other than cash) – transfer of non-salary – most have their own sections: a. § 105 health benefits b. § 119 meals and lodgingc. § 127 education benefits

Problems pg 911) Does § 102 apply?

a) father leaves daughter $20,000 in his willyes, 102 applies – this is an inheritance – excluded from gross income

b) father dies intestate and daughter receives $20,000 in real estate as his heiryes, 102 applies

c) will is challenged and case settles – daughter gets $20,000Lyeth v Hoey – treated as an inheritance

d) father leaves daughter $20,000 in his will stating that it is in appreciation for her long & devoted servicecan be argued either way: Duberstein generosity or payment for services?

e) father leaves $20,000 pursuant to written agreement for her services102 does not apply – this is payment for services – unless she can show that other siblings got the same amount and that she didn’t claim deduction for expenses

f) daughter enforces agreement against intestate father’s estateWolder – if it is supported by a written agreement, it can’t be a gift

g) daughter is executrix of estate & gets $20,000 for servicesnot 102 gift

h) father leaves $20,000 in lieu of all compensation as executrixthis is a 102 gift

2) if Wolder case arose today, would 102 apply?No, Wolder was an independent contractor, not an employee

Page 14: Beck Fed IncomeTax Outline

d. §§ 4xx retirement plans2. Gifts from an employer to an employee are includible as gross income. §

102(c).a. But if the "gift" falls within § 13 2 , there is no gross income and nothing

has to be accounted for. – catch-all of fringe benefits3. Policy

a. Congress thought it unfair to tax someone who is paid in cash, while leaving untaxed the person who is paid in products or services. (1) However, Congress also realized that fringe benefits are sometimes for

the benefit of the employer, and not intended as additional compensation.

(2) For example, a clothing retailer may offer a discount to employees so that they are encouraged to "show off" the newest fashions.

b. tax law on fringe benefits is now completely statutoryA. No-additional cost service § 132(a)(1) & (b)

1. A no-additional cost service is a service provided by an employer to an employee:a. That is offered in the ordinary course of the line of business of the

employer in which the employee works; andb. Where employer incurs no additional cost in performing the service (and

does not lose any income because of it).2. Non-discriminatory basis.

a. To be eligible for the exclusion, a no-additional cost service must be offered to a substantial number of not "highly compensated employees." § 132(i).

3. Coverage: The no-additional cost service applies to services given to retired and disabled employees, widows of former employees, and the spouses and dependent children of current employees. § 132(h). (Special Rule for Air Travel: Use of Air transportation by parent of an employee is treated as use by the employee § 132(h)(3)

4. A no-additional cost service can also be offered as a rebate or through reimbursement. Reg. 1.132-2(a)(3)

5. Free hotel rooms for hotel employees and free airline seats for airline employeesa. if employee works for more than one business owned by a conglomerate,

he may get the benefit from any of the related businessesb. written reciprocal agreements between businesses in the same business

allow benefits to be provided to employees from other companiesB. Qualified employee discount § 132(a)(2) & (c)

1. A qualified employee discount is a discount with respect to qualified property or services that does not exceed:a. The gross profit percentage of the price at which the property is being

offered by the employer to customers (goods at but not below cost); orb. 20% of the price at which the service is offered customers.

(1) Gross profit percentage is calculated by dividing the dollar difference between the aggregate sales and aggregate cost by dollar amount of

Page 15: Beck Fed IncomeTax Outline

aggregate sales (Aggregate sales - Aggregate cost)

Aggregate sales2. Qualified property or services are those (other than real property or property

held for investment) which are offered for sale to customers in the ordinary course of the line of business in which the employee works. § 132(c)(4).

3. Non-discriminatory basis: The qualified employee discount is also subject to the non-discriminatory limitation. § 132(i).

4. Coverage: The discount may be given to retired and disabled employees, widows of former employees, and the spouses and dependent children of current employees. § 132(h).

5. The discount can be offered as a partial rebate, and can be offered at purchase or through reimbursement. Reg. 1.132-3.

C. Working condition fringe § 132(a)(3) & (d)1. A working condition fringe is any property or service provided to an

employee that would otherwise be deductible by the employee if he had to pay for it. § 132(d).a. For example, an employee who gets reimbursed for his business expenses,

may exclude the reimbursement from gross income because the expenses would be deductible anyway.

2. no non-discrimination limitations D. De minimis fringe § 132(a)(4) & (e)

1. A de minimis fringe is any property or service that is small enough to make accounting for it impracticable. Eg., Employer provided coffee, water, etc.

E. Qualified transportation fringe § 132(a)(5) & (f)A qualified transportation fringe is excludable from gross income. 1. Three types of qualified transportation fringe.

a. Transportation in a commuter highway vehicle if such transportation is in connection with travel between the employee's residence and work.

b. Any transit passc. Qualified parking - Parking provided to an employee on or near the

business premises of the employer, but not parking that is also used for the employee's residence.

2. Limitation on exclusion a. pass or transportation in a vehicle: maximum allowable exclusion of $100

per month. § 132(f)(2)(A).b. qualified parking: maximum allowable exclusion of $175. § 132(f)(2)(B).

Page 16: Beck Fed IncomeTax Outline

II. Exclusions for Meals & Lodging §§ 107, 119(a) see § 119(d) Reg 1.119-1

The value of meals and lodging that are provided to an employee for the employer's convenience is excludable from gross income. To qualify for the exclusion meals must be offered on the business premises. To qualify for the exclusion the lodging must be offered:

Problems pg 1011) excluded from gross income? What statute?

a) employee of national hotel chain stays at hotel – there are empty rooms132a1 exclusion if offered to all employees

b) same as above but desk clerk bounces a paying customer to make room for the employeenot excluded from income Reg 1.132-2(a)(2) and (5)

c) same as a, but employee pays for room and later gets rebateexclusion applies Reg 1.132-2(a)(3)

d) spouse and children use room without employeeexclusion applies § 132(h)(2) & Reg 1.132-1(b)(ii)

e) employee stays in rival hotel which has written agreement to give 50% the 50% is excluded § 132(i)

f) employee is an officer of the hotel; rent-free is only given to officers; employees pay 60%not excluded from income - § 132(j)(1) Reg 1.132-8(a)(ii)(2)

g) employee works for the shipping line which is owned by the same conglomerate that owns the hotelnot excluded §132(b)(1) Reb 1.132-2(a)(1)(i) & -4(a)(1)(iii)

h) same as g but employee is comptroller of the conglomerateexcluded – he is in all the lines of business Reg 1.132-4(a)(1)(iv)

i) employee gets 20% discount on his employer’s insurance productexcluded §132(c)(1)(b)

j) employee is salesman in home electronics store. Store has sales of $1,000,000 and $600,000 cost of goods. Employee buys $2,000 VCR for $1,000

$800 is excludable from income §§ 132(a)(2) & 132(c)(1)(a) find gross profit percentage: 1,000,000 – 600,000 400,000 1,000,000 = 1,000,000 = 40% allowable discount$2000 x .40 = $800$800 discount is excludable but this employee got $1000 discount; $200 is included in income

k) employee attends convention in another town. Employer pays all costs.Excluded §§ 132(a)(3) & 132(d)

l) employer has a bar and provides employees with free drinks at end of weekexcluded §§ 132(a)(4) & 132(e)

m) employer gives employee a case of scotch for Christmas may be excludable if of low fair market value Reg 1.132-6(e)(1)

n) employee is an officer of the corporation which pays for his parking in a facility one block away. Non-officer employees pay their own parking fees.

$175 per month is excludable §§ 132(a)(5) & 132(f)(2)(b) no anti-discrimination ruleNB: NY exception Reg 1.132-6(d)(2)(a)(iii) – allows exclusion of the excess of the value of each one-way trip over $1.50 in special circumstances, like working late & unsafe to take public transportation

o) Employer gives employee $900 in vouchers for mass transit in a year prior to 2002$120 is included in income §§132(a)(5) & 132(f)in 2002, transit voucher benefits was increased from $65 to $100 per month excludable

p) Employer puts in a gym for use of employees and their families excludable § 132(j)(4)

Page 17: Beck Fed IncomeTax Outline

a. For the convenience of the employer; (the Beck test: can you ask for the cash? In this case, no)

b. On the business premises; andc. As a condition of employment (custom of the industry or necessary for safety)

1. The terms of the employment contract are not determinative of these issues and the IRS may scrutinize the situation to determine whether the lodging is really a condition of employment and for convenience of employer or just a pretext (substance over form). Herbert G. Hatt .

2. Often, the IRS will closely scrutinize these contracts in a closely held corporation

3. In Hatt, the court determined that the petitioner had shown sufficient business reasons for the requirement in his contract - the facts and circumstances to support exclusion:1. it was the practice in the industry to have someone on premises 24

hours a day2. the phone rang in his apartment

d. § 107 – no limit on parsonage exclusion as long as it is used for housing – current case in S Ct regarding this as violative of First Amendment

Chap 5: AwardsI. Prizes § 74 see § 102(c); 132(a)(4), (e); 274(j). Reg 1.74-1

A. Non-taxable prizesGenerally prizes are taxed. For prizes unrelated to employment to be excludable from gross income, four requirements must be met.1. The prize must be made in recognition of religious, charitable, scientific,

educational, artistic, literary, or civic achievement.2. The recipient must have been selected without any action on his part to enter

the contest or proceeding.3. The recipient is not required to render substantial future services.

a. But an appearance or short speech is thought to be permitted.

Problems pg 1061) Employer provides employee and family with residence on business premises. Rental value of $5,000 per year but employee pays $2000

a) what result if employee’s work does not require employee to live on the premises as condition of employment?Employee has $3000 additional income to report

b) what result if employer and employee simple agreed that employee should live on premises?IRS would scrutinize the agreement but the outcome would likely be the same as a

c) what result if employee’s work and contract require it and employer provides $3000 of groceries during the year?

Value of premises is not included in income but the $3000 for groceries may be. Courts are split as to whether groceries are meals or not. Depends on circumstances.

2) Planner incorporates her hotel and corporation purchases residence adjacent to business premises. Corporation by contract requires Planner to live in residence and also furnishes her meals. Planner worked at hotel and was on call 24-7.

IRS will scrutinize because this is a closely-held corporation. The residence will probably qualify under § 119(a)(2). Meals will if she can prove need for emergency calls availability Reg 1.119-1(a)(2)(ii)(a)

3) State highway patrolman on duty 8am to 5pm. Eats lunch near highway. State reimburses him. Not excludable because no substantiation supplied by the employee.

Page 18: Beck Fed IncomeTax Outline

4. The prize is transferred by the payor to a governmental unit or a recognized charity pursuant to a designation made by the recipient.a. Basically, the money cannot come under the control or dominion of the

recipient.b. employee was required to go on the trip and worked while there Alle n J.

McDonell

II. Employee Achievement Awards § 74(c)Gifts to employees are not allowed under § 102(c).But Employee Achievement Awards are excludable from income if the cost to the employer of the award does not exceed $400 if not part of an established award program, $1,600 if part of an established award program.

1. § 274 sets forth the requirements for an employee achievement award:a. The award must be an item of tangible property; b. The award must be given by employer for either length of service or safety

achievement;(1) Length of service awards cannot be given within the first five years of

employment, and can be given to the same employee only once every five years. § 274(j)(4)(B).

(2) Safety achievement awards cannot have been given to more than 10% of the employees, nor given to managers, clerks, administrators, or professionals. § 274(j)(4)(C).

c. The award must be given as part of a meaningful presentation, and the award must be given under conditions that do not create a likelihood that it is disguised compensation.

2. If an award does not meet the requirements of § 74 it may still be excludable from gross income as a de minimis fringe benefit under §132(a)(4), provided the requirements of that section are met.

III. Scholarships & Fellowships §§ 117; 127(a), (b)(1), (c)(1) Reg 1.117-6(b), (c)(1)-(4), (d)(1)-(3)

A. ScholarshipAmounts received as a "qualified scholarship" by an individual who is a candidate for a degree at an educational institution are excludable from gross income. § 117(a). 1. A qualified scholarship includes any amounts received for: tuition and

enrollment fees, books, supplies and equipment. § 117(b)(2).2. Amounts received for rent (room and board) are not excludable from gross

income.3. Expressly not excluded from gross income are amounts received as payment

for teaching, research, or other services required as a condition to the

Problems pg 1121) national sportswriters give award to most outstanding amateur athlete and give winner $5000. This year’s winner delivers an appropriate rejection “acceptance” speech and donates the money to a charity.

Includable in income because this is not §74(b) award (in recognition of religious, charitable, scientific, educational, artistic, literary, or civic achievement)

Page 19: Beck Fed IncomeTax Outline

scholarship. § 117(c).a. Thus, a qualified scholarship can be granted by an employer, but only if

there is no requirement that the recipient continue to work for the employer.

b. An exception is allowed for athletic scholarships. Student can be expected to participate but not required to and school cannot take away scholarship if student does not participate.

4. Reg 1.117-6(c)(4) defines degree candidate broadly and includes primary and secondary schools, undergraduate and graduate levels.

5. Beck queries: do we even need 117a if the scholarship is coming from the school and represents either a gift or a discount to a customer (a non-taxable event)?

B. Reduction in Tuition Any reduction in tuition provided to an employee of an educational organization for education below the graduate level is excluded from gross income. (a kind of working condition fringe) § 117(d). Requirements:1. The plan cannot be discriminatory in that it is only for highly compensated

employees. § 117(d)(3). 2. The student must be the employee.

a. The term "employee" includes an employee of the institution, spouses and dependents of employees, and graduate students who perform services. § 117(d)(2).

3. The Award must be from the educational institution itself.4. § 127 permits an employee to exclude up to $5,250 per year from gross

income for amounts paid by the employer for educational assistance (related to work).

Page 20: Beck Fed IncomeTax Outline

Chap 6: Gain from Dealings in PropertyI. Factors in the Determination of Gain §§ 1001(a), (b) first sentence, (c); 1011(a) Reg 1.1001-1(a)

basis = costamount realized (A/R) – adjusted basis (A/B) = gain (which is taxable) tax cost basis – if you have to take property into income (like the stock example)

you get a basis in that property that is equal to the amount that you took into income – so that if you sell it again, you wind up in the same position as if you had paid cash for it

1. Gross income includes gains derived from dealings in property. § 61(a)(3)2. The gain from the sale or other disposition of property is the excess of the

amount realized over the adjusted basis, and the loss is the excess of the adjusted basis over the amount realized. § 1001a. The amount realized is the sum of any money plus the fair market value of

any other property received. § 1001(b).b. The adjusted basis for determining a gain or loss is the cost basis adjusted

as provided in § 1016. § 1011.(1) The cost basis is the cost of acquiring the property. § 1012.(2) Adjustments to the cost basis must be made for various deductions

allowed with respect to the property. I.R.C § 1016. 3. There is a presumption that all gains realized are recognized, but losses are not

recognized unless there is a specific section so providing.

Problems pg 1151) Student in A.B. degree program gets scholarship of $6,000 toward tuition and room and board. Tuition is $3000 and room and board costs $3000. Student is required to do 300 hours of research for professor. Other non-scholarship students get paid $10 per hour.

a) what tax to student? No exclusion under § 117(c) Reg 1.117-4(c)(1) Or argue that $3000 is for room and board so $3000 is excludedOr allocate ½ to each, so $4500 is taxable and $1500 is excluded

b) What if all students are required to do 300 hours of research for professors?No difference, still not excludable

c) What if $6000 is athletic scholarship with no work requirement?Excludable § 117(c)

d) what if student receives $2500 tuition scholarship. Student’s spouse is employed by neighboring education institution and tuition sholarship is part of a nondiscriminatory plan between the institutions

excludable § 117(d)(2)(b) & § 117(d)(3)2) Lawyer receives $10,000 stipend to assist her while on leave to obtain her L.L.M. Stipend is part of firm plan requiring return to firm following their educational leave.

a) what tax?Exempt to $5250 § 127 ($4750 included in income)

b) what if she is not required to return to firm?Could be considered compensation for past service Excludable up to $5250 of related costs

c) What if she is not an employee but instead receives the stipend as a prize in an essay contest?Taxable under § 74 but § 74(a) excludes scholarships – all scholarships are competitively awarded and could be considered prizes – Beck says find the narrowest category. This could be considered a scholarship and therefore excludable.

Page 21: Beck Fed IncomeTax Outline

II. Determination of Basis §§ 109; 1011(a); 1012; 1016(a)(1); 1019 Reg 1.61-2(d)(2)(i); 1.1012-1(a)

A. Cost as Basis1. The basis of property is the cost of such property – whatever you pay for

something Stock costs $10 (that is the basis – tax free return of capital; cost recovery)Sell it for $100 – gain is $90 (amount realized - taxable)3 ways to recover cost:

1. Can be recovered right now by deducting it (reducing income)2. Can be recovered when the property is sold (capital gains) 3. Can be recovered in dribbles by depreciating real property (amortizing

intangible property) for things having useful life (land and stocks/bonds cannot be depreciated because they last forever)

2. When property is acquired in a barter transaction, the basis in the acquired property is its fair market value at the time of the exchange. Philadelphia Park Amusement Co. v. U.S..a. If in an arms length transaction, the acquired property cannot be valued, its

fair market value is assumed to be equal to the fair market value of the property that is given up.

b. In an exchange there are 2 issues: what are the tax consequences of the exchange? And what is the basis of the property received upon subsequent sale (which the judge says in this case is the value of the property received)

c. This is a variation of tax cost basis: whatever was taken into income becomes the basis

d. better to deduct than adjust basis because of time value of money (better to have $$ now) and it is better to reduce income than capital gains – faster depreciation is better (faster recovery of cost)

3. §§ 109 and 1019: value of improvements done by the lessor is not included in lessee’s gross income and does not increase basis - substitute for rent that is taken into income goes into basis – this effectively postpones recognition of the gain until the property is sold (not really an exclusion)

Page 22: Beck Fed IncomeTax Outline

B. Property Acquired by Gift § 1015(a) see §§ 1015(d)(1)(A), (4) & (6) Reg 1.1015-1(a)

1. If property is acquired by gift, the basis is the same as it was in the hands of the donor or last preceding owner by whom it was not acquired by gift. § 1015.a. if donor basis < FMV, then donee basis is donor basisb. if donor basis > FMV of the property at the time the gift is made, the basis

for determining loss to the donee (in the event of a subsequent sale at a loss) is the fair market value of the property at the time of the gift. (1) must have built-in loss (FMV > donor basis) & subsequent sale at a

loss (2) if sale is above fair market value and below basis, there is no gain and

no loss c. If the donor’s basis is not obtainable, the donee’s basis is the fair market

value of the property at the time acquired by the donor. d. Part sale/part gift: basis is the greater of amount paid by transferee and

transferor’s basis (1) donor has a gain if the donee payment is greater than donor basis §

1001 (no loss can be recognized in a part sale/part gift)(2) donee’s basis is the greater of donee cost and donor’s basis (the carry-

over basis)(or donor basis plus any gain recognized by donor) Reg 1.1015-4

2. Appreciationa. Any appreciation in the property that occurred while in the hands of the

donor does not affect the donor basis. Taft v. Bowers.b. For example, if the donor bought stock at a cost to him of $1,000, and the

stock is valued at $2,000 when the gift is made, the donee takes a basis of only $1,000.(1) But the donee does get a step up in basis for any taxes paid, and

attributable to the appreciation.

Problems pg 1211) Owner purchases some land for $10,000 and later sells it for $16,000

a) what gain? $ 6,000b) what if owner paid $1000 for option to purchase land later for $9000 and then later did so

basis in option is $1000, basis in land is $9000; gain is $6000c) what if owner sold the option for $1500 instead of buying the land

$500 gaind) what if owner purchased the land for $10,000, spent $2000 clearing it prior to sale and then sold it for $18,000

$2000 is adjustment to basis (unless it was deducted from income); $6000 gain e) what if owner rented the land to L for 5 years for $1000 cash per year and L spent $2000 clearing the land. Owner reported the cash rental as gross income; $2000 expenditures were properly excluded under § 109.

Still $10,000 basis & $6000 gain. Substitute for rent goes into basis but this is not rent substitutef) what if land valued at $10,000 is given to employee and his taxes go up $3000

still $10,000 basis in the landg) what if owner works in gallery and buys a $10,000 painting for $9,000 (under an employee discount program) and later sells it for $16,000

basis is $10,000 and gain is $7000 – $1000 discount in not excludable under § 132(a)(2)

Page 23: Beck Fed IncomeTax Outline

3. Basis of gift on which gift tax has been paid by donor: § 1015(d)(6): basis in a gift on which gift tax has been paid by donor has its basis increased by an amount in the same ratio of the appreciation in the value of the gift to the amount of the gifto Formula: (appreciation on gift/fair market value) x tax paid = adjustment

to basis o The more the gain, the more gift tax into basis o Eg. Relative buys apt for 20,000 – worth 30,000 – pays 6,000 gift tax –

gives to Gainer – Gainer sells to Shelterer – what is Gainer’s basis? 10,000/30,000 x 6,000 = 2,000 ; so carry over basis 20,000 plus 2,000 = 22,000

o See Reg 1.1015-54. Definition of Gift

a. To determine whether a gift has been made, courts and the IRS will focus on the substance of the transaction, not just the label given by the parties. In this case, taxpayer received stock for release of marital rights (contract sale, not a gift). Farid-Es-Sultaneh v. C ommissioner .

C. Property Acquired Between Spouses or Incident to Divorce § 1041 Reg 1.1041-1T(a) & (d)

1. The general rule is that no gain or loss is recognized on a transfer of property from an individual to a spouse, or former spouse if incident to divorce. § 1041.a. In these types of transfers the property is treated as if it were acquired by

gift.b. The basis of the transferee is the adjusted basis of the transferor.c. No sale – money is ignored and the property is simply treated as a gift d. No anti-loss rule – loss can be transferred – absolute carry over basis

2. A transfer is incident to divorce if it occurs within 1 year after the date on

Problems pg 1281) Donor gave donee property with no gift tax required. What gain or loss on subsequent sale?

a) property cost Donor $20,000, had FMV of $30,000 at time of gift and Donee sold it for:(1) $35,000: basis is 20,000; gain of $15,000(2) $15,000: basis is still 20,000; loss of $5000(3) $25,000: basis is again 20,000; gain of $5,000

b) property cost Donor $30,000, had FMV of $ 20,000 at time of gift and Donee sold it for:(1) $35,000: gain of $5000 no loss on the sale, so use donor basis(2) $15,000: loss of $5000 basis is FMV (3) $24,000: no gain or loss

2) Father bought land at $120,000; FMV now $180,000. Sold to Daughter for $120,000 cash calling the transaction part sale/part gift.

a) What gain to Father and what basis to Daughter under Reg 1.1001-1(e) & 1.1015-4?Father has gain of 0; Daughter takes Father’s basis of $120,000 and has a gain of $60,000If Father sold it to Daughter for $100,000, then Father has 0 gain, no loss and Daughter has $60,000 of gain (her basis is the greater of amount paid $100,000 and donor basis $120,000)If Daughter paid $140,000; Father has gain of $20,000 and Daughter has basis of $140,000 and gain of $40,000

Page 24: Beck Fed IncomeTax Outline

which the marriage ceases, or is related to cessation of marriage.a. It is presumed that transfers are related to the cessation of a marriage if

they occur within 6 years and are pursuant to a written decree or agreement. Treas. Reg. 1.1041-1T(b).

3. § 1041 does not cover transfers between spouses that are pursuant to a pre-nuptial agreement.

4. Policy Underlying § 1041a. Spouses file joint returns so they are 1 taxpayerb. A married couple is one economic entity, thus it makes no sense to tax a

"transfer" between the same entity.c. Spouses in different states may be treated differently, so § 1041 provides a

uniform federal law.5. § 1041 is really a deferral because there will be a gain or loss when the

transferee spouse sells to a third party.6. reverses the Davis case – big improvement because there is no new cash in the

transaction (no liquidity to pay the taxes)

D. Property Acquired From Decedent § 1014(a), (b)(1) & (6), (e) Reg 1.1014-3(a)

1. A person who acquires property from a decedent has a basis in the property equal to the fair market value of the property at the time of death. § 1014(a)(1).a. This gives a stepped-up basis to the recipient (with no income tax cost for

the appreciation while decedent owned it; you take the tax liability with you)

b. if you have loss property, sell it before you die or give it to your spousec. estate tax value is used for income tax value of property

2. But if the decedent received the property within a year of his death, and the estate transfers it back to the original owner, the original owner keeps the original basis. § 1014(a)

* If gift tax paid then need to use formula determining portion of gift tax applicable to gain that gets added to basis

II. Amount Realized § 1001(b) Reg 1.1001-1(a), -2(a), (b), (c) Examples (1) & (2)

The amount realized from the sale or other disposition of property is the sum of any money received plus the fair market value of any other property received. § 1001(b).

Problems pg 1301) Property purchased at $4000, FMV now $7000. H sells to W for $7000 cash

a) what tax? Noneb) W’s basis? $4000c) What gain to W if sold? $3000d) What result if FMV $3000 and H sold to W for $3000? $4000 basis; $1000 loss on the salee) What if W trades property with basis of $5000 and FMV of $7000 instead of cash to H?

W takes H’s basis and H takes W’s basis

Page 25: Beck Fed IncomeTax Outline

1. The relief of indebtedness is considered "other property received" § 1001.2. If you transfer appreciated property in satisfaction of a legal obligation there is

a taxable gain equal to the difference between the amount of the legal obligation and the adjusted basis in the appreciated property. Internati onal Freighting Corporation, Inc. v. Commissioner.a. For example, if you buy stock at a cost of $1,000, and a year later you

transfer the stock to cancel a debt of $2,000, you have realized a gain of $1,000.

b. When property encumbered by a non-recourse mortgage is transferred subject to the mortgage, the transferor realizes an amount equal to the unpaid balance on the mortgage. Crane v. Commissioner.(1) amount received = amount of debt offloaded + property recd + cash

amount recd – adjusted basis = gain.Beck stated the rules of Crane this way:

1. Borrowed money gives you basis in the property as long as it is invested in the property

2. Offloaded debt is part of the amount realized in the transaction (2) The transfer of a non-recourse mortgage is treated as if the transferor

had transferred a loan on which he was personally liable, no matter whether or not the property was worth less than the mortgage. Commissioner v. Tufts .

Symmetry of the transaction: if you have $1.8 of basis and 400,000 of depreciation, which you took as a deduction; when you sell for 1.4, you have 400,000 of gain on the transaction

3. If property is sold in portions, basis gets apportioned among the lots. Reg 1.61-6

Page 26: Beck Fed IncomeTax Outline

Chap 7: Life Insurance Proceeds and AnnuitiesI. Life Insurance § 101(a), (c), (d), & (g) Reg 1.101-1(a)(1), (b)(1), -4(a)(1)(i), (b)(1), (c)

A. Non-taxable proceeds The proceeds of life insurance received by reason of the insured's death are

excludable from gross income. § 101(a). Payments can be payment on death, or for the transfer of the policy or in the case

of whole life, the investment gain on the premium.

Problems pg 1531) Mortgagor buys land from Seller for $100,000. Mortgagor borrows $80,000 and pays $20,000 cash.

a) what is Mortgagor’s basis? $100,000b) 2 years later, land has value of $300,000 and Mortgagor still owes $80,000. Mortgagor borrows an additional $100,000. Is this income? Noc) What is Mortgagor’s basis if $100,000 is used to improve the land? $200,000d) What if Mortgagor uses $100,000 to buy stock and bonds? $100,000e) What if, as in d, Mortgagor sells land for $120,000 (mortgages of $180,000 and value of land $300,000). What is Purchaser’s cost basis? $120,000

f) What if, as in d, Mortgagor gives land subject to mortgages and worth $300,000 to Son. What is Son’s basis?Part gift/part sale – 2 part analysis under § 1015

Donor DoneeA/R 180,000 300,000A/B 100,000 180,000

80,000 120,000g) What if Mortgagor gives the land to his spouse? It’s a gift under § 1041. Basis is $100,000h) What if, as in d, land declines in value to $180,000 and Mortgagor transfer the land to the bank?

$80,000 of gain ($180,000 amt received by discharge of debt minus basis of $100,000)i) What if land value declined to $170,000? Same result; amount discharged is amount received

3) R purchase apt for $200,000 and gives it to G when it is worth $300,000. R paid $60,000 in gift tax. G sells to S.a) What gain or loss to G if sale is for $320,000?

Portion of gift tax applicable to gain gets added to basis. Value of gift 100,000FMV of gift 300,000 x gift tax paid 60,000 = 20,000G’s basis is 20,000 + 2000 = 22,000. Gain is $10,000NB: the more the gain, the more gift tax into basis

b) S’s basis in the apt is $32,000c) What if R paid $8000 cash and $12,000 mortgage for apt and paid $3000 gift tax? Gave it to G. S took apt for $20,000 cash subject to the full mortgage. First calculate adjustment to basis for gift tax: (10,000/18,000) x 3000 = 1667

R G SA/R 12,000 32,000A/B 20,000 21,667 32000

(8,000)1015(d)(6) – built in gain

Page 27: Beck Fed IncomeTax Outline

101(a) – tax free if in payment for the death of the insured 101(a)(2) – if transferred for valuable consideration after the policy is issued, not

excludable (buyer’s basis is always excludable, but not the gain)o 2 exceptions:

if it was a gift (carry over basis) or transferred to spouse § 101(2)(A)

if it was transferred to the insured, partner of insured, to partnership in which insured is partner or to a corporation in which insured is a shareholder or officer (key man insurance) § 101(2)(B)

(if the transfer is to someone who could have bought the policy in the first place)

1. The proceeds are excludable only if the insured dies.a. Amounts received under a life insurance contract on the life of an insured

who is terminally or chronically ill are treated as if received by reason of death. § 101 (g)(1)(A)(B).(1) For the chronically ill, only life insurance payments sufficient to cover

the cost of medical expenses incurred are treated as if received by reason of death; for the terminally ill, all monies received under a life insurance policy are excludable from gross income. § 101(g)(3).

b. If a life insurance policy is sold to a viatical settlement provider (a person who is licensed in the business of trading life insurance contracts) by a terminally or chronically ill person, the amount realized from the sale is excludable from gross income. § 101(g)(2).(1) Note that the viatical settlement provider does not get to take

advantage of the § 101 exclusion when the insured dies2. Eligible Beneficiaries

a. Any named beneficiary may take advantage of exclusion.b. If a corporation takes a life insurance policy out on a key employee, it may

exclude the proceeds received by reason of the employee's death.B. Interest payments

Interest payments made on the amount excludable under § 101 are gross income. § 101(c).1. This rule anticipates a situation where the beneficiary leaves the policy

amount with the insurance company, and relegates himself to receiving only interest payments.

C. Payments over timeA beneficiary who takes life insurance payments over time, may exclude from gross income an amount that is apportioned equally over the amount of years the payments are to be made; any payment received above that amount is gross income. § 101(d).1. For example, if a beneficiary is entitled to a $100,000 life insurance payment,

but decides to take an annuity under which the insurance company will pay $5,000 for the next 25 years, the amount that the beneficiary may exclude from gross income each year under § 101 is represented by the fraction $100,000/25yr, or 4k/yr.

Page 28: Beck Fed IncomeTax Outline

2. If the beneficiary receives payments beyond the expected term of the annuity, he may still exclude the same fractional amount.a. But if the beneficiary dies before the end of the expected term, the estate

does not get to deduct a loss on the un-recovered portion.b. under new rule which applies if you buy an annuity not related to a death

benefit, you track the basis and payments beyond basis are fully taxable and death before full re-payment of basis is deductible back 2 years (§ 172 look backs on business expenses)

II. Annuities (the new rule) § 72(a), (b), (c) Reg 1.72-4(a), -9(table V)1. Any portion of the annuity payment attributable to the investment is excluded

from gross income. § 72(b).a. The portion of the annuity payment which is excludable from gross

income is determined by the following ratio: Investment Amount/Expected Return.(1) The expected return is the product of each payment amount and the

number of payments (Payment amount x No. of payments).(2) Beck’s formula: benefit/life expectancy = non-taxable portion (keeps

track of basis) b. The exclusion is only allowed until the investment is fully recovered, after

which all payments are subject to full taxation. § 72(b)(2).(1) If the annuitant dies prior to the termination of the payments, the

annuitant's estate is allowed a deduction equal to the un-recovered portion of the investment. § 72(b)(3)(A).

Problems pg 1581) Insured died with policy to pay Beneficiary $100,000. What tax under several alternatives?

a) $100,000 cash: no tax under § 101(a)b) beneficiary leaves $100,000 with company and they pay her $10,000 interest this year

$10,000 income § 101(c)c) elects payment of $12,000 per year for life expectancy of 25 years

divide basis by life expectancy to get non-taxable$100,000 / 25 = 4,000 non-taxable$12,000 – 4000 = $8000 taxable

d) if she lives beyond her life expectancy, the result is the same2) Corporation buys $1 million life insurance on employee. Employee dies, benefit to corporation.

a) paid for by corporation: tax freeb) bought by employee and then sold to corporation for $20,000:

$980,000 not excludable; basis is excluded § 101(a)(2)(b)c) if insured is shareholder, then all is excludable

3) Insured buys $100,000 life insurance for $40,000a) sells policy to child for $60,000 and on death $100,000 is paid to child

insured is taxed on $20,000 gainchild is taxed on $40,000 § 101(a)(2)

b) transfer to spouse: no tax § 1041c) insured sells to viatical settlement because he is terminally ill

no tax to insured § 101(g)buyer profit of $20,000 is taxable

Page 29: Beck Fed IncomeTax Outline

2. Where the annuity contract calls for a refund of any investment (premiums) not recovered by the annuitant, the value of any potential refund (determined by the annuitant's life expectancy) is subtracted from the "investment in the contract," decreasing the excludable ratio. § 72(c).

a. The refund itself, however, is not taxed, for it is merely a return of capital.Chap 8: Discharge of IndebtednessI. Included in income §§ 61(a)(12); 102(a); 108(a), (b)(1)-(3), (d)(1)-(5), (e)(1) & (5); 1017(a), (b)(1), (2), (3)(A) & (B) Reg 1.61-12(a); 1.1001-2(a), 2(c) Ex. 8

Gross income includes income from the discharge of indebtedness. 61(a)(12). If you settle a legal obligation for less than the amount of the obligation, the

difference becomes taxable income. When you borrow money, you are not taxed on it because you have to repay it.

But if you don’t have to repay it, then you are taxed on the amount you have been forgiven, in the year in which it is forgiven.

Not all discharges of debt result in taxable income – look to the economic substance: forgiven loan to employee is compensation (and subject to payroll taxes which discharge of debt would not be) – loan to child forgiven is a gift – loan to shareholder which is forgiven is a dividend – this is called spurious discharge of debt

To be taxable, debt which is discharged must be from a loan of money – not just the avoidance of a loss Eg: Mom lets Dad out of child support payment (not a loan) so not discharge

of indebtedness No income if taxpayer is insolvent or in chap 11 – but if taxpayer has tax

attributes, in lieu of forgiveness, these will be reduced For example:

1. A company that issues bonds with a face value of $100 and retires those bonds by repurchasing them for $95 each, recognizes gross income of $5 per every bond repurchased. U.S. v. Kirby.a. This case is generally cited as defining discharge of debt as creating income.

2. The settlement for less than face value of an unenforceable obligation is not gross income. Za rin v. Commissioner . result is correct; reasoning is wrong debt is unenforceable, so there is no debt; but Beck disagrees with this and

says there is still debt Beck says: avoidance of a loss is not taxable as a gain

Problems pg 1631) $48,000 policy with $3000 per year benefit over 24 year life expectancy

a) what is taxable? 48,000/24 = 2000 tax free; $1000 is taxableb) what is taxable in the 30th year of payments? $3000 taxable; basis has been exhaustedc) insured dies after 9 years of payments. What tax deduction to estate?

9 x 2000 = 18,000 48,000 – 18000 = 30,000 unrecovered basis, deductible loss on last return d) 76,500 cost for $3,000/year with life expectance of 34 years. What is taxable?

76,500/34 = 2250 non taxable $750 is taxable

Page 30: Beck Fed IncomeTax Outline

o Zarin is no better off than before the forgiveness – it is a nullity for tax purposes

o this is a return of capital: allowing loser to collect from winner what was formerly his – avoidance of loss is not taxable

o there was never any loan of money (casino had never given him anything to spend, only a free line of credit to gamble with)

Don’t cite Zarin for anything – while the result is right, the reasoning is wrong

II. Exceptions to 61(a)(12)A. Under § 108(a)(1) there are 4 exception to the general rule that discharge of

indebtedness is includable in gross income.1. There is no gross income if the discharge occurs in bankruptcy. 108(a)(1)

(A).2. There is no gross income if the discharge occurs when the taxpayer is

insolvent. 108(a)(1)(B). a. The amount of the exclusion is limited to the extent of the taxpayers

insolvency. Peculiar definition of insolvency for this section is that taxpayer’s liabilities exceed assets (usual definition is the inability to meet obligations as they mature)

b. judicially developed insolvency exception: no income arises from discharge of indebtedness if the debtor is insolvent both before and after the transaction; and if the transaction leaves the debtor with assets whose value exceeds remaining liabilities, income is realized only to the extent of the excess.

3. There is no gross income if the indebtedness discharged was incurred directly with the operation of a farm. 108(a)(1)(C).

4. There is no gross income if the debt was incurred in connection with real property used in a business or trade. 108(a)(1)(D).

B. Under § 108(b)(2), a taxpayer who elects to exclude a discharge under sections 108(a)(1)(A), (B), or (C), must reduce tax attributes (in lieu of tax) in the following order:

1. Net Operating Loss2. General Business Credit3. Minimum Tax Credit4. Capital Loss Carryovers5. Basis in the property of the taxpayer6. Passive activity loss and credit carryovers.

C. The discharge of a debt owed to the seller of property (a purchase-money debt) is treated merely as a retroactive reduction in price by the seller (not as income). § 108(e)(5)1. debtor must also reduce his basis in the property in an amount equal to that of

the discharge.D. discharge of indebtedness which constitutes a gift or bequest is not treated as

income. § 102.

Page 31: Beck Fed IncomeTax Outline

Problems pg 1801) Poor borrowed $10,000 from Rich several years ago. What tax consequences to Poor if Poor pays off the so far undiminished debt with :

a) A settlement of $7,000 of cash? $3000 taxable as debt dischargeb) A painting with a basis and fair market value of $8,000? $2000 taxable as debt dischargec) A painting with a value of $8,000 and a basis of $5,000? $5000 taxable (3000 capital gain & 2000 debt discharge)d) Services, in the form of remodeling Rich’s office, which are worth $10,000?

$10,000 taxable; these are payment for services rendered (a barter transaction reduced to cash) and is considered spurious forgiveness of debt

e) Services that are worth $8000? $8000 is payment and $2000 is debt dischargef) Same as (a) above, except that Poor’s Employer makes the $7,000 payment to Rich, renouncing any claim to

repayment by poor. $7000 is payment and $3000 is debt discharge2) Mortgagor buys land for investment from Seller for $100,000 with $20,000 of cash and $80,000 mortgage. Land increases in value to $300,000, Mortgagor borrows another $100,000 from the Bank. Mortgagor uses the $100,000 of loan proceeds to purchase stocks and bond. Several years later when the principal amount of the mortgages is still $180,000, the land declines in value to $170,000. Mortgagor transfers the land to the Bank, and the Bank discharges all of the Mortgagor’s indebtedness.

a) What are the tax consequences to Mortgagor ? See Reg 1.1001-2 (a) and 2 (c) Example 8A/R 180,000 discharge of debtA/B 100,000

80,000 gain taxable to taxpayer, in a non-recourse mortgagebut in a recourse mortgage, it looks like:A/R 170,000A/B 100,000

70,000 gain 10,000 debt discharge income (bank can only collect

3) Businessman borrows $100,000 from Creditor to start an ambulance service. He then purchases ambulances for use in his business at a cost of $100,000. Assume the ambulances are his only depreciable property and, unrealistically, that after some period of time their adjusted basis and value are still $100,000. What consequences under Section 108 and section 1017 in the following circumstances:

a) Businessman is solvent but is having financial difficulties and the Creditor compromises the debt for $60,000$40,000 debt discharge income

b) Same as (a) above, except that Creditor is also the ambulance dealer who sold the ambulance to the businessman and, as a result of depreciation deductions, the adjusted basis of the ambulance is $35,000.

§ 108(e)(5) applies: original purchase nowA/B 100,000 60,000 basis is reduced to reflect debt dischargeDeprec 65,000 65,000New A/B 35,000 (5,000)

taxpayer is taxed on $5000 depreciation recapture income (negative basis is not allowed)

Page 32: Beck Fed IncomeTax Outline

Chap 9: Damages And Related ReceiptsDamages for physical injury and sickness are accorded express congressional treatment in the Code, but other types of damages must be dealt with by courts under general tax principles.

I. Damages for Business Loss for lost profits, damages are an income substitute and are taxed as ordinary

business income for destruction of property (including goodwill), an award of damages is

treated like an amount realized from a forced sale (and the basis is reduced by the amount of the award; if excess, that amount is taxable)

Damages are taxable as gross income if they are recovered in lieu of taxable income. Raytheon Production Corporation v. Commissioner. 1. For instance, if a plaintiff recovers damages attributable to lost profits, the

recovery is taxed as if it were profits; but if damages are awarded for a loan default, the damages attributable to the principal are not taxed, because that is a mere return of capital.

c) same facts as in (a) above, except that Businessman is insolvent and his liabilities of $225,000 exceed his assets (the ambulance worth $100,000) by $125,000. Further assume Businessman has no net operating losses, general business credit carryovers, minimum tax credit, capital loss carryovers, passive activity losses or credit carryovers, or foreign tax credit carryovers. The Creditor discharges $40,000 of the $100,000 loan without any payment.

Liabilities Assets InsolvencyOriginal situation: 225,000 100,000 (125,000)After discharge: 185,000 100,000 (85,000)Taxpayer is protected by §108(a) because he is insolvent before and after the discharge so he has no income

d) Same as (c), above except that Businessman has a $30,000 net operating loss.Since there is $40,000 of income forgiven but not taxed under § 108(a), you loose $ for $ tax attributes up to the amount of income that was not taxed. Here he looses the full $30,000 of loss carry forward.

e) Same as (c), above except Businessman’s liabilities exceed his assets by $25,000.Liabilities Assets Insolvency

Original situation: 125,000 100,000 (125,000)After discharge: 85,000 100,000 15,000Insolvent before but solvent afterward. Taxpayer has $15,000 income from debt discharge.

4) Decedent owed Friend $5000 and Nephew Owed decedent $10,000.a) At decedent’s death friend neglected to file a claim against Decedent’s estate in the time allowed by state law and

Friend’s claim was barred by the statue of limitations. (Let us defer our concern for nephew). What results to Decedents Estate? $5000 income

b) What result to the estate in (a) above (with nephew still in cold storage ) if instead Friend simply permitted the statue to run stating that she felt sorry for Decedent’s widow, the residuary beneficiary of his estate.

Could be called a gift under § 102 and not taxedc) Now, what result to Nephew if decedent’s will provided that his estate not collect Nephew’s debt to the estate.

Inheritance under § 102 and not taxed

Problems pg 1821) Plaintiff brought suit and unless otherwise indicated successfully recovered. Discuss the tax consequences:

a) suit was based on a recovery of $8000 loan to Debtor. Plaintiff recovered $8500 cash, $8000 for the loan plus $500 interest. $8000 is return of capital, $500 is income and taxableb) What result to Debtor if he instead transferred land worth $8500 with basis of $2000 to satisfy obligation? What basis does Plaintiff take?

Debtor has realized gain of $6500 but can deduct $500 as an interest expenseCreditor takes a tax cost basis of $8500 and is taxed on $500 of income

c) suit was based on breach of contract and Plaintiff recovered $8000 in lost profits and $16,000 in punitive damages - all of it is taxable as ordinary income (Glenshaw Glass)d) suit was based on claim of injury to goodwill of business arising from breach of contract. Plaintiff had $4000 basis for goodwill & valued at $10,000 at time of breach.

(1) settled for $10,000 for goodwill destroyed: $4000 return of capital & not taxable; $6000 taxable as gain(2) recovers $4000 for partially destroyed goodwill, now worth $6000: $4000 is return of capital, no tax leaves $2K deferred

gain(3) recovers $3000, goodwill worth $7000 after breach: Basis $4000 - A/R $3000 leaves $1000 basis; leaves $6000 deferred gain

Page 33: Beck Fed IncomeTax Outline

II. Damages & Recoveries for Personal InjuryA. Damages § 104(a) Reg 1.104-1(a), (c), (d)

Any damages, except for punitive damages, received for physical injury or physical sickness are excludable from gross income. § 104(a)(2)1. Recovery for emotional distress is excluded if the emotional distress is

incurred on account of physical injury, and damages are for medical care received attributable to emotional distress.

2. Punitive damages for wrongful death are excluded if they are the only form of wrongful death recovery available. § 104(c).

3. If personal injury settlement provides for annual payments that include interest, interest is also excludable under § 104(a)(2) if settlement does not mention interest. Rev. Rul. 79-313.a. The rationale is that the interest is mere compensation for the present

value of money, which would not be taxed if the settlement payment were in one lump sum.Problem: Offer to settle Personal Injury case:

$1 million cash now would be tax free under 104(a)(2) $2 million over 20 years is also all tax free

Jury Award $1,000,000.00 for lost earnings – Tax-free - Return of loss human capital what is lost is not earnings but the ability to earn

B. Insurance and Workers' Compensation §§ 105(a)-(c) & (e); 106(a) Reg 1.105-1(a); 1.106-1

1. Employer's contributions to accident or health plans are not included in gross income. § 106(a). § 106 complements § 105 by excluding from gross income the value of health and accident insurance premiums paid by the employer to cover the employees.

2. Gross income does not include amounts attributable to work related injuries or sickness that are paid under a workmen's compensation act. § 104(a)(1).

3. Gross income does not include amounts received under accident or health insurance policies for personal injury or sickness paid for by insured without employer contributions. § 104(a)(3)a. excludes all payments received, even if the insured recovered under

separate policies an amount greater than his medical expenses. (think AFLAC)(1) Interaction of §§ 104(a)(3) & 105(b): If an individual has two accident

and health insurance policies, the premiums of one being paid by the individual and the other by the employer and both compensate individual for the same illness then allocation of benefits has to occur. Under § 104(a)(3), all proceeds from the individual’s policy are excludable from income, but § 105(b) limits the exclusion for the employer-provided policy to amounts that reimburse the individual for medical care. In the case of payments received by both policies, the amount excludable under § 105(b) is the proportionate share of benefit received to medical expenses incurred.

Page 34: Beck Fed IncomeTax Outline

Amount paid by employer’s policy § 105 Total Paid by insurance §§ 104 & 105 x expenses = excludable under 105 (b)

4. Gross income includes amounts received by an employee under insurance for personal injuries or sickness to the extent such amounts are attributable to employer contributions. § 105(a).a. But employer-provided reimbursements for medical expenses are

excludable from gross income. § 105(e) gives companies who self-insure same treatment as those buying a plan.

b. Payments received through health or accident insurance provided by an employer for loss of a body member or body function, or disfigurement are not included in gross income if the payments are computed with regard to only the nature of the injury and not the absence from work. § 105(c).(1) A worker will often receive payments of this type under workmen's

compensation, which are excludable under § 104(a)(1), but § 105(c) excludes from gross income payments for non-work-related disfigurement.

5. All exclusions under §§ 104 and 105(b) are restricted by including in gross income reimbursements of medical expenses that were deducted in a prior year.

Page 35: Beck Fed IncomeTax Outline

Problems pg 1901) Plaintiff brought suit and successfully recovered in the following situations. Discuss the tax consequences to Plaintiff.

a) P, a professional gymnast, lost the use of her leg after a psychotic fan assaulted her with a tire iron. P was awarded damages of $100,000. tax free and not included in gross income. § 104(a)(2)b) $50,00 of the recovery in (a), above is specifically allocated as compensation for scheduled performances P failed to make as a result of the injured leg.

tax free and not included in gross income. This may look like payment for services but it is actually compensation of the loss of ability to earn.

c) The jury also awards P $200,000 in punitive damages. This is not excludable from gross income and is taxable. Under § 104(a)(2) and Glenshaw Glass punitive damages are includable in gross income and are therefore taxable

d) The jury also awards P damages of $200,000 to compensate for P’s suicidal tendencies resulting from the loss of the use of her leg.

This is tax free and not includable in gross income because the emotional injury for which P is being compensated is a direct result of a physical injury. A taxpayer can exclude a recovery for emotional distress only if the emotional distress is the byproduct of physical injury.

e) P in a separate suit recovered $100,00 of damages from a fan who mercilessly taunted P about her unnaturally high squeaky voice, causing P extreme anxiety and stress.

taxable because it is an emotional injury and does not stem from a physical injuryf) P recovered $200,000 in a suit of sexual harassment against her former boss.

taxable. This is not a physical injury. There may be touching in this sexual harassment suit but this touching is not central to the injury. If there was harmful physical touching then it would be excluded.

g) P dies as a result of the leg injury and P’s parents recover $1,000,00 of punitive damages awarded in a wrongful death action under a state statue.

Wrongful death. The one exception to this rule is that punitive damages received for wrongful death, under a state statue that does not allow for compensatory damages, will be excludable. § 104(c)(1) “the phrase, other than punitive damages will not apply to punitive damages awarded in a civil action”

2) Injured and Spouse were injured in an automobile accident. Their total medical expenses incurred were $2,500a) year of accident, deducted $1,500 on joint return and filed suit against the wrongdoer. In next year, they settled their claim for $2,500. What income tax?

$1000 excludable & $1500 is income under § 111: Tax Benefit Rule: a recovery from a previously deducted item becomes income and taxable (to avoid a double tax benefit for the same item).

b) Spouse was ill but, fortunately, they carried medical insurance & additionally Spouse had insurance benefits under a policy provided by Employer. Spouse’s medical expenses totaled $4,000 and they received $3,000 of benefits under their policy and $2,000 of benefits under Employer’s policy. To what extent are their benefits included in their gross income? $2,000 $2000 + $3000 x $4,000 = $1600 exempt & $400 taxable; $3000 from their policy is not taxedc) Under the facts of (b), above, may injured spouse deduct their medical expenses?

not deductible, because the couple did not pay their medical expenses; the insurance company did. § 213(a) states “there shall be allowed as a deduction the expenses paid during the taxable year, not compensated by insurance or otherwise, for the medical care of the taxpayer…”

3) Injured who has 20 year life expectancy, recovers $1 million in a personal injury suit arising out of a boating accident.a) What are the tax consequences to the injured if the $1 million is deposited in a money market account that pays 5% interest? Principal is not taxable but the interest earned is taxableb) What are the tax consequences to injured if the $1 million is used by injured to purchase an annuity to pay injured $100,000 a year for life?

Invested amount/expected return = portion excludable1,000,000/20 = 50,000 per year of basis from the recovery is tax –free and 50,000 is taxable as profit..

c) What are the tax consequences to Injured if the case was settled and in the settlement, Injured received payments from defendant of $100,000 a year for life?

Not taxable. Even though economically identical to above. If you don’t use word interest and don’t buy annuity, it is tax free. There are periodic payments

Hypo on Damages:Tax Lawyer gives client bad advice to file jointly as opposed to separately; taxpayer paid $25,000 more than if he had filed separately. The lawyer offers to pay the client $25,000. The tax collector calls this income?Result: The court ruled that the lawyer caused the client an injury which caused the client to lose money so therefore it is not taxable and the client is no richer. Old Colony Trust does not apply, because here it was a return of capital..

Page 36: Beck Fed IncomeTax Outline

Chap 10: Separation And DivorceI. Alimony and Separate Maintenance Payments

A. Direct Payments §§ 71; 215(a) & (b); 7701(a)(17) Reg 1.71-1T(a) & (b)

1. Alimony payments are included in the gross income of the recipient, and deducted from the gross income of the payor. §§ 71(a) & 215 (act as an income splitting device).

2. For a payment to be considered "alimony" for tax purposes, five requirements § 71(b)(1):a. The payment must be in the form of cash.

(1) Checks and like instruments are considered cash, but promissory notes are not.

b. payment must be to ex-spouse (or on behalf) pursuant to a divorce/separation instrument.(1) A divorce/separation instrument includes a decree of divorce or

separate maintenance or a written instrument incident to such a decree. § 71(b)(2)(A).

(2) A written separation agreement also qualifies as a separation instrument. § 71(b)(2)(B).

(3) A decree for support is considered a separation instrument. § 71(b)(2)(B).

c. The divorce/separation instrument cannot designate payment as other than alimony.(1) The parties are permitted to characterize "alimony" as something else

(non-deductible/ non-includable) in order to forego income splitting. § 71(b)(1)(B)

(2) payment is not for child supportd. The parties may not live in the same household if they are divorced or

legally separated under 71(b)(2)(A) a divorce decree or settlement.e. The liability for alimony payment must cease upon the death of the

recipient, and there may be no other payment obligation thereafter. § 71(b)(1)(d)(1) The instrument cannot require alimony for a specific period of time if

there is no provision providing for the cessation of alimony upon death of the recipient spouse.

(2) But if local law requires that alimony cease upon death of the recipient, then no provision for cessation is needed in the instrument.

3. Property Settlements and Recapturea. Although not expressly mentioned anywhere in the Code, § 71 is intended

to apply only to payments for spousal "support."b. To prevent parties from characterizing property settlements as alimony

through the use of large cash payments, the Code provides for the recapture of excess payments made in the first few years following the divorce or separation i.e., "front loading." § 71(f).(1) the front loading provisions prevent a party from making enormous

"alimony" payments the first two years after the divorce or separation,

Page 37: Beck Fed IncomeTax Outline

with a substantial drop off in the third year; 15k grace allowed each year

c. Determining the Amount of Recapture(1) First, calculate the excess payment made in the second post-separation

year. § 71(f)(4).(2nd year payment) - (3rd year payment + $15,000) = Excess payment

year 2(2) Second, calculate the excess payment made in the first post-separation

year. § 71(f)(3).(2nd year - 2nd year excess) + 3rd year

(1st year) - 2 + $15,000

(3) Finally, add the excess payments for both the first and second post-separation years.(a) included in third year as gross income to payor, and deducted by

from gross income by recipient§ 71(f) Calculation of Recapture

Year Amount Step1 80K 2 80 - [(80 – 35 + 30)/2 +15] =

80 – 52.5 = 27.52 80K 1 [80 – (30 +15) = $353 30K

Page 38: Beck Fed IncomeTax Outline

Steps:1. Determine Year 2 excess [year 2 – (year 3 + 15)]2. Determine Year 1 excess {year 1 - [(Average of year 2 adjusted + year

3) + 15]}3. Total recapture = Year 1 excess + Year 2 excess

35.0 + 27.5 = 62.5 NB: If the amounts paid in year 1,2, and 3 are within 15K of each other there

will be no recapture BUT if there is a steady decline of 15K, there will be recapture of 7,500 in year 3

d. There are three exceptions to recapture. 71(f)(5). (1) If the divorce or separation instrument ties payments to some form of

variable income, over which neither party has control, alimony payments are allowed to fluctuate beyond the limits permitted by the recapture provisions. §71(f)(5)(c)

(2) Recapture does not apply if the payments cease before the third post-separation year due to the death or remarriage of the parties. §71(f)(5)(a)

(3) Recapture is also inapplicable if the support payments are made under a decree that is not incident to a divorce or separation. (In other words, payments made pursuant to a decree of separate maintenance)

Problems pg 2031) Determine whether the following payments are accorded ‘alimony or separate maintenance” status and therefore and

includible in the recipient’s gross income under § 71(a) and deductible under § 215(a). Unless otherwise states, Andy and Fergie are divorced and payments are called for by the Divorce Decreea) The divorce decree directs Andy to make payment of $10,000 per year to Fergie for her life or until she remarries.

Andy makes a $10,000 cash payment to Fergie in the current year. Yes, this is deductible alimony. b) Same as (a) above, except that Andy transfers his $10,000 promissory note to Fergie.

not alimony. A promissory note is not cash. It is a promise to pay. Alimony must be cash . § 71(b)(1)c) Same as (b), above, except that Andy transfers a piece of art work, having a fair market value of $10,000.

No this is not alimony. This is not cash. This would be treated as a gift under § 1041d) Same as (a), above, except that in addition the decree provides that the payments are nondeductible by Andy and

are excludible from Fergie’s Gross income. Yes this is alimony. “Opt-out” of alimony is effective by saying so

e) Would it make any difference in (d), above, if you learned that Andy anticipated that he would have little or no taxable income in the immediate future, making § 215 deduction practically worthless to him, and as a consequence of this, agreed to the “non-deductibility” provision in order to enable Fergie to taxes on the payments? No difference here except tax motivation

f) What result in (a), above if divorce decree directs Andy to pay $10,000 cash each year to Fergie for 10 years? not alimony because it is requiring Andy to pay Fergie for 10 years w/o including a provision that if Fergie dies he doesn’t have to pay anymore. Alimony must end at the death of the payee. § 71(b)(1)(D)

g) Same as (f), above, except that under local law Andy is not required to make any post-death payments.Then it would be alimony

h) Same as (a), above, except the divorce decree directs Andy to pay $10,000 each year to Fergie for a period of 10 years or her life, whichever ends sooner. Additionally, the decree requires Andy to pay $15,000 cash each year to Fergie or her estate for a period of 10 years. Andy makes a $25,000 cash payment to Fergie in the year.

$10,000 alimony$15,000 not alimony because payment continues after death to her estate

i) Same as ( a), above, except that at the time of payment, Andy and Fergie are living in the same house. Not alimony. “A payment made at the time when the payor and the payee spouses are members of the same household cannot qualify as alimony or separate maintenance if the spouses are legally separated under a decree of divorce or separate maintenance” § 71(b)(1)(C) & Regs 1.71-IT (A-9)

j) Same as (i), above, except that Andy and Fergie are not divorced or legally separated and the payments are made pursuant to a written separation agreement instead of a divorce.

If there is a written separation and the parties are living in the same household this still qualifies as alimony. If there is a non-final order you can live in the same household. If the order is final the parties can’t live in the same household

Page 39: Beck Fed IncomeTax Outline

B. Indirect payments § 71(b)(1(A) Reg 1.71-1T(b)(q 6 & 7)1. Alimony payments made to third parties are anticipated by § 71, for it requires

that alimony be "received by (or on behalf of) a spouse." a. potential for abuse arises when payor controls payment and receives a

benefit therefrom.(1) eg, a person making alimony payments can make payments to payee's

2) divorce decree requires Tina to make the following payments (which meet all the requirements of section 71 (b) to Ike:Year 1 $80,000Year 2 $40,000Year 3 $10,000

a) What are the tax consequences of these payments to Tina and Ike?Step 1: Year 2 - (Year 3 + 15) 40 – (10 +15) = 15Step 2: Year 1- [(Year 2 (adjusted) + Year 3)/2] +15 80 - [ (40 - 15) + 10)/2 ] + 15 = 47.5 Step 3: Year 2 excess (from step 1) + Year 1 excess (from step 2)

15 + 47.5 = 62.5 total excess to be recaptured Tina must declare $62.5k income in year 3 and Ike can take it as a deduction from income

b) What results if the payments areYear 1 $80,000Year 2 $70,000Year 3 $60,000 1) 70 – (60 + 15) = -5 no excess (0 and minus amounts don’t count)

2) 80 – (70 + 60/2) + 15 = 0 no excess No recapture c) What results if the payments are

Year 1 - $30,000 Year 2 - $40,000 Year 3- $80,000 Payments are increasing. § 71(f) does not apply.d) What results if the payments are:

Year 1 $80,000Year 2 $50,000Year 3 $80,000 1) 50 – (80 + 15) = - 45 no excess

2) 80 – (50 + 80/2) + 15 = 0 No recapture e) Suppose that instead of requiring Tina to make payments set forth in (a) above, the divorce decree requires Tina to

pay Ike 50% of the net income (before taxes) of her oil business for three years. The payments above represent 50% of the net income from the oil business for the respective years. What tax consequences to Tina and Ike?

First and foremost, this is OK. There can’t be property settlement if you have fluctuating payments not in control of the payor spouse. § 71(f)(5)(c) says that if fluctuating payments are not in control of the payor spouse it is deductible as alimony and no recapture is taken.

f) What are the tax consequences if the decree instead provides for the level payments of $80,000 per year for three years, but Ike dies at the end of year 2 and the payments terminate at that time according to the express provisions of the instrument? § 71(f)(5)(a): the recapture rules do not apply because of death or remarriage of former spouse

g) What result in (a), above, if the payments are made pursuant to a § 71(b)(2)(c) decree for support?§ 71(b)(2)(c) describes an order of temporary maintenance, one of the exceptions to recapture

h) Tina and Ike are allegedly divorced and live in the same household in year one. Tina moves to a new Apartment at the beginning of year two. Under the divorce decree, Tina makes payments to Ike of Year 1 $120,000Year 2 $80,000Year 3 $70,000Year 4 $60,000What results to Tina and Ike in each of the years?

Year 1 is not treated as alimony because they are living in the same household. §§ 71(f)(6) & 71(b)(1)(c)Years 2 – 4 result in no recapture as in problem 2(b) above

Page 40: Beck Fed IncomeTax Outline

landlord, but not if the payor is the landlord or owns an interest in the property. Reg. 1.71-1T.

b. Payments made on a life insurance policy by the payor spouse will likely be considered alimony if the payee spouse owns the policy and is the irrevocable beneficiary. I.T. 4001.

II. Property Settlements § 1041; see 1015(e) Reg 1.1014-1T(b)Property settlements pursuant to a divorce are tax neutral events (gifts). § 1041.

1. A transfer is incident to divorce if it occurs within 1 year after the date on which the marriage ceases, or is related to cessation of marriage.a. presumed that transfers are related to the cessation of a marriage if they

occur within 6 yrs and pursuant to written decree or agreement. Reg.1.1041-1T(b) (rebuttable presumption)(1) after 6 yrs presumed to be not incident to the divorce. (also a

rebuttable presumption)

Problems pg 2071) Tom and Nicole are divorced. Pursuant to their written separation agreement incorporated into their divorce decree, Tom

is required to make the following alternative payments which satisfy the Section 71 (b) requirements. Discuss the tax consequences to both Tom and Nicole.a) Rental payments of $1000 per month to Nicole’s landlord.

This is alimony. The H has no interest in the property $1000 income to Nicole and $1000 deduction to Tom § 71(b)(1)(a)

b) Mortgage payments of $1000 per month on family home transferred outright to Nicole in the divorce proceedings.This is alimony. The H has no interest in the property.

c) Mortgage payments of $1000 per month as well as real estate taxes and upkeep expenses on the house where Nicole is living which is owned by Tom. This is not alimony. The H has an interest in the home.

2) Roseanne agrees to pay Tom $15,000 a year in alimony until the death of either or the marriage of Tom. The alimony satisfies § 71(b) requirements. After 3 years, Tom is concerned about Roseanne’s life expectancy and they agree to reduce the alimony amount to $10,000 a year if Roseanne provides Tom $100,000 of life insurance on her life.a) What are the tax consequences to Tom and Roseanne if Roseanne purchase a single premium $100.000 policy on

her life for $60,000 and she transfers it to Tom? she is transferring the policy. Alimony has to be in cash. The policy is property § 1041 and not cash

b) What result if Roseanne instead pays Tom $60,000 cash and he purchases the policy for $60,000? Alimony because it is in cash and not subject to recapture because after 3 years

c) What result if Roseanne buys an ordinary policy on her life for $5,000, transfers it to Tom, and agrees to transfer $5,000 cash to him each year so he can pay the annual premiums on the policy?

The policy is property, not alimony; The payments each year are alimonyd) Same as (c), above except that Roseanne pays $5000 annual premiums directly to the insurance Company?

This is still alimony. The only difference from c above is that it is made to a 3rd partye) Same as (d) above, except that instead of transferring the policy to Tom, Roseanne retains ownership of the policy

but irrevocably names Tom as the beneficiary. not alimony because she retains an interest in the property.

Page 41: Beck Fed IncomeTax Outline

(2) Receiving spouse takes carry over basis – no transfer of gain or loss

III. Other Tax Aspects of DivorceA. Child Support § 71(b)(1)(D); (c) Reg 1.71-1T(c)

1. Child support payments are not deductible by the payor.a. The policy rationale is that there is a moral and legal obligation to pay

child support, so Congress does not need to encourage it. 2. Types of Child Support Payments

a. Payments specifically designated as child support in a written divorce or separation instrument. § 71(c)(1)(1) In the case of these payments it is easy to determine what qualifies for

income splitting under § 71(c) & what does not.b. Payments not specifically designated as child support but reduced upon

happening of some contingency relating to the child (marriage, reaching majority, et al.) and designated in the written instrument. § 71(c)(2)(A) anti-Lester rule

c. Payments not designated as child support but are reduced on the happening of some contingency clearly related to the child, but not designated in the written instrument. § 71(c)(2)(B) also part of anti-Lester ruleThe bright line test for clearly related to the child:Reg 1.71-1T(b)(q18) gives 2 situatons: (1) where payments are to be reduced within 6 months of the child

reaching majority (18 or 21), or (2) (where there are 2 or more children) where payments are to be reduced

more than twice, the ages of the children at the time of their reductions must be at least 2 years different when the reductions occureg: reduction for child 1 is when he is 17yr, 4 mths, 10days; then

Problems pg 2121. Michael and Lisa Marie’s divorce decree becomes final on January 1st, 2000. Discuss the tax consequences of the

following transactions to both Michael and Lisa Marie:a) Pursuant to decree, Michael transfers to Lisa Marie in March 2000 a parcel of unimproved land he purchases 10

years ago. The land has a basis of $100,000 and a fair market value of $500,000. Lisa Marie sells in 4/2000 for $600,000.

transfer to Lisa Marie is not taxed. Lisa Marie takes $100,000 basis; $500,000 taxable gain when she sells. Will likely be treated as capital gains unless she is a merchant in land. (Holding period is tacked between spouses meaning that it was long tem gain as the property was held by the couple for more than 1 year)

b) Same as (a) above, except land is transferred to satisfy a debt that Michael owes Lisa Marie. The land has a basis of $500,000 and a fair market value of $400,000 at time of transfer. Lisa Marie sells the land for $350,000.

Anti-loss provision of § 1015(a) does not apply; Lisa Marie takes Michael’s basis and has a loss of $150,000

c) What results if pursuant to the divorce decree, Michael transfers the land in (a) above , to Lisa Marie in March 2005.

Considered property settlement pursuant to divorce decree because it was done within 6 years of divorced) Same as ( c) above, except that the transfer is required by a written instrument incident to a divorce decree.

Same e) Same as (c), above except the transfer is made in March,2007.

This is over 6 years but it is a rebuttable presumption. In this case, since it is pursuant to a divorce decree it doesn’t matter how long it takes. It is OK.

Page 42: Beck Fed IncomeTax Outline

reduction for child 2 must be 19 yr, 4 mths, 11 days (at least 2 years away, either up or down, from that age) – so you can Lesterize child support (from the Lester case) but the more children you have the longer you will be paying child supportbut see pg 214, this is easy to beat by adjusting the drop dates so that there is 2 years between the ages of the children on the drop date

3. § 71(c)(3): If payment is less than amount specified, it is first considered child support and then alimony.

Chap 11: Other Exclusions From Gross IncomeI. Gains From the Sale of Principle Home § 121

Reg 1.121-1(a)-(e), -1(f) ex. 1,6,10 & 11, -2(a), -3(a), (b) ex 1

A. Background: There has always been a favorable treatment towards this type of gain for several reasons:1. It seems inappropriate to tax people upon the sale of their home because they

usually do so to change jobs, accommodate a growing family or downsize after reaching retirement, all circumstances that are often out of the taxpayer's control.

2. A home is the largest asset in many taxpayers' estate, and taxing a sale would create a large increase in tax liability.

B. The Rule: gross income does not include the gain from the sale or exchange of a principle residence. § 121.

a. To be eligible for exclusion, taxpayer must have owned and used property as a principle residence for at least two total years in the five year period preceding the sale. § 121(a)(1) Whether a residence is "principle" is determined by the intent of the

Problems pg 2141) Sean and Madonna enter into a written support agreement which is incorporated into their divorce decree at the time of

their divorce. They have one child who is in Madonna’s custody. Discuss the tax consequences in the following alternative situations:a) The agreement requires Sean to pay Madonna $10,000 per year and provides that $4000 of the $10,000 is for the

support of their child. $6,000 taxable income deductible by him and includable by her § 71(b)(1)(a) $4,000 child support not taxed § 71(c)(1)

b) The agreement requires Sean to pay Madonna $10,000 per year, but when their child reaches age twenty-one, the amount is to be reduced to $6,000 per year.

Same as above; $4000 is clearly child support c) The agreement requires Sean to pay Madonna $10,000 per year but that the payments will be reduced to $8,000 per

year on January 1, 2008, and to $6,000 per year on January 1, 2012. Sean and Madonna have two children: Daughter (born June 17, 1990), and Son (born March 5, 1993)

On 1/1/08, Daughter will be 17 yrs, 6 months & 13 days old. Reduction date is within 6 months of 18th birthday, so flunk the first test for this reduction. On 1/1/12, Son will be 18 yrs, 8 months, 25 days old. There is not more than 2 years between their ages on the drop dates, so $2000 per year per child is considered child support and $6000 is alimony.

d) What result in (a) above, if Sean pays Madonna only $5,000 of the $10,000 obligation in the current year?$4000 child support and $1,000 is alimony (Child support payment is counted before alimony is counted)

Page 43: Beck Fed IncomeTax Outline

taxpayer.b. Limitations

(1) The amount of gain excluded from gross income on any sale shall not exceed $250k. § 121(b)(1).(a) In the case of joint returns the exclusion limit is $500k if:

i. There is a joint return for the year of the sale.ii. Either spouse meets ownership requirementiii. Both spouses meet the use requirements, andiv. Neither spouse is ineligible for the benefits because of a

previous exclusion within the last two years.(2) Exclusions are limited to one every two years.

c. The ownership and use requirements, as well as 2 year limitation do not apply if:(1) The sale or exchange occurred due to a change in place of

employment, health, or unforeseen circumstances.(a) In which case the portion of the $250,000 ($500,000 for joint

returns) that may be excluded is in the same ratio as the shorter of the actual ownership and use during the prior five years or the time between the prior and current sale to two years. § 121(c)

2. Depreciation: § 121(d)(6) – gain is taxed in the amount of depreciation taken

Problems pg 2261) Determine gain that married couple filing jointly must include in income:

a) sold principal residence for $600,000. Purchased for $200,000 several years ago and have lived there for that time$500,000 exclusion applies; all $400,000 of gain is excluded from income

b) purchased another principal residence for $600,000 and sold it 2 ½ years later for $1 million$400,000 is excluded from income

c) what if second sale occurred 1 ½ years laterif the reason for the quick sale is one mentioned in § 121(c)(2)(B), then they could exclude 75%otherwise, not excludable

d) what if homeowners were granted non-recognition under former Roll-over provisions and had basis in second residence of $200,000?

$800,000 gain – no special rule for this; $500,000 is excludable from incomee) what if house was summer residence used for 3 months a year?

Doesn’t qualify; no exclusionf) What if homeowner who met the ownership and use requirement is a single taxpayer. Sold house for $400,000 with adjusted basis of $190,000 and taking depreciation of $10,000 for home office use.

$210,000 gain but $10,000 is from depreciation so taxable § 121(d)(6); balance of $200,000 is excluded

2) Single Taxpayer buys residence for $500,000 and one year sold for $600,000 due to transfer by employer.a) What gain included in income?

Gets ½ of $250,000 ($125,000) but since his gain is only $100,000 he can exclude all of thatb) what if he sold for $700,000?

$200,000 gain; $125,000 excluded, $75 included in income3) Taxpayer has owned and lived in residence for 10 years, the last year with Spouse. Sell for $500,000; basis $100,000

a) if file joint return, what income? Only get $250,000 exclusionb) what if they had lived together for 2 years in the house? Then, get full exclusion (here $400,000) c) what if, after one year of marriage, Taxpayer deeds one-half of residence to Spouse pursuant to divorce decree. Spouse continued to live in house and one year later, sold the house for $500,000?

Get full exclusion § 121(d)(3)(A)&(B)d) what if, after one year of marriage, Taxpayer deeds one-half of residence to Spouse pursuant to divorce decree. Taxpayer continued to live in house and one year later, sold the house for $500,000?Same as above

Page 44: Beck Fed IncomeTax Outline

Identification of the Proper TaxpayerChap 12: Assignment Of IncomeI. The Progressive Tax System & Income Splitting §§ 1(a) – (e), (h); 6013(a) see §§ 1(g); 63; 66; 73

The progressive tax system taxes higher income individuals at higher rates. § 1. For this reason individual taxpayers often attempt to transfer some income to

another individual in a lower income bracket, thus lowering the total tax liability. Who is the taxpayer? Income shifting to someone in a lower tax bracket – 1948

allowed spouse to file jointly and split income 1986 § 1(g): Kiddie Tax prevents shifting to children under 14. Children under

14 are taxed on all unearned income in excess of $1000 at the tax rate of their highest rate parent (or custodial parent)

II. Income From ServicesA. Assignment of Income

The person who earns and has the right to receive income cannot transfer tax consequences by assigning a portion of the income prior to its receipt to another individual by anticipatory assignments or contract. Lucas v. Earl.1. the fruit has to be taxed to the tree on which it grew 2. if you earn it you are taxed on it – if you control it, you are taxed on it3. this was not an avoidance of tax strategy by the taxpayer (contract formed in

1901 before income tax) – this partnership for a family is treated differently from a business partnership – Beck thinks this is decided wrongly

B. Renunciation of incomeA taxpayer does have the right to make an anticipatory renunciation of income and avoid any tax liability, if he neither receives the income nor directs its disposition. Giannini v. Commissioner.1. he controlled $$ because of his position on the board – Beck thinks this was

decided wronglyC. Waiver of Executor Fee

The executor of an estate may waive his right to receive statutory commissions without incurring an income tax liability. Rev. Rul. 56-472.1. If the waiver is not executed prior to the performance of services, there must

be some evidence of an intent to render the services gratuitously. Rev. Rul. 66-167.

D. FacultyAmounts received for services performed by a faculty member or a student of the university's school of law under the clinical programs and turned over to the university are not includable in the recipient's income. Rev. Rul. 74-581.1. unique factual situations – like doctors in hospital or police officers – don’t

have control: the right to receive and enjoy and they were acting as an agent of their organization while earning the money

Page 45: Beck Fed IncomeTax Outline

III. Income From PropertyA. General Rule

The general rule is that the owner of property is taxed on the income from the property. An assignment is ineffective for tax purposes if only the income from the property is assigned, while the assignor keeps the property itself. Helvering v. Horst.1. the right to receive the income creates the tax liability 2. got the enjoyment of the income by giving it to his son3. giving away the property would prevent liability for the tax on income

B. Exceptions1. But income can be successfully assigned for tax purposes if the assignor does

not own the property which produced the income. Blair v. Commissioner.a. life estate (taxpayer has no interest in corpus) – income completely

assigned to children – he only had the right to receive the income, and that is what he gave away (the income interest was his tree and not the fruit)

2. The owner of income producing property may also assign income from the property for valid consideration in an arm's length transaction. Estate of Stranahan v. Commissioner. (because son bore the risk that the loan would not be re-paid)a. In other words, income from property may not be assigned gratuitously for

tax purposes, but it may be sold in a valid transaction.3. If the owner of income producing property transfers both the property and the

income therefrom, he has shifted the tax consequences to the recipient.a. Courts & IRS will look to the substance of a transaction to determine

whether property is being transferred, or if merely income therefrom is being assigned. Susie Salvatore.1) Transferring the property after the contract of sale was signed was too

late to avoid the tax on the proceeds of the sale. C. Income Earned But Not Realized

1. If income from property has been properly assigned by sale, transfer of the underlying property or otherwise, and income has been earned or matured, but

Problems pg 2541) Executive has a salaried position with Hi Rolling Corporation under which she earns $80,000 each calendar year.

a) Who is taxed if the Exec, at the beginning of the year, directs that $20,000 of her salary be paid to her aged parents? She is taxed. She can’t shift income even if this is done before the money is earned. She can waive the income but not shift it.

b) Who is taxed if Executive at the beginning of the year directs that $20,000of her salary be paid to any charity the Board of Directors of Hi rolling selects? (Executive is not a member of the board).

The Executive is showing some control over the income. If the Corp. decided not to give the income to charity but keep it the Executive would have some rights against the Corp.

c) Same as (b), above, except that Executive makes the same request to a $10,000 year-end bonus which Corporation has announced toward the end of the year, based on services rendered during the year?

Still taxed because she still controls itd) Who is taxed if Executive , in her corporate role, gives a series of lectures on corporate finances at a local business

school and, pursuant to her contract with Hi Rolling, turns her $1,000 honorarium over to the Corp.? The executive isn’t taxed under Rev. Rule 78-581 (she was acting as an agent for the Corp.)

Page 46: Beck Fed IncomeTax Outline

not received or realized on the date of assignment, the income is taxable to the assignor. Rev. Rul. 69-102.a. For example, if the owner of corporate stock sells the shares after a

dividend has been announced but not paid, the seller of the stock is taxed on the dividend.

b. The donor is taxed on that portion of income which had accrued up to the date of the gift. He is taxed at the time when the donees receive the income and the donees are taxed on the income from the date of the gift.

Deductions in Computing Taxable IncomeChap 14: Business DeductionsI. Introduction §§ 1; 63

A. DeductionsDeductions are said to be a matter of "legislative grace."Therefore, a taxpayer must find a Code provision that specifically authorizes the

deduction sought.B. Definition

Exclusion is a receipt which is not taxable (like a gift)Deduction is an expense which reduces taxable income The big picture:§ 61 gross income§ 62 deductions (exclusions which reduce gross income) – most favored

deductionadjusted gross income (it is good to get this low)

§ 63 standard deduction or itemizing deductions – second class deductionstaxable income

Deductions are amounts subtracted from gross income, with the difference giving the taxpayer his "taxable income." § 63.

C. Trade or Business RequirementThese are aimed only at taxpayers, individuals or corporations, engaged in a trade or business.1. The Code allows deductions for all ordinary and necessary expenses paid or

incurred in carrying on a trade or business. § 162(a).2. 2 main difficulties:

a. business v personal – lots of grey area here, like uniforms, meals & entertainment, commuting, child care etc.

b. capital v ordinary – question is one of timing: best is to recover cost immediately by deducting it from gross income and not capitalizing it to be part of basis to be recovered later – rule of thumb: if the item has a useful life longer than a year, it must be capitalized

II. The Anatomy of the Business Deduction A. “Ordinary and Necessary” § 162(a) Reg 1.162-1(a)

1. While a necessary expense is one that is appropriate and helpful in the business judgment of the taxpayer, ordinary expenses are those that are common and accepted in the taxpayer's particular trade or business. Welch v.

Page 47: Beck Fed IncomeTax Outline

Helvering.a. the payments were actually re-purchase of goodwill (a capital outlay) b. ordinary is defined as anything that is not a capital outlay (therefore

currently deductible)(1) if capitalized, it gets basis, and when he sells the business, he will

recapture his basis(2) land and goodwill cannot be depreciated – but now, purchased

goodwill can be amortized and depreciated

B. “Expenses” §§ 162(a); 263(a) Reg 1.162-4; 1.263(a)-21. Expenditures that produce benefits lasting beyond the taxable year are

generally capital expenses and are non-deductible, even if the expense does not create a separate asset. INDOPCO, Inc. v. Commissioner.a. creation or enhancement of an asset is capitalizedb. expenses for re-structuring the corporation are capitalized because they are

for the duration of the corporation’s existence, for a time longer than the current tax year

c. “deductions are the exception to the norm of capitalization” – scariest statement in the opinion – resulted in rulings allowing deductions for advertising, building repairs, severance pay, training costs, and pollution clean-up (§ 198)(1) but if an expenditure which is ordinarily deductible is used to create a

property with a life longer than 1 year, the expenditures must be capitalized . Commissioner v Idaho Power Co.

2. Expenses made pursuant to remodeling projects are capital expenses that cannot be deducted, even if the expenses were deductible if isolated from the project as a whole. Norwest Corporation and Subsidiaries v. Commissioner.a. repair is ordinary; improvement is capital – if an expenditure for some

property is only to maintain in the same useful condition, it is deductible as a repair – if an expenditure is intended to make the property better or capable of some additional use, it is capitalized

Problems pg 3191) Taxpayer is a businessman, local politician who is also an officer –director of a savings and loan association of which he

is the founder. When, partially due to his mismanagement, the savings and loan began to go under, he partially donated nearly ½ million dollars to help bail it out. Is the payment deductible under section 162?

This is not deductible. This is contribution to capital, like buying stock. However, this particular taxpayer won and was allowed to deduct. He showed the payment was necessary to repair his reputation.

2) Employee incurred ordinary and necessary expenses on a business trip for which she was entitled to reimbursement upon a filing a voucher. However, Employee did not file a voucher and was not reimbursed but instead, deducted her costs on her income tax return. Is Employee entitled to a section 162 deduction?

IRS said this was not a necessary expense because she did not have to bear the expense but could have been reimbursed

Page 48: Beck Fed IncomeTax Outline

b. asbestos removal as part of a rehabilitation is not deductible

C. “Carrying On” Business § 162(a); 195; 262 Reg 1.195-1(a)1. Section 162 does not allow a deduction for expenses incurred in entering a

trade or starting a business. Morton Frank.a. But if the taxpayer elects, start-up expenditures can be amortized (treated

as deferred expenses, deductible over a period of at least five years.) § 195. (1) To be eligible for a § 195 deduction, the taxpayer must actually have

started the business, and the expenses must be of the type allowable under § 162, i.e., "ordinary and necessary."

(2) Start-up expenses not amortized under § 195, must be capitalized or treated as a nondeductible expense.

(3) Upon disposition of the business, a taxpayer may deduct those start-up expenditures that were amortized, but not previously deducted. § 195(b)(2).

2. Individuals Engaged in a Trade or Business as Employeesa. Although an individual cannot deduct expenses incurred in entering a

trade or business, he may deduct expenses incurred in searching for a new job within a trade or business that he is engaged in. § 162.(1) But if a substantial period of time elapses between cessation of the

taxpayer's previous employment and his efforts to find new employment, the Treasury will not allow the deductions on the ground that the taxpayer is not "carrying on a trade or business." Rev. Rul. 75-120.

b. Section 162 deductions are not allowed for expenses incurred in looking for a first job, or entering a new trade or business.(1) exception: Hundley case – payment for employment-seeking services

is contingent upon employment and does not become due until

Problems pg 3311) Contaminator incurs asbestos removal costs on a business building. Consider the extent to which they are currently

deductible in the following alternative situations and assume, unless otherwise indicated, the building is not § 198(c) “qualified contaminated site”a) Contaminator acquired the building aware that it contained asbestos.

Depends on when the building was obtained. Now, when we know about the hazards of asbestos or many years ago, when such knowledge was lacking? If one knows that they have to get rid of asbestos before they but the building this would be reflected in the purchase price. Then it would be a capital expenditure. If one obtains the building knowing that it has asbestos but not knowing that it would be a problem, then it would be considered repair and would be capitalized.

b) Same as (a), above except the building is at a Section 198(c) “qualified contaminated site.” Currently deductiblec) Several years ago, Contaminator constructed the building containing the asbestos.

The Contaminator wouldn’t put asbestos in knowing that he would later have to take it out. Most likely this would be repair to put back in condition it was before and would be capitalized. Or a repair to stisfy regulations. Currently deductible

d) Contaminator acquired the building unaware that it contained asbestos.This would be given repair treatment; to bring to code is a repair (no change to use or capacity)

e) Same as ( d), above, except that the removal occurs in conjunction with a remodeling of the building. Like Norwest, not deductible

Page 49: Beck Fed IncomeTax Outline

employment is secured(2) politicians and judges cannot deduct expenses incurred in re-election

process

III. Specific Business DeductionsA. Reasonable Salaries § 162(a)(1) see §§ 162(m); 280G Reg 1.162-7, -8, -9

1. deduction for reasonable salaries and compensation for services rendered. § 162(a)(1). a. avoidance of double tax to corporation is accomplished by paying out

profits in the form of a deductible expense (like interest on a loan from the shareholder or pay out the profits as salaries) – this only works for closely held corporation (1) Tax Court applies 7 factors to determine reasonableness of the salary

paid to CEO Exacto Spring Corporation v Commissionera) type and extent of services renderedb) scarcity of qualified employeesc) qualifications and prior earning capacity of the employeed) contributions of the employee to the business venture

Problems pg 3391) Determine the deductibility under §§ 162 and 195 of expenses incurred in the following situations

a) Tycoon, a doctor, unexpectedly inherited a sizable amount of money from an eccentric millionaire. Tycoon decided to invest a part of her fortune in the development of her industrial properties and she incurred expenses in making a preliminary investigation.

This is not deductible under section 162 of the IRC b/c a taxpayer must be “carrying on a business”. The doctor is not “carrying on” a business b/c there is no business to “carry on”. The investment is too different than the doctor’s actual practice of medicine. The cost however are deductible under section 195. Section 195 can be considered “pre-opening expenses”.

b) The facts are the same as in (a) above except that Tycoon, rather than having been a doctor, was a successful developer of residential and shopping center properties.

Yes, this could be deductible under section 162. Taxpayer has a better argument b/c he is “carrying on ‘ a trade or business

c) The facts are the same as in (b), above except that Tycoon, desiring to diversify her investments, incurs expenses in investigating the possibility of purchasing a professional sports team.

This is too different and the taxpayer is stuck with section 195d) The facts are the same as in (a) , above except that Tycoon then begins developing industrial properties. Tycoon is

then advised by her lawyer that her expenses qualify as section 195 “start up ‘ expenses. Since Tycoon has commenced developing properties may she forego a section 195 election and deduct her prior expenses under section 162?

No option of taking the deduction; cannot “go back” and take § 162 deductions e) The facts are the same as in (d) above. However, after two years Tycoon’s fortunes turn sour and she sells the

business at a loss. What happens to the deferred investigation expenses? Section 195(b)(2) allows taxpayer to deduct remaining basis immediately

2) Law student’s Spouse completed secretarial school just prior to student entering law school. Consider whether Spouse’s employment agency fees are deductible in the following circumstancesa) Agency is unsuccessful in finding Spouse a job.

Isn’t deductible. 1st job isn’t in trade or business.b) Agency is successful in finding spouse a job.

Not deductible; first job. § 195 does not apply to job searchc) Same as (b) above, except that Agency’s fee was contingent upon its securing employment from Spouse and the

payments will not become due until the Spouse has begun working. Deductible under Hundley decision

d) Same as 9(a) and (b), above, except that Spouse previously worked as a secretary in Old Town and seeks employment in New Town where student attends law school.

It is deductible b/c looking for a job in the same profession.e) Same as (d), above except that Agency is successful in finding Spouse a job in New Town as a bank teller.

The problem here is with the line of work. Could argue that it is still a clerical type of job

Page 50: Beck Fed IncomeTax Outline

e) net earnings of the employerf) prevailing compensation paid to employees with comparable jobsg) peculiar characteristics of the employer’s business

(2) opinion by Posner of law & economics: disfavored the 7 factor test and used the independent investor test – if investor was getting a good return (more than 13 percent) as a result of the CEO’s efforts, and the salary is approved by these independent investors, then the salary is an arm’s length deal and not unreasonable

(3) Beck feels the 7 factor test is still valid where there is total self-dealing (no independent investors to test)

b. The reasonableness of an employment contract is usually challenged only in the context of closely-held corporations or family owned businesses. Harolds Club v. Commissioner.(1) percentage agreements are not unreasonable even if excessive, if they

are fair when made (part of a free bargain between parties of equal bargaining power)

(2) Beck questions whether this bargain was fair “when paid”B. Travel “Away from Home” §§ 162(a)(2); 274(n)(1) see § 274(c), (h) & (m)(1) &

(2) Reg 1.162-21. The Code specifically provides a deduction for travel expenses (50 % of meals

& all lodging) incurred while away from home in pursuit of a business or trade. §162(a)(2).a. These deductions are allowed in order to offset duplicated living expenses.

(but this is not in the statute, and is not dispositive) (1) A itinerant taxpayer with no permanent home may not deduct travel

expenses. Rosenspan v. U.S.(a) IRS had considered home to mean tax home (principal place of

employment) but this was used to prevent deduction for commuting expenses

(b) used the Flowers test to determine if an expense is deductible:1. expense must be reasonable and necessary2. incurred while away from home (plain & ordinary meaning)3. in pursuit of business

b. An individual can have only one "home" for tax purposes. Andrews v. Commissioner.(1) time spent and money earned determine which is the primary home

c. A taxpayer is not treated as being "away from home" if the period of employment lasts over one year. § 162(a).

d. nature of trip is determined by number of business days e. domestic travel: airfare is fully deductible; meals and lodging on business

days only are 50% deductible § 274nf. international travel of more than a week and 25% or more is non-business,

airfare, meals and lodging are prorated a. convention or meeting held outside North America is deductible if it is

as reasonable to have it outside NA as it would be to have it in NA

Page 51: Beck Fed IncomeTax Outline

Problems pg 374 1) Commuter owns a home in Suburb of City and drives to work in City. He eats lunch in various restaurants in City.

a) May Commuter deduct his costs of transportation and/or meals ?Reg 1.162- 2( e) No. Commuter’s fares are not considered as business expenses and are not deductible

b) Same as (a), above but Commuter is an attorney and must often travel between office and City Court House to file papers, try cases, etc. May Commuter deduct all or any of his costs of transportation and meals?

Can deduct travel but not meals. Meals you have to be away overnight and then only deduct 50%c) Commuter resides and works in City, but occasionally must fly to Other City on Business for his employer. He eats

lunch in other City and returns home in the late afternoon or early evening. May he deduct all or a part of his costs?He can only deduct travel because he didn’t stay overnight. If he takes deduction, it is an unreimbursed employee business expense and can only deduct if 2% of AGI § 67

2) Taxpayer lives with her husband and children in City and works there.a) If her employer send her to Metro on business for two days and one night each week and if Taxpayer is not

reimbursed for her expenses, what might she deduct? See section 274 (n) (1). Taxpayer is allowed to deduct all her costs, 50% of the cost of meals.

b) Same as (a), above, except that she works three days and spends two nights each week in Metro and maintains an apartment there. She can deduct the rent for the apartment, because she maintains it solely for business

c) Taxpayer and Husband own home in City and Husband works there. Taxpayer works in Metro, maintaining an apartment there, and travels to City each weekend to visit her husband and family. What may she deduct?

Reason for seeing husband is personal. There is no business reason for seeing him. Falls within the crack of the rule. Not deductible.

3) Burly is a professional football player for the City Stompers. He and his wife own a home in Metro where they reside during the 7-month “off season”.a) If Burly’s only source of income is his salary from the Stompers, may Burly deduct any of his City living expenses

which he incurs during the football season? The IRS theory is that City is Burly’s tax home b/c this is where he works. As far as his Metro home he can’t deduct either b/c that home is unrelated to work

b) Would there be a difference in result in (a), above, if during the 7-month ‘off season” Burly worked as an insurance salesman in Metro?

Andrews case and he should be able to deduct in one of the two places in which he works.4) Temporary works for Employer in City where Temporary and his family live.

a) Employer has trouble in Branch City office in another state. She asks Temporary to supervise the Branch City Office for nine months. Temporary’s family stays in City and he rents an apartment in Branch City. Are temporary’s expenses in Branch City deductible?

This is deductible b/c it is less than 1 year (Remember the one year rule) and duplication of expensesb) What result in (a), above, if the time period is expected to be nine months, but after eight months it is extended to

fifteen months? See Rev Rule 93-86 Should be able to deduct for 1 year but for the remainder he can’t c) What results in (a), above, if Temporary and his family had lived in a furnished apartment in City and he and his

family gave the apartment up and moved to Branch City where they lived in a furnished apartment for nine months?Yes this is deductible b/c it is away from home

5) Traveler flies from her personal and tax home in New York to a business meeting in Florida on Monday . The meeting ends late Wednesday and she flies home on Friday afternoon after two days in the sunshine.a) transportation, meals, and lodging deductible? Reg 1.162-2(a) and (b)

Domestic travel for business purpose. 3 business days and 2 personal days. Transp is deductible. 50% of meals on MTW and all of MTW lodging.

b) May Traveler deduct any of her spouse’s expenses if he joins her in the trip? § 274 (m)(3) No c) What results in (a), above, if Traveler stays in Florida until Sunday afternoon?

Ignore weekends, primary purpose still business.d) What results in (a) above, if traveler takes a cruise ship leaving Florida on Wednesday night and arriving in New

York on Friday? See § 274(m)(1) deduction is limitede) What result in (a), above, if Traveler’s trip is to Mexico City rather than Florida? § 274(c) Prorate transp expensef) What result in (e) above, if Traveler went to Mexico City on Thursday and conducted business on Thursday, Friday,

Monday, and Tuesday, and returned to New York on the succeeding Friday night? See Reg. 1.274-4 (d) (2) (v)- If trip is > a week and 25% or more of time is personal then prorate transp according to which days are business and personal. Here, 6 bus days out of 9 which means 33% of time is non-business 2/3 of trips is buss therefore 2/3 of transp deductible

g) What result in (e), above, if Traveler’s trip to Mexico City is to attend a business convention? See § 274(h) no deduction. if you go to convention held outside NA area you get no ded unless it is as reasonable for it to be held outside NA as inside NA. If you have conference of dentists in Paris and all NY Dentists, then not reasonable. NA areas defined in § 274(h)(3)(A) Mexico in NA, so limitation doesn’t apply

Page 52: Beck Fed IncomeTax Outline

C. Necessary Rental and Similar Payments § 162(a)(3) Reg 1.162-11(a)1. The Code allows a deduction for rentals or other payments required to be

made as a condition to the continued use or possession of business property to which the taxpayer has not taken title or in which he has no equity. § 162(a)(3).a. A taxpayer may not seek business deductions for necessary rental

payments if the rental or lease agreement is, in substance, a sale. Starr's Estate v. Commissioner.(1) very unlikely that the lessor will reclaim the property at the end of the

lease (2) the total lease payments total the purchase price buyer/lessee - advantage to the taxpayer was that a lease is deductible;

in a sale, only the depreciation and interest are deductible sellor/lessor: in a lease, all payments are income, gets depreciation; in

a sale, gets basis and interest income – the tax benefits of the depreciation can be substantial; consider airlines which do not own planes because they don’t need the depreciation – the banks get the depreciation

b. Intra-family gift and leaseback arrangements that create the need for a business to make rental payments will be scrutinized to determine whether the transaction has no business purpose or whether the taxpayer retained substantial control over the property, in which case there will be no deduction allowed. White v. Fitzpatrick.(1) Beck thinks this is best left to Congress to decide – husband was trying

to income split but nothing in the transaction was illegal – although the transactions were not valued in an arm’s length manner at fair market value

(2) transfers and leasebacks are done by doctors for medical buildingsD. Expenses for Education §§162(a); 262; 274(m)(2) Reg 1.162-

5(a), (b)(1), (2)(i), 3(i), (c), (d), (e)(i)1. Education expenses incurred to fulfill the requirements of a taxpayer's

employment or profession are deductible as necessary and ordinary business expenses. Hill v. Commissioner; Treas. Regs. § 1.162-5(a)(2).a. deductible if education is intended to improve or enhance skills in present

business or trade (continuing education or further specialization)b. not deductible if education is:

(1) intended to qualify you for a new trade or profession (law or medicine)(2) the minimum required for the job you already have(3) travel for education

Page 53: Beck Fed IncomeTax Outline

2. A taxpayer may deduct education expenses incurred in maintaining or improving his professional skills. Coughlin v. Commissioner; Treas. Regs. § 1.162-5(a)(1).

IV. Miscellaneous Business DeductionsA. Introduction §§ 162(a); 274(a), (d), (e), (k), (l), (n) Reg 1.162-20(a)(2); 1.274-

2(a)(1), (c), (d)1. Business Meals and Entertainment

a. Although business meals and entertainment expenses may be deducted as ordinary and necessary expenses, the Code imposes some limitations and requires substantiation with respect to such expenses.(1) The expenses must be either "directly related to" or "associated with"

the taxpayer's trade or business. § 274(a)(1)(A).(a) "Directly related to" means that business must go on during the

entertainment or meal with a client. (b) "Associated with" requires the business be conducted immediately

preceding or following the entertainment.(2) Only 50% of the meal and entertainment expenses can be deducted. §

274(n)(1).

Problems pg 3931) Alice, Barbara, Cathy and Denise, doctor, dentist, accountant (CPA) and lawyer, respectively. In the current year, after

some time in practice as an orthopedic surgeon, Alice, who was called upon to give medical testimony in malpractice suits, decided to go on to law school so as to better understand this aspect of her medical practice. Barbara enrolled in a course of post-graduate study in orthodontics, intending to restrict her dental practice to that specialty in the future. Cathy enrolled part time in law school (with eventual prospects of attaining a degree) so as to better perform her accounting duties in areas in which law and accounting tend to overlap. And Denise took a leave of absence from her firm to enroll in an LL.M course in taxation, intending to practice exclusively in the tax area. Which, if any, is incurring deductible expenses of education?

Alice: She is qualifying for a different profession; this is not deductible. If she was taking a course for doctors in malpractice law which carried no law school credit but involves going as non-matriculated student in a law school, it would be continuing education so would be deductible.Barbara: Orthodontics study is specialization within her field and would be deductible Cathy: This isn’t deductible because she already has fulfilled the requirements in her field. Denise: This is deductible as specialization in field

2) Assume Denise’s expenses in problem 1, above are deductible. If she is a practitioner is Seattle, Washington, who travels to Gainesville, Florida for a year to participate in their LL.M program, what expenses, in addition to tuition and books, may she deduct?

Does it make a difference that there is program closer to home? No, it shouldn’t. 3) Carl earned a bachelor’s degree in education and he teaches world history in a junior high school. In the current year he

contemplates a summer European tour doing things that will be helpful to his teaching efforts. He wished to know if he may deduct his expenses. What do you advise?

see § 274(m)(2): “No deduction shall be allowed under this chapter for travel as a form of education.” Before could write whole thing off as education. Now travel is not deductible for education.

4) Dentist attends a five day dental seminar at a ski resort. All of the seminar proceedings are taped and Dentist skies on clear days and watches all of the tapes on snowy days or in another off-the slopes times prior to his return home. Are Dentists travel, meals and lodging deductible?

Very suspicious.

Page 54: Beck Fed IncomeTax Outline

(a) The 50% limitation applies to any deductible meal.i. meals eaten on a business trip are subject to the 50% limitation.

(b) The meals cannot be lavish or extravagant. § 274(k)(1)(A).(c) 50% limit applies to face value of any ticket to entertainment

event. § 274(l)(1)(A).(3) The taxpayer claiming the deduction must be present. § 274(k)(1)(B).(4) Expense must be substantiated by receipt.

b. The cost of entertainment facilities is expressly non-deductible.(1) Entertainment facilities include sky boxes, and social and athletic

clubs among others. § 274(a)(1)(B).(2) But costs related to the use of such facilities are deductible if they

meet the requirements of § 162 and the limitations of § 274.(a) For example, 50% of the cost of a reasonable business meal eaten

at the taxpayer's country club may be deducted, even though the country club dues are non-deductible.

2. Uniforms: The cost of acquiring and maintaining a uniform required for taxpayer's employment is deductible if the uniform is one that is not suited for general use.

3. Advertising: Unless they are used to acquire a capital asset, advertising expenses are deductible in the year they are incurred or paid. Treas. Reg. § 1.262-1(b)(8).

4. Dues: Dues paid to organizations that directly relate to a business are deductible. Regs. § 1.162-20(c)(3).

5. Lobbying Expenses: As a general matter, expenses incurred in petitioning government or any similar activity are nondeductible. § 162(e)(1).

6. Political contributions are non-deductible

Page 55: Beck Fed IncomeTax Outline

B. Business Losses §§ 165(c)(1); 280B1. A deduction is allowed up to the basis for losses relating to property incurred

in a trade or business. I.R.C § 165. (minus any insurance recovery) a. To be deductible, the loss must be connected with some realizing event,

i.e., a sale. Treas. Regs. § 1.165-1(b).b. Loss due to an accident or destruction, is limited to the difference between

the value before and the value after the accident, up to basis (and basis is reduced by that difference)

2. A taxpayer may not deduct losses sustained on account of the demolition of a structure which the taxpayer owns. § 280B.a. basis of the destroyed building (and the non-deductible demolition costs

capitalized) is added to the landV. Depreciation §§ 167; 168

A. Introduction1. The Code treats depreciation as an operating expense, allowing deductions for

the exhaustion, wear and tear, and obsolescence of property used in a business or trade. §§ 167, 168.a. Prerequisites for Deduction

(1) The property must be used in a business or trade, or be held for the production of income, i.e., rental property.

(2) deduction can be claimed with respect to property that will be

Problems pg 3961) Employee spends $100 taking 3 business clients to lunch at a local restaurant to discuss a particular business matter.

The $100 cost includes $5 in tax and $15 for a tip. They each have two martinis before lunch.a) To what extent are Employee’s expenses deductible?

deductible to 50%. employee is deducting this; that means he is not getting reimbursed by employerb) To what extent are the meals deductible if the lunch is merely to touch base with the clients?

Not deductible at allc) What results if Employee merely sends the three clients to lunch w/o going herself but picks up their $75 tab?

Not deductible-The employee would have to be thered) What results in (a), above, if in addition, Employee incurs a $15 cab fare to transport the clients to lunch?

The real question is that if this is so close to the lunch that Employee gets 50% of the cab fare. Teacher says it is more likely to be 100% deductible.

e) What results in (a), above, if the Employer reimburses Employee for the $100 tab?If employee deducts and gets reimbursed, he must report the reimbursement as income.

2) Businessperson who is in New York on business meets with two clients and afterward takes them to the Broadway production of the Producers. To what extent is the $300 cost of their tickets deductible if the marketed price on the tickets is $50 each, but Businessperson buys them from hotel concierge fro $100 each?

50% of face value of ticket is deductible3) Airline pilot incurs the following expenses in the current year.

a) $250 for the cost of a new uniform. Deductible subject to § 67 2% floorb) $30 for dry cleaning the uniform. deductible c) $100 in newspaper ads to acquire a new job as a property manager in his spare time.

Not deductible b/c this is a job search for new line of businessd) $200 in union dues This is deductible. Bar dues are also deductiblee) $50 in political contributions to his local legislator who he hopes will push legislation beneficial to airline pilots.

Not deductiblef) $500 in fees to a local gym to keep in physical shape for flying-

Not deductible. But could be argued. Total § 162 deductions: $480

Page 56: Beck Fed IncomeTax Outline

consumed or wear out.(a) Unimproved real property (land) is non-depreciable. Treas. Regs.

§ 1.167(a)(2).b. Useful Life Concept

(1) Under the prior depreciation system, a taxpayer was required to identify the useful life of depreciable property, then take deductions over the length of that useful life. (a) The Accelerated Cost Recovery System (ACRS) in place today

virtually eliminates this concept and simplifies the process. § 168(b)(4).

(b) Useful life is generally 5 yrs and recapture is quicker than before. Planes and heavy equipment is 9 yrs and 27 ½ yrs for residential buildings and 39 yrs for commercial property (buildings). (depreciation = basis x amount given in IRS)

(c) Half-year convention: built-in deduction for first year no matter when property is purchased. (1) For depreciation purposes, property is treated as having been

placed in service during the midpoint of the year. So the taxpayer is allowed only a half-year deduction in year one.

(2) Taxpayers are prevented from taking advantage of the rule by purchasing equipment near the end of the year. § 168(d)(3)(A).

c. Depreciation Methods(1) The "straight-line" method allows equal deductions to be taken over

the useful life of the property. (a) For example, if the taxpayer has a basis of $1000 in machinery

with a useful life of 10 years, after which the value of the machinery is zero, the taxpayer is allowed an annual deduction of 10% of the basis.

(2) Under the "declining balance" method a uniform rate of deduction is applied to the unrecovered basis in the property.

2. The Relationship of Depreciation to Basisa. A taxpayer's basis in depreciable property is annually reduced in an

amount equal to the allowable deduction, whether the taxpayer chooses to claim the deduction or not. § 1016(a)(2).

3. Depreciation deductions may be claimed on property that is subject to business and personal use, but only to the extent the property is used for business purposes; all other tax attributes must be allocated between personal and business use. Sharp v. United States.a. think of home office: if you depreciate the home office and sell your home

at a loss you might actually have a gain on the business part (if it has been depreciated to zero)

B. Special Depreciation Rules on Personal Property §§ 168(k); 179; 280F(a) & (b); 197

1. § 168(k) Additional Depreciation: extra 30% depreciation on most types of new personal property acquired between 9/10/2001 and 9/11/2004 – basis is

Page 57: Beck Fed IncomeTax Outline

adjusted down before application of remaining depreciation calculations2. § 179 Bonus Depreciation: Taxpayers are allowed an additional deduction on

some types of property that qualify for ACRS treatment in the year the property is acquired.a. dollar limits: this year 24,000 and next year 25,000 with dollar for dollar

reduction for any amount over $200,000 (1) eg: 224,000 of property put into service this year = no bonus

deduction; $210,000 of property put into service this year = 14,000 bonus deduction

b. cannot exceed taxable income from the business during the yearc. both 168k & 179 can be applied and then the regular depreciation under §

168 eg: $100,000 property put into service – take 30% off, leaving 70,000 subject to 168 additional depreciation, to get 54,000 in depreciation and have basis of 46,000 remaining – and then take 5-year table depreciation – more than half the cost of the equipment has been deducted in the first year (economic stimulation)

d. if equipment costs less than 24,000, the entire amount can be deducted as depreciation (and 0 basis remaining)

4. § 280F(a): Limitations on Luxury Cars (costs more than $12,800; applies to leased cars too)a. year 1 – 2,560 + an additional 4600 under 168k

year 2 – 4,100year 3 – 2,450year 3 – 1,475 (and for subsequent years until recovery complete)

5. § 280F(b): Limitations on listed property (property used for entertainment or recreation, telecommunications equipment, and computer not used exclusively for business) business airplanes, boats etc. used for fun and businessa. allocation for depreciation calculation is based on amount of time used for

business and amount of time used for personal use - if used for less than 50% business, can’t use favorable 5 year accelerated table to depreciate, use § 186g

b. if amount of business use decreases, depreciation for past years in excess of the slower depreciation is re-captured as income

6. § 197 Amortization of goodwill: depreciable if purchased over 15 years, straight line

7. investment tax credits - repealed in 1986 – better than depreciation or deduction – they were a dollar for dollar tax reduction for the amount of the credit (reduces tax not the income which is taxed)

C. Special Rules on Realty §§ 168; 42; 46; 471. Real property that is eligible for depreciation deductions is divided into two

types: residential rental property and non-residential rental property.a. Under the ACRS both types of properties are subject only to the straight

line depreciation method and both have lengthy recovery periods. § 168.2. Congress has provided special incentives for investments in older property

and low-income housing. §§ 42, 46, and 47.

Page 58: Beck Fed IncomeTax Outline

a. 20% tax credit for rehabilitation of a certified historic structure or 10% on any building built before 1936

Chap 15: Deductions For Profit-Making, Nonbusiness ActivitesI. Expenses for production of income §§ 212; 274(h)(7) Reg 1.212-1(g), (k), (l), (m); 1.262-1(b)(7)

Ordinary and necessary non-business expenses incurred in collecting or producing income, or in managing, conserving or maintaining income-producing property are deductible under § 212. (below the line deduction and subject to § 67 2% floor)

A. § 212 or § 162§ 212 was enacted because expenses incurred in managing personal business, like investments, are not deductible under § 162's provision for deduction of expenses incurred in carrying on a business. Congress passed § 212 due to ruling in Higgins v. Commissioner.1. Keeping the records of his investments were not sufficient to be a trade or business2. Had he participated in the management of the corporation in which he was

invested, his expenses might have been deductible. B. Limitations & Restrictions

Deductions under § 212 are subject to same limitations and restrictions as deductions under § 162.1. Litigation expenses incurred in defending or protecting income-producing

property are not deductible because they are not ordinary and necessary. Bowers v. Lumpkin.a. The ordinary and necessary requirement is not a strict, technical one,

however. The requirement is satisfied when the expenses are adequate, helpful and necessary in light of real life circumstances.

b. acquisition costs and costs to perfect title are part of the capital expense2. A showing of a proximate relationship between the non-business expense and

the income is not necessary in the Fifth Circuit. Surasky v. U.S.a. applied §162 language of ordinary and necessary (appropriate and helpful)

3. Such a showing is required by the Internal Revenue Service and in other circuits, however. Revenue Ruling 64-236. allows deduction for proxy fights expenses. (compare to Indopco)

C. Personal expenses are not deductible § 2621. Thus, legal expenses incurred defending a claim that arises out of one's

personal life, even if it may have some effect on income-producing property, are not deductible. Meyer J. Fleischman.a. Courts apply a but/for origin of claim test to determine if a claim arises out

of one's personal life or out of business or income-producing activities. (1) For example, if the claim would not exist but for the personal

relationship, expenses incurred in litigating that claim are personal and not deductible.

b. Legal expenses incurred in collecting income, such as alimony, however, may be deducted as non-business expenses, because the claim arises out of

Page 59: Beck Fed IncomeTax Outline

collecting income.Beck comment: Husband gets no deduction for resisting alimony but wife can get deduction for claiming and trying to collect alimony.

II. Expenses arising out of Transactions entered into for profit §§ 121(a), (d)(6); 165(a), (b), (c)(2); 167(a)(2); 168(a); 212 see §§ 195; 280A Reg 1.165-9(b); 1.167(g)-1; 1.212-1(h)

Expenses incurred in transactions entered into for profit are also deductible.A loss resulting from sale of property for less than it was purchased is not deductible, unless the property constitutes a transaction entered into for profit and is not a personal residence.

1. A personal residence may be converted to a transaction entered into for profit by showing more than mere abandonment, like abandonment followed immediately by demolition or renting. William C. Horrmann.a. § 212(2): deduction for maintenance and depreciation if converted to

investment property – had to be held for production of income – even though never rented, he had tried to rent it

b. § 165(c)(2): loss deduction for property used in trade or business or from a transaction entered into for profit – (c)(3) loss on residential property is

Problems pg 4651) Speculator buys 100 shares of sound company stock for $3,000, paying her broker a commission of $50 on the purchase.

Fourteen months later she sells the shares for $4,000 paying a commission of $60 on the sale.a) She would like to treat $100 paid as commissions as § 212 expenses. Why? Can she? See Reg 1.263 (a) –2(e).

commissions are capital expense – added to basis and subtracted from amount realized – gain is amount realized minus adjusted basis - timing issue and deducting is worth double what reducing capital gains is worth (different tax rate) but rules require capitalizing itpurchase has basis of $3050; sale has amount realized of $3940; gain of $895

b) What results in (a), above if she sells the shares for $2500 paying a $45 commission on the sale? See § 165 (c) (2)2500 – 45 = 24552455 – 3050 = (595) loss on sale

c) Speculator owned one-tenth of one percent of the Sound Company Stock but, an eager investor during the time she owned the stock, she incurred $500 of transportation, meals and lodging expenses in traveling 1000 miles to new York City to attend Sound’s annual shareholder meeting. May she deduct her cost under section 212 (2)?

might be allowed under § 212 but cost is not reasonable in proportion to the size of her investment d) What result in (c), above, if instead Speculator owned 10% of the total outstanding Sound stock, worth $300,000?

more reasonable in relation to size of investmente) What result to Speculator is she incurred the expenses in (c), above, to attend a seminar on investments?

§ 274(h)(7): no deduction for investment seminars f) Speculator owns 10% of Sound’s stock worth $300,000 and she incurs $10,000 in legal fees and personal costs

investigating the operation of the business after the business has some serious setbacks. Is the $10,000 deductible?Under Surasky: could be conservation of property – if you have a large investment in stock and use a reasonable amount to determine what is going on with the business, it may be deductible

2) After reading Fleishman case, consider in what situations :a) Payor Spouse may deduct attorneys’ fees incurred in getting a divorce. In defending against alimonyb) Payee Spouse may deduct attorneys’ fees incurred in getting a divorce. In claiming & collecting alimonyc) Payee Spouse’s attorneys’ fees incurred in getting a divorce was deductible by Payor if Payor pays them.

As part of alimony agreement3) Planner consults his attorneys with respect to his estate plane. They decide to make various inter vivos gifts and draft his

will. To what extent if any, are Planner’s legal fees deductible under Section 213 (3)? Under Section 212 (2)? See revenue ruling 72-545, 1972-2

To the extent the legal counsel relates to tax advice

Page 60: Beck Fed IncomeTax Outline

not deductible unless loss is from a casualty loss – if taxpayer had actually rented or sold, he would have gotten the deduction but merely listing the property for sale or rent is not sufficient (there was no transaction)

c. 165-9(b)(2) – basis for a loss on a property converted from personal use is the lesser of the fair market value at the time of conversion or the adjusted basis at the time of conversion – prevents shifting of personal loss to business

2. It is not necessary to show the property was offered for rent before being sold in order to show a personal residence was converted into a transaction entered into for profit. a. The key question in determining whether a personal residence has been

converted into a transaction entered into for profit is to look at whether, in light of all the circumstances, the taxpayer had an expectation of profit. Lowry v. U.S.(1) didn’t want a tenant spoiling the property and didn’t want the hassle of

applying to the tenant’s association for the right to rent – believed the property value would appreciate

(2) can hold property for speculative income purposes (not only current income) as stated in the Regs

Chap 16: Deductions Not Limited To Business or Profit-Seeking ActivitiesI. Interest § 163

problems pg 4752) Homeowner purchased their vacation residence for $180,00 ($20,000 which was allocable to the land). When it was worth $160,000 ($20,000 of which was allocable to the land), they moved out and put it up for sale but not rent, for $170,000

a) May they take deprecations for expenses and depreciation on the residence? If so, what types of expenses would qualify?

Lowry: holding property for speculative gain and so entitle to deduct the cost of maintaining it – but this seller doesn’t look like he is holding it out for gain but that he is simply selling his house (amount he is asking is so close to market – when you sell your house, you always ask for more than you want)

b) Assume instead that they rented the property and properly took $10,000of depreciation on it. What result when they subsequently sell the property for:1) $145,000?

under Reg 1.165-9(b)(2) there are 2 choices for basis for property converted from personal use: the lesser of fair market value at the time of conversion or the adjusted basis at the time of conversion (similar to loss rules for gifts) - (180 – 10 = 170) or fair market value at conversion (160 – 10 = 150) amount realized is 145 – 150 is basis and loss is 5

2) $175,000?only use 165-9 if there is a loss – using adjusted basis there is a gain of 5

3) $165,000?conundrum – using 165-9 there is a loss using adjusted basis but a gain using fair market value (same as with gifts) – if you sell the property for an amount between the 2 possible bases, there is no gain and no loss – sold for an amount over the adjusted basis, there is a gain; sold for an amount less than fair market value (minus depreciation) there is a loss; sold for an amount in between, it is a nothing and there is no tax, no deduction

c) What result in (b) (2), above, if the property had been Homeowner’s principal residence, they had owned it and used it for 4 of the prior 5 years. And the deprecation was taken after 1997?

5 of gain which is due to prior depreciation is not protected by § 121 – since 5 of gain is produced due to the 10 of depreciation – they will be taxed on the 5 of gain

Page 61: Beck Fed IncomeTax Outline

Interest paid or accrued on indebtedness is deductible under § 163.Interest is the amount one has contracted to pay for the use of borrowed money, or the compensation for the use or forbearance of money. Revenue Ruling 69-188.

1. To be deductible, payment of interest must be incidental to an unconditional and legally enforceable obligation. Revenue Ruling 69-188.a. However, the obligation need not exist before interest is paid. b. It is also not necessary for the payment to be called interest, so long as the

payment represents payment of a set amount for the use of borrowed money and not other services.

c. Thus, payment made from one's own money to buy down interest rate points, even if called a loan-processing fee, may be deducted as an interest payment.

Under the old rule, an interest-free loan resulted in no taxable income to the borrower because had the borrower taken out an interest-bearing loan, the interest would have been deductible. J. Simpson Dean.

1. This rule was changed when Congress enacted § 7872, which covers the tax consequences of interest-free and below-market loans.a. All loans that have an interest rate that is below market, or have no interest

at all, are re-characterized such that the payment of interest is imputed.b. corporate loans to shareholders, employer loans to employees, and gift-

loans – imputes interest as income to the lender and deductible to the borrower(1) exception: an interest-free gift-loan under $100,000 is exempt unless it

is invested and earns income (to prevent income shifting) A. Personal Interest § 163(h)

§ 163(h) disallows the interest deduction for personal interest.1. Interest on qualified residences is usually deductible. This includes interest

payable on loans used to purchase a qualified residence, refinancing and home equity loans.a. limits on deductibility – acquisition limit: interest on loan of $1,000,000 –

home equity limit: interest on loan of $100,000b. acquisition – to buy or to buildc. home equity – equity in the home (but can be used to buy or build also),

loan can be used for any purpose – Beck doesn’t think this is fair d. old mortgage debts over the limits (pre 1987) are grandfathered in and all

the interest is deductible B. Qualified Educational Loan § 221

1. Interest payable on qualified educational loans may also be deductible under § 221.

C. Loans to purchase tax-exempt property § 265(a)(2) 1. Interest payments on loans used to purchase tax-exempt property, such as

municipal bonds or certain life insurance policies or annuities, may not be deducted. § 265(a)(2).a. However, for the deduction to be disallowed, more than mere

simultaneous purchase of the tax-exempt property and obtaining the loan must be shown.

Page 62: Beck Fed IncomeTax Outline

D. Investment interest § 163(d)1. Investment interest may also be deducted, but only to the extent the taxpayer

has net investment income (more income than expenses related to the

investment).

III. Taxes §§ 164(a), (b), (c), (d)(1); 275; 1001(b)(2) Reg 1.164-3(a)-(d)State, local, foreign and real property taxes are deductible. § 164 (see also § 111)Federal income taxes are not deductible. § 275Sales taxes are not deductible.

Problems pg 5053) Taxpayers purchase home in 1998 as principal residence. They obtain a loan secured by the residence. What portion of the interest paid is deductible? (a) Purchase price and FMV is $350,000. Taxpayers get mortgage of $250,000.

all of the interest is deductible – acquisition indebtedness less than $1 million (b) same as (a), but Taxpayers by 2000 have reduced balance to $200,000 and FMV of resident has increased to $400,000. In 2000, Taxpayers take out 2nd mortgage for $100,000 secured by residence to add a 4th bedroom and den

there is $200,000 of equity in the home as result of increased FMV – 2nd loan interest is deductible as home equity loan but could be acquisition debt also because it is being used to build a part of their home – they have an additional 100,000 in equity in the home after the new debt

(c) same as (b) but they use the loan money to buy a Ferrarihome equity loan – interest is deductible and can spend it on whatever they want

(d) same as in (a). Taxpayers have paid off $200,000 of the $250,000 1998 mortgage. Residence is worth $500,000. Taxpayers borrow $200,000 on residence, $50,000 used to pay off the 1998 loan and then to pay off personal debts.

borrowed 200 – 50 is acquisition indebtedness (deductible) – 100 is home equity (deductible) but interest on the remaining 50 is non-deductible

(e) same as (a), but in 1998, Taxpayers purchase 2nd home in FL for $1,250,000. They finance $950,000 secured with the home, use it 45 days a year and elect to treat it as a qualified residence.

acquisition cost – borrow 950 – already have 250 outstanding, so have $1,200,000 in loans and are over the limit – only the interest on $1 million is deductible, plus $100,000 of home equity indebtedness; interest on $100,000 is taxable

(f) same as (a), but year of acquisition was 1985. Taxpayers used $200,000 toward home purchase and $50,000 to buy a sailboat. By 1998, Taxpayers have reduced principal to $175,000. They re-finance the property (now worth $500,000) with a loan for $300,000. They use $175,000 to pay off 1985 loan and remaining $125,000 to pay off education loans.

borrow 300 – 175 is acquisition (replacing old loan), 100 is home equity – so interest on 25 is non-deductible

5) Investor incurs investment interest of $100,000. What is deductible?a) sells stock at $60,000 gain, has $20,000 in dividends and $10,000 in deductible adviser fees

interest is deductible to the extent of gain – net investment income gain can count if the taxpayer elects to treat it as income – 60dividends – 20 fees – 10 deducted

70 net income so 70 of the interest is deductible against her income and 30 of the interest is not deductible

b) the interest is on loans used to purchase tax exempt bonds not deductible –§ 265(a)(2)

c) same as above but ½ is used to purchase stocks and bonds and ½ is used to purchase tax exempt bondsonly ½ of the income would offset the expense so only 50 of the interest is deductible

Page 63: Beck Fed IncomeTax Outline

A. Property taxes Property taxes may only be deducted by the person to whom they are assessed. Cramer v. Commissioner.1. To determine whom the property taxes are assessed upon, one must look to

state statutory law to see who is responsible for the taxes (in this case, who was the owner of record or the occupant).

2. If someone other than the person upon whom the taxes were assessed pays them, they may not take the deduction. Instead, the payment is said to be a gift to the property owner.

3. If the property is sold in the middle of a year, the property taxes are allocated between the buyer and seller in proportion to the amount of time each owned the property, regardless of who paid the property taxes. Per diem as to who

owns the property.

Chap 17: Restrictions on Deductions I. Introduction

The fact that Congress has provided for a deduction does not necessarily mean the deduction will be allowed. restrictions & limitations have been placed on

Problems pg 5091) Which of the following taxes would be deductible as such under § 164?

a) state sales tax not deductibleb) state real property tax of $1000 for which A becomes liable on Jan 1st but sells land on July 1st

A can deduct $500 – B can deduct $500c) state income tax

yes – state income tax is deductible – if you overpaid and get a refund, it is income § 111 if you got a tax benefit from it – if you take standard deduction and got no tax benefit in the year of payment, it is a tax-free return of capital

d) federal income tax not deductiblee) state gasoline tax not deductible (not a general sales tax because its item-specific)(excise)

2) Which is deductible?a) state tax on cigarettes provided by the taxpayer to customers

if an item of tax is both a tax and a business expense, the character as business expense makes it deductible

b) filing fee required to be paid by candidates entering state primaryno – tax is defined as a compulsory contribution (General exaction) to a govt for govt purposes in which the taxpayer has no rights – if you buy the right to go somewhere, or use something, it is not a tax – this is not a tax but it may be a business expense – under 162, candidates filing expenses are not deductible

3) Son in college owns substantial securities. Father pays Son’s intangibles tax.a) May Father deduct? Not deductible § 164; under Cramer, he doesn’t owe it, so it is not deductible to him – 164a2 b) Is it deductible by Son? Yes, if he has sufficient income

4) Employer withholds FICA and pays FICA match. a) deductible?

she can deduct the withheld tax and the FICA as a business expense but it is not a tax to her and they are not deductible to the employee because it’s a federal tax which cannot be deducted

5) city builds road and assesses adjacent landowners. Are payments deductible?not deductible – 164c1 – no deduction for tax assessed against local benefit – he can add them to basis – the property has been improved

Page 64: Beck Fed IncomeTax Outline

deductions to eliminate tax shelters.1. tax shelter is a way of hiding income from one source using losses attributable

to another.2. There are two types of restrictions: substantive and procedural.

a. Substantive restrictions can be either takeaway provisions or postponing provisions. The goal of both is to disallow deductions for artificial losses, those losses attributable to a different activity than the one from which income is derived.

b. Procedural restrictions are rules that promoters of tax shelters must follow or be penalized for failing to comply. Generally, these require the promoter to register the scheme and keep a list of all participants.

II. Activities not engaged in for profit § 183(a)-(d)Whether an activity is engaged in for profit is determined by § 183.

A. Presumption of profit motive1. § 183 creates a rebuttable presumption that an activity that has produced gross

income in three of the past five years which exceeds the amount of deductions attributable to that activity is one engaged in for profit (the safe harbor test). The Commissioner may rebut. (2 years out of 7 for horse breeders)

2. If no presumption arises, taxpayer may qualify by using objective standards taking into account all the facts and circumstances. a. taxpayer had the objective of making profitb. it may be sufficient that there was a small chance of making a large profitc. activity carried on in a businesslike manner or in a manner similar to

others of the same nature which are profitable 3. If activity is engaged in for profit, §§ 162 & § 212 deductions re allowed

without limitation.B. Deductions if not for profit (Hobby) § 183(b)

If the activity is one not engaged in for profit, then § 183(b) allows deductions only to the extent the gross income from the activity exceeds the claimed deductions. (Hobby loss)1. This limitation does not apply to business, investment or personal deductions,

such as taxes.2. If hobby makes money, you have income. If you have deductions, apply §

183(b), deductions are limited to gross income from activity. (quarantine – deductions cannot spread to other income)a. allowable interest and taxes must be allocated to the hobby – eg. Interest

and tax on the garage where you store the antique cars you collect III. Restrictions on Deductions of Homes § 280A(a), (b), (c)(1), (3), & (5), (d)(1), (e), (f), (g)

A. Rental of HomeGenerally, § 183 allows expenses attributable to renting a home if the rental is an activity engaged in for profit. If the rental is not engaged in for profit, the restrictions of § 183(b) apply and deductions will only be allowed to the extent that gross income from the rental exceeds the deductions.

B. Limits under § 280AIf the rental is the taxpayer's residence, then § 280A limits the availability of

Page 65: Beck Fed IncomeTax Outline

deductions depending on the number of days the residence is rented.1. If the residence is rented less than fifteen days during the year, no deductions

are allowed. However, income from the property is not required to be included in the taxpayer's gross income.

2. If the residence is rented more than fourteen days, deductions are allowed only insofar as the deductions that are attributable to the rental activity exceed the deductions allowable on the residence in general, irrespective or its being rented.a. The remaining deductions may be taken, but only to the extent the rental

income exceeds allocable interest and taxes.3. A residence is a dwelling unit that the taxpayer uses for personal purposes

more than the greater of fourteen days or ten percent of the days for which it is rented. If less than that, it is a rental unit. a. A taxpayer's personal use may be disregarded for purposes of determining

whether property constitutes a residence if the period of personal use precedes or follows a qualified rental period.

b. A qualified rental period means rental of the property for twelve consecutive months.

c. deductions can only be taken to the extent of net incomegross rental income less interest and taxes otherwise deductible on a per diem basis for rental periodthis gives you the 280A limit on deductible rental income expensesminus out of pocket expenses for the business use allocated to rental periodminus depreciation allocated to rental period eg: Taxpayer owns vacation home. Rents for 60 days and uses it for 30

days per year. Gross rental is $5400. Real estate taxes and qualified interest are $3600. Other expenses related to the home are $7800 ($1500 for repairs and $6300 for depreciation). House was rented for more than 14 days and used by taxpayer for more than the greater of 14 days or 10 percent of actual rental time (Taxpayer does not meet the minimal personal use requirement of § 280A(d)). Taxpayer’s deductions are limited by gross rental income as follows:Gross rental 5,400Less allocable portion of taxes and interest, 60/365 of $3600 600§ 280A(c)(5) limit on deductions 4,800

Allocable portion of expenses 60/90 of $1500 1,000Allocable portion of depreciation 60/90 of $6300 4,200Depreciation deduction gets limited to $3,800 because that is the max allowable after deducting other expenses

C. Home Office § 280A(c)Expenses attributable to a home office are deductible under § 280(a) so long as a portion (based on space) of the home is used exclusively and on a regular basis as the principal place of business, as a place of business which is used by patients,

Page 66: Beck Fed IncomeTax Outline

clients or customers, or as a separate structure not attached to the dwelling unit. 1. An employee may also deduct home office expenses if the home office exists

solely for the convenience of the employer.2. Deductions are limited, however, to the amount gross income from the home

office exceeds deductions allowed for the home in general, irrespective of the home office, and all other expenses attributable to the business but not the home.a. Deductions disallowed due to income limitations may be carried over to

subsequent years.

IV. Illegality or Impropriety § 162(c), (f), (g)A. Criminal Defense

Public policy does not bar otherwise valid deductions for ordinary and necessary legal expenses incurred in defending criminal charges arising out of carrying on a business. 1. Public policy exception may be applied to disallow an otherwise valid

deduction if allowing the deductions would frustrate a clearly articulated nationwide or statewide policy. Commissioner v. Tellier.

2. The test for non-deductibility is the severity and immediacy of frustration that would result if the deduction were allowed.

3. § 162 c & f – no deduction for fine or violation of law and no deduction for illegal kickback – if not one of those, there is not a denial of deduction due to

Problems pg 5251) Taxpayer owns a vacation home that rents for 90 days and uses for 30 days.

a) Limited by § 183 or § 280A?§ 280A limitations apply because the house is used personally for more than 10% of the time rented (and more than 14 days)

b) what is deductible?gross income from rent 3000interest & tax deduction – 90/365 x 2000 = 500 (Beck calls this a haircut)so we have net income of 2500out of pocket 90/120 x 1600 = 1200 deductibleso now have 1300 deductible left depreciation 90/120 x 2000 = 1500 depreciationbut only 1300 of the depreciation is allowed

c) are taxes and mortgage interest deductible? yes fully deductibled) what if T rents the home for 3 weeks and uses it for 1 week

rental income only - § 183 applies – see if it made money in the last 3 years out of 5 similar calculation but all the interest and taxes would reduce the income and no carry forward

2) Taxpayer has home office. Used exclusively and on a regular basis as principal place of business2000 gross income-400 I/T1600 income1600 expenses for secretary and supplies0 net income350 of utilities and depreciation can be carried forward

3) Widow rents rooms to students. Is she limited by § 280A?widow is not limited by 280A – boarders are treated as not in your home and are fully deductible as rental income reg § 1.280A-1c

Page 67: Beck Fed IncomeTax Outline

public policy a. eg: hitman expense for bullets would be deductible

Punitive damages deductible unless they are antitrust treble damages

Chap 18: Deductions For Individuals Only I. Adjusted gross income

Adjusted gross income is a means of equalizing taxable income between individuals based on the source of their respective income.

Adjusted Gross Income and Deductions- ABOVE THE LINE DEDUCTIONS § 62A. Computation- A number of deductions constitute a subtraction from gross income to produce a concept called adjusted gross income, or AGI. B. Principles Deductions under AGI- § 62 defines adjusted gross income, allowing the following principal deductions from gross income

i. Alimony- A taxpayer may deduct the amount of alimony he or she pays. § 215ii. Certain retirement savings.- A taxpayer may deduct certain contributions to an individual

retirement account (IRA) and other plans. § 219iii. Moving Expenses- A taxpayer may deduct qualifying moving expenses incurred in moving

his or her household to take a new job. § 217iv. Business expenses & losses- If a taxpayer’s expenses of doing business exceed the income

from that business the net loss will be deductible subject to certain limitations (includes “washes” of reimbursed employee expenses). § 165 ( c) (1)

v. Capital losses- A taxpayers capital losses are deductible, subject to limitations of § 1211Taxable Income and Deductions. – BELOW THE LINE DEDUCTIONS § 63a. Computation. – A second group of deductions is subtracted from AGI to produce taxable income.

The taxpayer may subtract either the standard deduction or the itemized deduction (but not both), and the rational taxpayer will choose to deduct the larger of the two. § 63 defines taxable income

b. Standardized Deduction- The standard deduction is a statutory fixed amount based on the taxpayers filing status

c. Itemized Deductions- the itemized deduction is the aggregate of a number of deductions, principally the following.i. Home mortgage interest.- A taxpayer may deduct qualified residence income. § 163ii. Taxes.- A taxpayer may deduct state and local income and property taxes, but not sales tax.

§ 164iii. Casualty loses,- A taxpayer may deduct the excess of casualty losses over casualty gains to

the extent that they exceed 7.5% of the AGI. § 213iv. Medical expenses- A taxpayer may deduct certain medical expenses to the extent they exceed

7.5% of AGI.v. Charitable contributions- A taxpayer may deduct contributions to certain qualifying

organizations, subject to certain limitations. § 170vi. Bad debts- Debts that become worthless during the taxable year generate a loss, the character

of which depends on whether the debt is a business or nonbusiness bad debt. whether the business . § 166

vii. Miscellaneous expenses. Certain expenditures are deductible only to the extent that they exceed 2% of the taxpayer’s AGS. § 67

d. Personal exemption

Section 61 tells you what Gross Income is.Section 62 gives you a round of deduction which gets you to adjusted gross income. Section 62 is considered above the line deduction. Section 62 gets you from Gross income to AGI Section 63 is a second class of deductions which gets you from AGI to taxable income

Page 68: Beck Fed IncomeTax Outline

* Gives you a choice between standardized deductions and itemized deductions

II. Moving expenses §§ 62(a)(15); 82; 132(a)(6) & (g); 217 Reg 1.217-2(b)(2), (3), (4), & (8)

Moving expenses are deductible above the line. See § 62(a)(15).A. Distance & Time

Under § 217, moving expenses may be deducted provided the distance between the taxpayer's former residence and new workplace is more than fifty miles than the distances between the taxpayer's former residence and old workplace and the taxpayer is employed for at least 39 weeks of the first twelve months at the new location. (doubled if self-employed)Eg: old residence: 5 miles to old work – New work must be 50 miles + 6 miles - regulations hint that the move must be to the general area of the new work – if no current job, you can get the deduction for a move of 50 miles or more

B. What is included

Problems pg 5481) Which are these? § 62 or § 63 deductions?

a) police uniform – an unreimbursed employee expense – below the line and subject to 2% floor (§ 67) b) salesman unreimbursed expense of entertaining client - below the line – misc itemized deduction – only 50% under 274n2 and subject to § 67c) what if employer reimburses – see Reg 1.162-17(b)(1), (2), (3) & (e)(3)

(1) if reimbursed, it’s a wash - for employer, 50% haircut on deduction to employer (2) overpaid reimbursement, extra is income(3) underpaid, the remainder is a deduction

d) what if employer entertains the client? above the line but 50% haircute) tuition for business refresher course relating to employment - below the line subject to § 67f) payments for the following items: below the line – mortgage interest, medical expenses (subject to 7.5% floor), charitable contributions and taxes on propertyg) taxes and interest relating to rental unit - above the line – business expensesh) loss on stock held for investment - above the line capital loss – limit $3,000j) state income taxes - below the line – not treated as an expense of the business (it is the personal income)k) tax prep fee, part for personal, part for business - $150 above the line (business expense) & $250 below the linel) alimony payment - above the line m) deductible moving expense - above the line – work related n) teacher buys $350 in supplies - only $250 is above the line § 62(a)(2)(d)

Page 69: Beck Fed IncomeTax Outline

Moving expenses include amounts paid to pack, ship, transport, insure and store the taxpayer's possessions and the cost of moving, excluding meals, which the taxpayer's employer did not reimburse.

III. Medical Care §§ 213(a), (b), (d)(1)-(4) & (9); 263(a)(1) Reg 1.213-1(a)(1), (e)(1)-(4)(i)(a)

Amounts paid for the taxpayer's, the taxpayer's spouse or dependents' medical care are deductible under § 213, subject to 7.5% of AGI floor for deductibility and only if you itemized your deductions.

A. InclusionsMedical care includes amounts paid for the diagnosis, cure, mitigation, treatment or prevention of disease and for transportation primarily for and essential to the diagnosis, cure, mitigation treatment and prevention of disease.

B. Addition to home1. Expenditures that constitute a permanent addition to a home are deductible as

medical expenses only insofar as they do not increase the value of the home over the amount expended. Raymon Gerard a. deduction allowed for amount over which the expenditure increased the

value of the homeC. Food and lodging

1. Food and lodging expenses incurred en route to a place of medical treatment, but not after arriving, are deductible as medical expenses. Montgomery v. Commissioner.a. Reasonable lodging expenses and 50% of food expenses paid while

Problems pg 5531) L works in Town X and lives in Sub X 10 miles away. Decides to work in Town Y and moves family to Town Y.

a) How far away from Sub X must Town Y be for his moving expenses to be deductible?60 miles from suburb of X

b) How far away from Sub X must Town Y be located if L just graduated & this is his first job?50 miles from old home

c) If self employed what time requirement?78 weeks

d) What if L is a partner in law firm?partner is self employed, so 78 weeks

e) What if only an associate (and so an employee)?only 39 weeks

f) What is deductible for moving? $400 for movers, $150 for transp, $100 in lodging & $200 for meals$ 650 above the line; everything but the meals

g) any difference if L’s spouse also takes a job in Town Y?no difference; only one of them can take the deduction

h) What if employer reimburses him all $850 of expenses?$200 income

2) employee moves and deducts expense but gets disallowance notice from IRS. deductions must be taken in the year of the move – she can amend her 1998 return within the next 5 days (3 year statute of limitation on amending)

3) Employer moves employee to town and she buys a house. They ask her to move again and she cannot sell her house without a loss. Employer buys the house for her cost.

not a cost of moving for Keen – she got her basis, but the $5,000 avoidance of loss could be considered income – employer has a $5,000 business expense (cost of labor)

Page 70: Beck Fed IncomeTax Outline

receiving treatment from a hospital on an in-patient basis are deductible under § 213(d)(2).

D. Long term careAmounts paid for qualified long-term care of a taxpayer, the taxpayer's spouse or dependents is also deductible.

E. Business expenses1. In some instances where the nature of the taxpayer's work clearly requires a

medical expense be incurred to insure sufficient performance or where goods or services purchased are primarily used for work and the IRS Code and Regulations are silent on their deductibility, such expenses, although medical in nature, may qualify as business expenses under § 162. Three elements must be present:a. nature of work clearly requires itb. the expense is not paying for something used in personal activitiesc. Code and Regs are otherwise silent as to the treatment of the expense

IV. Exemptions §§ 151, 152, 7703 Reg 1.151-1(b) & (c)(2)In addition to specific deductions available to individual taxpayers, § 151 provides exemptions for the individual taxpayer, his spouse and any dependents. $2,000 per person below the line

A. Personal exemption § 1511. available to the individual taxpayer and his spouse without restriction

a. unless eligible to be claimed on another’s return as a dependent § 151(d)(1)

b. joint return gets 2 exemptionc. filing separately: can only claim spouse if spouse has no gross income §

151 (b)2. a set deduction that is subtracted from the taxpayer's adjusted gross income to

determine the taxable amount. Use $2,000 for this class3. § 151(d)(3) – phase out: if you make too much money (4 different versions)

you lose 2% of your otherwise eligible exemptions for each $2500 AGI exceeds this threshold – instead of increase the tax rate, Congress cut into the exemptions – unfair, affects taxpayers unequally – this creates a bubblea. § 7703 marital status definition: determined at end of year

(1) if married on last day of year, then married all year(2) legally separated is considered not married

Problems pg 5651) medically necessary addition to home: cost $4100, increased value of home by $2100; other medical expenses of

$1020a) deductible: $2000 for AC, then $1020 more – apply 7.5% to get $900 – first 900 is out so she can deduct 2120b) deduct $400 per year?

deduction has to be taken in the year the expense incurred – depreciation does not apply2) employee traveling on business gets ill and has to stay in hotel for 2 extra days

deductible as a business expense 3) patient pays doctor up front

payment to the doctor is a payment against future services

Page 71: Beck Fed IncomeTax Outline

(3) married is considered not married § 7703(b) if taxpayer:a. files separate returnb. maintains household where child/dependent lives for more than ½

the yearc. spouse does not live there for last 6 months of the year

B. Dependents §152 Reg 1.152-1, -4T1. To claim a dependent exemption, the taxpayer must show:

a. relationship test: that the person claimed as a dependent is a blood relative, adopted or foster child, or an unrelated individual living as part of the taxpayer's household, and

b. support test: that the taxpayer provides over one-half of the support for the person claimed as a dependent(1) if more than ½ support is provided, the dependent does not have to be

living with the taxpayerc. gross income test: the person claimed does not have gross income over the

amount of the exemption allowed.(1) The gross income requirement is waived if the dependent is the

taxpayer's child and is under nineteen or a full-time student under twenty-four; child can earn any amount if parent pays more than ½ their support

(2) § 152(d) scholarship does not count as support – see Reg § 1.152-1(3) Spouse is never a dependent.

2. Custody rule: § 152(e) – custodial parent can claim without reference to support test – exemption can be transferred to non-custodial parent if done in writing – see Reg § 1.152 – 4T

Page 72: Beck Fed IncomeTax Outline

3. § 152(c) – multiple support agreements: if qualified related people have provided over ½ the support in aggregate, one who paid over 10% of the

support can claim the exemptionFor non-related dependents, you are allowed the exemption deduction only if the dependent lives with you, and if the cohabitation does not violate local law

Problems pg 5711) married, separate return, spouse has no income – how many exemptions?

a) spouse earned interest on tax exempt bonds2 exemptions for himself and spouse – interest is not included in gross income

b) received $100 gift from uncle2 personal exemptions – gifts are excluded from income

c) spouse earned $50 gain from sale of the bonds1 exemption – gain is income, so spouse exemption not allowed

d) since spouse exemption disallowed, can taxpayer claim as spouse as dependentno, § 152(a) spouse can never be a dependent

e) taxpayer and spouse file joint return2 exemptions

2) married, separate return, furnished over ½ support of person earning less than $2000 & not living with taxpayer – Is this a dependent?

a) wife’s brother § 152(a)(9) – brother in law is dependentb) 1) wife died year before

relationship persists even if link is broken – still a dependent 2) H & W divorced

same resultc) wife’s sister’s husband

relation too far away – double in lawd) wife’s sister’s husband lived with taxpayer during year

dependent exemption allowed under § 152(a)(9)e) son earned $2000 but is full time student

dependent f) daughter earned $500 but married during year and filed joint return

not a dependent under § 151(c)(2) g) but daughter’s spouse had little income and filed return only to get a refund

can still be claimed as a dependent if there is no tax owed because there is no tax consequence to the joint return

h) 18 year old son, taxpayer pays $2000 in support; $3000 from uncle for supportfather cannot claim son but uncle might be able to (but this is a gift)

i) 18 year old son, taxpayer pays $2000 and son gets $3000 scholarshipscholarship doesn’t count so father can claim son as dependent § 152(d)

Page 73: Beck Fed IncomeTax Outline

C. Standard Deduction § 63, 67, 68, 7703 Reg 1.67-1T(a)1. An individual taxpayer may take the standard deduction provided for in § 63,

or taxpayer can itemize deductions. – choose the higher of the twoa. this is in addition to personal exemptions as aboveb. and in addition to § 62 deductions

2. The standard deduction is a set amount based upon the taxpayer's status as single, head of household, married, blind or elderly. a. $3000 for individual – use this for this class § 63(c)(2)b. $5000 for joint returnc. $2500 for married filing separately d. get another $600 if 65 or older § 63(f)(1) – considered 65 on the day

before your 65th birthday – Reg § 1.151-1(c)(2)e. get another $600 if blind § 63(f)(2)

(1) if unmarried, over 65 and/or blind get $750 for each § 63(f)(3)f. limitation on dependent standard deduction – if eligible to be claimed as a

dependent by another taxpayer, the standard deduction is the greater of $500 or $250 + earned income (remember someone eligible to be claimed as a dependent by another gets no personal exemption and is limited as to standard deduction as stated here) – Disadvantages Trust Fund Kids

3. § 67: 2% floor applies to § 212 and unreimbursed business expenses 4. § 68 phase out: if you make too much money (over $100,000), you loose 3%

of itemized deductions in excess over AGI up to 80% of the deductionseg: $200,000 AGI - $100,000 over threshold in § 68(b)(1) – he looses 3% of

excess so he loses $3000 of his itemized deductions 5. Sinyard v Commissioner: tort award, 1/3 to lawyer – if this were a business

expense under § 162, the payment to lawyer would be a wash (above the line deduction) – but this is a personal suit award, so is either under § 212(1) production of income or an unreimbursed employee expense but either way it is a miscellaneous itemized deduction under § 67 and is below the line (and would have to exceed income by more than 2%)a. the problem in this case is that the taxpayer had used Alternative

Minimum Tax and so was allowed no deduction at all b. what if considered the cost of acquiring property and capitalize it? Beck

says this would make the problem go away – no income realized

3) father has no gross income – $4000 support provided by X 15%, T 25%, A 20%, B 10% and C 30% - who gets the dependent exemption?

Under § 152(c) T, A, and B have contributed 55% in aggregate – either T or A can claim (they are over 10%) could claim the exemption if the other files a declaration that he is not claiming it

4) W & H divorce; W gets custody of son – who gets dependency exemption?a) W supports 40% & H support 60%

W has custody and the exemption – no support testb) W waives exemption

H can claim it under § 152(e)(2)c) son lives with Grandpa for 9 months

nothing changes, legal custody is what mattersd) not divorced, but W moves out – son lives with H – support provided equally

H gets exemption if there is no legal custody, then defacto custody is based on time spent

Page 74: Beck Fed IncomeTax Outline

c. under § 265(a)(1): legal fees to earn a personal injury tax free award (non-income under § 104a) are non-deductible – if award is part tax-free and part taxable, then an allocation of legal fees is made

Legal fees for civil rights cases are above the line deductions

Can’t deduct fees for obtaining tax-free income

Problems pg 5751) $20,000 AGI, single personal exemption with the following itemized deductions: 1000 interest, 500 taxes, 1500 unreimbursed employee expenses, 200 tax prep, 300 bar dues

a) what is the taxable income?§ 62 deductions = 1500 tax and interestthen calculate § 67 subject to 2% floor 1500 unreimbursed employee expense + 200 tax prep + 300 bar dues = 20002% x 20,000 = 400 – so only 1600 is deductibletotal deductions are 1500 + 1600 = 3100taxpayer can take standard deduction of 3000 or itemize 3100taxpayer can take 1 exemption for $2000 taxable income is 20,000 – 3100 – 2000 = 14,900tax rate under § 1 of code, unmarried is 15% of 14,900

b) what if taxpayer is 65reg § 1.151-1(c)(2) considered 65 on the day before your 65th birthdayunmarried and 65 gets an additional $750 – so standard deduction is $3750 and he will take the standard instead of itemizing – 20,000 – 3750 – 2000 = 14,150

c) married filing joint return$5,000 standard deduction and 2 exemptions for $4,000 = $11,000 taxable income

d) if interest is $4000then their itemized deductions are greater than the standard deduction 4500 + 1600 = 6100 > 500020,000 – 6100 – 4000 = $9,900 TI

2) single taxpayer, child of X, student has investment income of $4000 and no deductions – X claims student as dependenta) what is taxable income?

4,000 AGI – exemption of 0 (he is claimed by another taxpayer) – standard deduction of 500 (investment income is not earned income) = $3,500

b) 1000 earned income and 3000 investment income4000 AGI - 0 exemption - 1250 standard deduction (earned income plus 250) = 2750 TI

c) 4000 earned income personal exemption still 0 and takes standard deduction of $3000 = $1000 TI

3) who should itemize? A pays 4000 state income tax and B pays noneA will itemize (state income tax is below the line; 4000 > 3000) and B will take the standard If 4000 is alimony paid by A, then it is deducted before AGI and she gets the standard deduction also

4) H & W file separatelyif one itemizes, both must itemize – if one takes standard, both must take standard

5) H & W live apart; children live with W $3000 is standard for single - $2500 is for married under § 7703b, she is unmarried and properly takes the $3000 standard deduction H is still considered married can only take the married filing separately deduction of $2500

6) multiple support agreement - X pays 2000 for food, Y pays 2000 for lodging and Z pays 2000 for medicalthese are valid family members for tax purposes so any of them could claim Dad as dependentimportant issue is which one of them is in the higher tax bracket and needs the deduction? And medical expenses are deductible for dependents, so if Z’s medical deductions exceed 7.5% of AGI, he could use the deduction

Page 75: Beck Fed IncomeTax Outline

The Year of Inclusion or DeductionChap 19: Fundamental Timing PrinciplesI. The Cash Method of Accounting: Cash Receipts and Disbursements

A. Receipts § 446, 451 Reg 1.446-1(c)(1)(i); 1.451-1(a), -21. A cash method taxpayer includes income in the year it is received. Income

may be in the form of cash, check (treated as the same as cash), or property.2. Under the cash method of accounting, the value of a check actually received is

includable in the year of receipt, even if the check cannot be converted to cash until the following year. As such, the receipt of a check is deemed a recognition of income. Charles F. Kahler. a. could have endorsed the check over to someone else – he had control of

the funds - 3. Promissory notes or other evidence of indebtedness received as payment for

services constitute income to the extent of their fair market value. However, a note received only as an evidence of indebtedness, and not as payment, will not be regarded as income at the time of receipt. Williams v. Commissioner.a. In Williams, the court determined that an unsecured promissory note,

which has no fair market value, is not the equivalent of cash and as such is not includable in a cash method taxpayer's income in the year of receipt.

4. A readily marketable agreement (obligation/promise) to make future payments that qualifies as the equivalent of cash is taxable upon receipt as cash had it been received by the taxpayer instead of the obligation. Cowden v. Commissioner.a. An obligation is a cash equivalent if

(1) The obligor is solvent,(2) There is an assignable and unconditional promise to pay, not subject to

set-offs, and(3) The obligation is of a type normally transferable to lenders at a

discount not substantially greater than the prevailing premium for money.

5. Even when a taxpayer does not have the physical item of income in her hands (actual receipt), it may be that the taxpayer is deemed to have constructively received the income, and is therefore liable for the tax due on that income. Paul v. Hornung.a. Constructive receipt occurs when income is credited, set apart, or

otherwise made available to a taxpayer unless the taxpayer's control of its receipt is subject to substantial limitations or restrictions. Income Tax Reg. § 1.451-2(a).

b. The idea behind the principle of constructive receipt is that if income is made available to a taxpayer, and the only thing that keeps her from collecting the income is her own action (or inaction), then the income is received and must be taxed; a taxpayer cannot turn her back on income to avoid taxes. – Compare in deferred compensation situations – constructive receipt would kick in unless the taxpayer is in a position to assume the risk a an unsecured creditor for their own salary to be paid out in the future

B. Disbursements § 461(a) & (g) Reg 1.461-1(a)(1)

Page 76: Beck Fed IncomeTax Outline

1. Generally speaking, a cash method taxpayer can only take a current deduction for expenses actually paid during the taxable year.

2. However, when expenses relate to the creation of an asset which has a useful life substantially beyond the taxable year (a capital asset), taxpayers are required to prorate those expenses and may not take the full deduction for the expense in the year of actual payment. Commissioner v. Bolyston Market Ass'n.a. For example, if a taxpayer pays $1,000.00 in the year 2001 for a three year

insurance policy on real property (that is a capital asset), she must allocate the $1,000.00 over the three year term of the policy and deduct the corresponding pro rata share each year.

b. but the party who receives the payment must declare the amount received as income

3. In some situations, the IRC specifically provides for the deductibility of expenses by cash method taxpayers. One such situation is covered by § 461(g).a. § 461(g)(1) requires that taxpayers who employ the cash method of

accounting allocate deductions for prepaid interest to the specific periods to which those deductions relate (i.e., prorate the value of prepaid interest over the life of the loan).

b. There is one exception to this requirement, however, which is codified in § 461(g)(2). As stated in Cathcart v. Commissioner, that section provides that the general rule contained in subsection (g)(1) does not apply when cash method taxpayers prepay points (a processing fee paid at the time the loan is taken out) on their home mortgages (whether incurred to purchase or improve a home) with funds not obtained from the lender (i.e., the funds come from their own pockets). Those who qualify for this exception are entitled to deduct the entire amount of prepayment in the year in which it is paid. See also Rev. Rul. 87-22.

c. Cathcart borrowed the points as part of the loan and was not allowed to deduct them in the current year.

d. No deduction for points on re-financing (not for buying or building). If a loan is part re-financing and part new loan, the portion for re-financing is not deductible because it’s not acquisition indebtedness for these purposes. Rev. Rul. 87-22.

4. Though the law recognizes a doctrine of constructive receipt, there is no doctrine of constructive payment; deductions are only permitted when they are actually paid. Vander Poel, Francis & Co., Inc.a. mail drop counts as payment

5. A payment for a deductible expense made to a third-party with a bank-issued credit card is a deduction when the charge is made. a. exception: A payment made to the issuer of the credit card (e.g., a

department store credit card) is not deductible when charged, but only upon payment of the credit card bill.

Page 77: Beck Fed IncomeTax Outline

The Characterization of Income and DeductionsChap 21: Capital Gains And Losses § 1( H ), 1222 I. Introduction

To determine how certain gains and losses are taxed, they must be analyzed quantitatively and qualitatively because certain types of gains and losses are entitled to special treatment.

A. Capital Gain1. Three necessary elements for capital gain:

a. capital asset in hands of taxpayerb. sale or exchange of the capital assetc. holding period of at least 1 yr and a day

2. long term capital gain tax rate is 15%3. short term capital gain tax rate is taxpayer’s ordinary income rate (max 35%)

but it is not ordinary income – it is much more valuable than long term gains or ordinary income – reason: the loss rules

4. system of capital gains and losses is a quarantine system (like the investment interest income system): deduction against gains can only be deducted against related income

5. capital losses are deductible infinitely against capital gains - if losses exceed gains, you can only deduct $3000 against other income

example of why short term gains are better than ordinary income:lawyer earns fees of $100,000 – $100,000 in capital losses - What is his taxable income? Because there were no capital gains, he can only deduct $3000 and he has $97,000 in income - the other $97,000 of loss is carried forward until it is depleted if the income is not legal fees but is short term capital gains, he now has $0 taxable income

II. Calculating Capital Gains § 1222A. Net from the inside out

To calculate the tax on a capital gain realized by a noncorporate taxpayer, the following steps must be followed:1. First subtract short-term capital losses from short-term capital gains and long-

term capital losses from long-term capital gains. 2. If there is gain on both sides, the net short-term gain is taxed as ordinary

income and the net long-term gain is taxed under the applicable capital gain rate (usually 20%). Example: make a column for short term & a column for long term Short Term Long Term 10 50(20) (10) 30 30 20 net short term gain (ordinary tax rate) 70 net long term gain (taxed at 20%)a. The applicable rate for net long-term capital gains depends on the type of

asset involved. Collectibles are taxed at 28%, the sale of depreciable real estate is taxed at 25%, most other capital assets, including stocks and

Page 78: Beck Fed IncomeTax Outline

bonds, are taxed at 20%. In certain cases, capital assets may be taxed at 18%, 15%, 10% or 8%.

b. if you have a loss in one column and a gain in the other, simply net the two. If the net results in a short term gain, you are taxed on it at the ordinary rate; if it results in long term gain, then taxed at 20%

Short Term Long Term 10 10(20) (10) 50 (30) 40 net short term gain (30) net long term loss

Here, 30 net short term gain is taxed at the ordinary rateIII. Capital Losses § 1211, 1212

A. LimitationUnder § 1211(b), capital losses may be deducted first against capital gains and

then, to the extent they exceed capital gain, may offset no more than $3,000 of ordinary income.

Short Term Long Term 10 50(20) 10(30) (30)(40) net short term loss 30 net long term gainHere, the net is 10 of short term loss; so 3,000 could be deducted and 7,000 of short term loss would be carried forward to subsequent years

1. if losses are experienced in both columns, then the $3,000 comes out of short term gains first – why is the $3000 taken from short term first? Because it is better to deduct short term loss – losses keep their character so carrying forward short term losses is better because it reduces ordinary income which is taxed at a higher rate – but the rule requires offsetting the short term – Beck calls this an anti-taxpayer rule because more long term loss

B. Carry forwardAny capital losses that may not be deducted in a given year may be carried over to offset capital gains, and again, to the extent they exceed capital gains, up to $3,000 of ordinary income, in subsequent years. § 1212(b)1. When carrying over capital losses, they retain their original character as short-

term or long-term and offset the short-term and long-term capital gains in the subsequent year to which they are carried.

#24 11/25/02 Chap 21 D – F Capital Gains (skip D3)IV. Definition of Capital Asset § 1221

property held by a taxpayer whether or not connected to his trade or business – but not inventory or copyrights – includes personal use assets (loss on home is non-deductible but gain is taxable as capital gain)

A. Exceptions1. eight classes of property are excepted out of the definition such that if the

property falls within one of those eight classes, it is not a capital asset by definition.

Page 79: Beck Fed IncomeTax Outline

a. inventory – for a dealer or merchant and sold in the ordinary course of trade(1) same property in the hands of one taxpayer may be capital assets while

in the hands of another taxpayer would be inventory (a) this occurs most frequently with land - land can be either capital

asset or inventory - if that is the dealer’s business, gain will be taxed as ordinary income; if investor, as capital gain – but can occur with anything, like bottles of wine or stock

(b) § 1236 requires dealer in stock to identify it immediately as either inventory or capital asset

b. depreciating business property – gets better treatment under § 1231 than capital assets; losses are not limited by the capital loss $3000 limitation; get ordinary loss – Beck calls this a pro-taxpayer rule

c. copyright material when gain is realized by the one producing the material (or one who takes his basis, ie heir) – if copyright material is bought and re-sold, the gain to the re-seller is taxed as capital gain (1) exception: patents – § 1235 - all income forever is treated as capital

gain Beck says: Congress loves inventors and hates artists

d. accounts or notes receivable – back up to the inventory exceptione. government publications received for less than ordinary purchase pricef. commodities derivativesg. hedging transactions are always ordinary gains or losses (not capital)h. business supplies.

2. In determining whether property fits within the statutory definition, the stated purpose of the taxpayer is of no importance.a. If it appears from the facts and circumstances that the property was held

primarily for sale to customers in a taxpayer's trade or business, the property is not a capital asset. Mauldin(1) A number of helpful factors in determining whether property sold by a

taxpayer was held primarily for sale to customers in the ordinary course of trade or business: (a) purposes for which the property was acquired, whether for sale or

investments(b) continuity and frequency of transactions where the sales were in

furtherance of the occupation of the taxpayer (5 or more transactions a year)

(2) Mauldin was found to be a dealer even though he never replaced his inventory

(3) dealer will subdivide, improve the property, sell at retail (4) can be a dealer in one year and not in the next depending on the

circumstancesb. To determine whether property is held primarily for sale to customers,

primarily means primary and not simply a substantial purpose. MalatB. Income Property

Often the trick in applying the rules for capital gains and losses lies in knowing

Page 80: Beck Fed IncomeTax Outline

what is a capital asset and what is merely ordinary income-producing property. 1. § 22(a) defines gross income as including rent payments. Thus, when a

landlord cancels a lease at the request of a tenant and in exchange for a sum certain less than the landlord was entitled to under the full-term of the lease, the amount received by the landlord is ordinary income. Horta. Moreover, the landlord cannot deduct the amount of rental payments that

he gave up the right to receive as a loss.b. the payment was a substitute for rent, not a return of capital

2. Tenants are treated differently than landlords with regards to cancellation of a lease. a. When a lease is canceled or a leasehold interest is sold, the amount paid to

compensate the tenant for giving up his interest in the lease is a capital gain because the transaction is said to be a sale or exchange of a capital asset. Metropolitan Building Co.(1) the giving up of a lease by a tenant fits the legal requirements of a sale

or exchange and a gain realized by the tenant on such a transaction is capital gain

(2) in Hort, the tenant left and the landlord still had the asset but in Metropolitan, the payment was to the landlord to leave and Olympic received the leasehold (an asset with a value)

3. Selling the rights to money under a lottery win does not result in capital gains and is treated as ordinary income Watkins v. Commissioner4. Selling tenure, etc. rights to earn ordinary income from salary are treated as ordinary income when you sell these rights

V. Sale or Exchange RequirementUnder § 1222, capital gain or loss, whether short-term or long-term, requires a

sale or exchange of a capital asset. A. Introduction

Determining when a sale or exchange of a capital asset has taken place is fairly simple. 1. In certain situations, however, a sale or exchange may occur that results in

imputed capital gain. a. For instance, when a trustee uses securities to satisfy a specific bequest of

cash, the amount saved by the trustee from not having to disburse cash is imputed as a capital gain to the trust or estate. Kenan(1) barter rule: if there is a debt and the taxpayer pays off his debt with

appreciated property, it is a sale and there is a capital gain (2) in this case, there was a pecuniary obligation to pay a specific dollar

amount – if the gift had been of x shares of stock, the value would not have mattered and there would have been no tax to the trust

2. § 1271 Treatment of amounts received on retirement or sale or exchange of debt instruments: amounts received by the holder on retirement of any debt instrument shall be considered as amounts received in exchange thereof. (so you get capital gain or loss; usually bond debt)a. This rule overrules Galvin Hudson (since 1997): the payment of money in

Page 81: Beck Fed IncomeTax Outline

satisfaction of a debt is a sale or exchange as far as the creditor is concerned (even though when the debt is paid, even if for less than full value, no property is acquired by the creditor). Galvin Hudson

Skipped a lot – go to the tape-----------------------------------------------

3. There are also specific situations that the tax code defines as a sale or exchange, even though no sale or exchange has actually occurred. a. For instance, under § 1235 obtaining licenses to use patents or copyrights

is treated as a sale or exchange, even though no property technically changes hands.

B. Correlation with prior transactionsIn determining whether a sale or exchange has taken place, as well as in looking at whether or not an asset is capital or ordinary, reference is often made to tax treatment of transactions in prior tax years. 1. Looking back at transactions in prior years does not violate the annual

accounting method used in computing income tax liability since prior years are not actually reopened. Arrowsmitha. Thus, if a judgment is obtained against a dissolved corporation and paid

by former stockholders who became liable solely because of distributions of the corporation's liquidated assets, the payment of that judgment is a capital loss since the distributions, when made, were capital gains. Arrowsmith

2. Looking back at prior tax treatment of transactions also aides in determining whether money that was previously included in a taxpayer's gross income under the claim of right doctrine and refunded in a later year may be deducted in full or only in part. Skelly Oil Co.

-----------------------

VI. The Holding Period§ 1223 requires that for a capital asset to qualify for long-term treatment, it must be held for a year and a day

A. Acquisition dateThe holding period for a capital asset begins to run on the day following the date of acquisition of the asset. Revenue Ruling 66-71. Thus, if a capital asset is acquired on 1-1-01, it will not be entitled to long-

term capital gain or loss treatment until 7-2-01. B. Trade Date

The calculation of the holding period is no different for debentures (unsecured bonds) sold on the registered market or over the counter. 1. Although such debentures are acquired through execution of a contract and

delivered at a later date, the date they are contracted for, known as the trade date, does not start the clock running for purposes of the holding period. The holding period begins to run from the date after they are delivered to the date they are sold. Revenue Ruling 66-97a. The date debentures are sold is the trade date. Revenue Ruling 66-97

Page 82: Beck Fed IncomeTax Outline

In a gift, donee takes the donor holding period – holding periods tack Any property from a decedent which is sold is automatically long term

#25 12/2/02 Chap 22 A – B, D §§ 1231, 1245tape middle of AManner of property – has to be long term (minimum yr and day)

3 Means of Disposition:salecondemnation or requisition by government – involuntary transactiondestruction(fire, flood, shipwreck, storm, theft) involuntary conversion

2 Categories of Property:trade or business property – get 1231 treatment in all 3 means of dispostioninvestment property gets 1231 treatment if the conversion is involuntary (last 2) but sales

are not 1231

2 big wrinkles:1231(a)(4)(c): “Firepot” – All gains and losses from destruction; if there is a gain

(insurance), combine it with all the others – but if the net in destruction conversions are losses, take the loss, separate from the other 2 types of transactions – very good for taxpayer – preserves the loss and does not reduces the capital gain from the other transactions (if any)

1231(c): Recapture:

When you get a 1231 gain, you must look back to the prior 5 years for 1231 losses – If there are losses which were deducted, they are recaptured and recharacterized as ordinary income to the extent of the deducted losses

No rule against taking the gain in the earlier year and taking the loss in the next year

Case: WasnokCourts are split as to whether owning and renting one house is a business

Williams & McGowan: Sale of all the assets is the sale of each asset – each with its own basis etc. and tax treatment

§ 1245: Mean provision and overrides 1231 453 etc.Depreciation Recapture – if you have gain from depreciable personal property (1231

property minus land & bldg)(vehicle, computers, office furniture)

Page 83: Beck Fed IncomeTax Outline

if you sell it at a gain, any gain due to depreciation is ordinary income – does not apply unless you actually took the depreciation

EX: Truck used in business – cost 100, depreciation of 50 so basis is 50 – sell it at 120, basis of 50 and gain of 70 – under 1245, the gain is turned into 50 of ordinary income (called 1245 gain) and 20 of 1231 gain – this is done before the firepot calculation

Changes the character of the gain, not the amount – only applies to gain from depreciation, not to losses

Same pattern as in depreciation for real estate § 121

1245 taint sticks to the property – but death wipes out the taint because on death you get stepped up basis and the gain is wiped out anyway

#26 12/4/02 Chap 23 A & B Bad Debts, Charity (skip pp812 – end, no probs)

Chap 23: Deductions Affected by Characterization PrinciplesI. Bad Debts and Worthless Securities §165(g)(1) & (2); 166(a)-(e) Reg 1.165-5(a)-(c);1.166-1(c), (e)-(g), -2(a) & (b), -5(a)-(b)

A. "There shall be allowed as a deduction any debt which becomes worthless within the taxable year." § 166(a)(1).1. IRC § 166(b) deals with the amount that can be taken as a bad debt deduction.

B. When determining whether a bad debt deduction under § 166(a)(1) is appropriate, a taxpayer must in turn determine (1) whether there is a debt, (2) whether it is a bad debt, and (3) whether it is a business bad debt.

C. What Constitutes a Debt?1. To qualify for a bad debt deduction under § 166, there must exist a bona fide

debt which arises from a debtor-creditor relationship based on a valid and enforceable obligation to pay a fixed and determinable sum of money. Howard S. Bugbee.a. Whether a transfer of money creates a bona fide debt depends on the

existence of an intent by both parties, substantially contemporaneous to the time of such transfer, to establish an enforceable obligation of repayment. Bugbee.

2. In some cases, the forgiveness of a loan can be construed as a gift (see § 102 and its accompanying cases for the definition of a gift). When forgiveness of a loan is construed to be a gift, there is no debt and thus there can be no bad debt deduction.

D. What Constitutes a "Bad" Debt?1. A "bad" debt is a debt that is uncollectible.2. Debts can be either wholly or partially bad.

E. Why Must a Debt be Characterized as Business or Nonbusiness?1. IRC § 166(d) differentiates between business and nonbusiness bad debts, and

sets forth the way each is treated under the code.a. Business bad debts are not treated the same as nonbusiness bad debts.

Under § 166(d)(1), business bad debt losses are deductible against

Page 84: Beck Fed IncomeTax Outline

ordinary income, while nonbusiness bad debt losses are deductible only as short-term capital losses.

b. IRC § 166(d)(2) defines the term nonbusiness bad debt.c. Nonbusiness and business bad debts are treated differently because of the

potential for abuse in nonbusiness settings.2. The character of a bad debt loss is determined by the relationship it bears to

the taxpayer's trade or business. A debt will only qualify as a business bad debt if it bears a direct relationship to the taxpayer's trade or business. Charles J. Haslam; United States v. Generes.

Beck mentioned the taxpayer’s motive a. For the purposes of defining a business bad debt, investment activities are

not considered the same as a trade or business (i.e., an investment-related debt is a nonbusiness debt).

b. A corporation's business is not necessarily the same as that of a shareholder, so a classification of a loan as a business loan depends upon the taxpayer's own business. Whipple v. Commissioner.

Bad Debts

Business or non-businessBusiness bad debts get favorable treatment: Deduction without limitNon-business bad debt can only be deducted as a short term capital loss

Business bad debt: either incurred in the trade or business or become bad and is transferred to trade or business. Partially worthless, the worthless part is deductible

Non-business Bad Debt: Treated like all other short term capital losses – must be wholly worthless

EX: You loan money to someone who become bankrupt and they can only pay 10 cents on the dollarIf business debt, you can immediately deduct 90% of the debt – if it is non-business, you cannot deduct until you know how much you will get and deduct the balance, whatever that is

For any kind, the debt must be worthless in whole or in part: this depends on the size of the debt

Is it a debt at all? Must be a valid debt; parties have established a creditor/debtor relationship and there is an expectation of re-payment. To screen out gifts

What can you deduct? Maximum is your basis (what you have loaned)Landlord/tenant – tenant owes rent – no bad debt – no basis in the debt – hasn’t loaned any money

Page 85: Beck Fed IncomeTax Outline

Alimony: No bad debt, no basis, this is an income item

Child support: no loan, no basis; “a bad is a loss; you can’t loose what you never had” all these are are promises that they will pay, these are disappointed promises

Year of worthlessness: hard to determine so there is a special statute of limitations of 7 years to allow for amendment of returns if IRS challenges a deduction

§111 Recovery of a previously deducted amount – if a bad debt is deducted in one year and then make good in another year, you must report the amount as income (same as you must do for state income tax returns) if there was a tax benefit

Worthless securities (Corp bonds and stocks) § 165(g): deemed to have been sold on the last day of the year in which it turned bad; could be long term or short term depending on when bought and then get treated accordingly in capital gains and losses

II. Charitable Donation Deductions § 170(a)(1)A. § 170(a)(1) states that, with certain exceptions, a deduction can be claimed for

the amount of any charitable contribution (as specified in § 170(c)) made during the taxable year.1. For the purposes of IRC § 170, a deductible contribution "is a voluntary

transfer of money or property that is made with no expectation of procuring a financial benefit commensurate with the amount of the transfer." Rev. Rul. 83-104.a. In ascertaining whether a given payment was made with the

expectation of something in return, the IRS focuses its examination on the external features of the transaction in question, thus obviating the need for imprecise inquiries into the motivations of individual taxpayers.

2. When money is donated to a charity, the amount of the deduction is the amount donated. When property is donated, the fair market value of the property generally serves as the amount which can be deducted (though in the case of appreciated property the amount deductible may be reduced by some or all of the value of the built-in gain).a. A gift of services or time is not deductible under § 170. Nor is a

gift to an individual (as opposed to an organization).b. If a taxpayer donates less than a full interest in a piece of property

to charity, a series of special rules applies.3. Congress allows the § 170 deduction because it relieves the federal

government of the obligation to provide the public with the services that the charities provide.

4. IRC § 170(c) divides the qualified charitable organizations into two groups: public charities (which are substantially funded by members of the

Page 86: Beck Fed IncomeTax Outline

general public) and private foundations (which are substantially funded by private persons or small groups).

5. Congress has imposed various ceilings on the total amount of charitable deductions that a single taxpayer may deduct in one year.a. IRC § 170(b) imposes a 50% ceiling on individual gifts to public

charities, meaning no individual may claim charitable deductions to public charities in excess of 50% of his or her adjusted gross income.

b. There is a 30% ceiling on gifts from individuals made for the use of public charities and on gifts to private charities.

c. The IRC allows a carry-over of up to five years of charitable donation deductions.

6. Charitable contribution deductions will only be permitted if they can be properly verified by means of a receipt, cancelled check, etc.

B. Rev. Rul. 83-104 provides an example of the workings of IRC § 170 in a religious school-contribution context.1. In Rev. Rul. 83-104 the Treasury Department concluded that "Tuition

expenditures by a taxpayer to an educational institution are . . . not deductible as charitable contributions . . . because they are required payments for which the taxpayer receives benefits presumably equal in value to the amount paid."

2. The Department also stated: "Similarly, payments made . . . on behalf of children attending . . . church-sponsored schools are not allowable deductions as contributions either to the school or to the religious organization . . . if the payments are earmarked for such children."

3. Finally, the Department further decided that "the fact that the payments are not earmarked does not necessarily mean that the payments are deductible," and that conversely, "a charitable deduction for a payment to an organization that operates a school will not be denied solely because the payment was, to any substantial extent, offset by the fair market value of the services rendered to the taxpayer in the nature of tuition" (i.e., if the amount of a contribution exceeds the fair market value of the education (as measured by tuition), the difference is deductible).

C. With respect to payments to organizations operated exclusively for religious purposes, a contribution is deductible only if such payment is a "contribution or gift" made with no expectation of gaining religious benefits or access to a religious service. Hernandez v. Commissioner.

D. Rev. Rul. 67-246 addresses the application of IRC § 170 in the context of payments "in connection with admission to . . . fund-raising activities for charity."1. In this ruling, the Treasury Department stated: "As a general rule, where a transaction involving a payment is in the form of a purchase of an item of value, the presumption arises that no gift has been made for charitable contribution purposes, the presumption being that the payment in such case is the purchase price."

a. In the case of the purchase of tickets to a fund-raising activity, this principle dictates that if the cost of the tickets to a charity banquet

Page 87: Beck Fed IncomeTax Outline

is equal to the admission price of a similar banquet for which tickets are normally purchased, no charitable deduction is available. For example, if the cost of a charitable dinner is $150, and the value of the dinner is $50, the taxpayer can take only a $100 deduction.

2. The Department further stated that "if a charitable contribution deduction is claimed with respect to [such a] payment, the burden is on the taxpayer to establish that the amount paid is not the purchase price of the privileges or benefits [i.e., the total payment exceeded the value of the item purchased] and that part of the payment, in fact, does not qualify as a gift."

3. In the normal case, an organization conducting a charitable event should specify what percentage of the ticket price represents the fair market value of the good received, and what percent can be deducted as a charitable contribution.

Deductible if:1) to a qualified institution – charitable status determined by IRS2) no quid pro quo – cannot receive anything of market value3) gift can be property or cash (cannot deduct value of time) but can deduct out of pocket expenses incident to volunteering your time – property at full fair market value (great if you have appreciated property because you don’t pay the capital gains)

§170(e): If you donate property which would otherwise have produced income, you can only deduct your basiscan’t donate more than 50% of income

IV. Casualty and Theft Loss § 165(a) & (c) Reg 1.165-1(e), -7(a)(1), (3), (5), -8(a)(1), (d)

A. The Nature of Casualty and Theft Losses Allowed1. IRC § 165(c)(3) provides that in the case of an individual, a deduction

may be taken for "losses of property not connected with a trade or business or a transaction entered into for profit [i.e., personal items], if such losses arise from fire, storm, shipwreck, or other casualty or theft."a. IRC § 165(c)(3) is an exception to the general rule that personal

losses (i.e., those unconnected with a trade or business) are non-deductible.

2. In order for a "casualty" loss to occur, there must be "an identifiable event of a sudden, unusual or unexpected nature" which causes the loss of property to occur.a. For instance, in Rev. Rul. 63-232, the Treasury Department

determined that a loss resulting from damage caused by termites does not qualify as a casualty loss because "termite infestation and the resulting damage cannot be inflicted with the suddenness comparable to that caused by fire, storm or shipwreck."

Page 88: Beck Fed IncomeTax Outline

3. The term "other casualty loss" in IRC 165(c)(3) refers to casualty losses involving physical damage or loss of physical property, and not just a decrease in value. Pulvers v. Commissioner.

4. The burden of proof is on the taxpayer to show that a § 165(c)(3) loss has occurred and that a deduction is appropriate. If the reasonable inferences from evidence presented regarding the disappearance of a valuable item point to theft, then a taxpayer is justified in taking a § 165 theft deduction. Mary Frances Allen.

B. Other Facets of the Casualty and Theft Loss Provisions1. Casualty losses are generally deductible for the year in which they are

sustained. In many cases this is the year of the casualty, but in other cases it may be the year in which the amount of loss is actually ascertained (which may not be the year in which the loss occurs).a. One limited exception to the general rule provides that if a casualty

loss occurs as a result of a disaster which subsequently leads to the area being declared a disaster area by the President, the deduction may be permitted for a year prior to the loss.

2. Theft losses are generally deductible in the year in which the theft is discovered. If additional losses stemming from the same theft are discovered in later years, the proper course of action is to file an amended return for the year in which the losses are first deducted.

3. If there is a reasonable prospect of recovery of the loss (such as insurance payments, return of stolen items, etc.), no deduction will be permitted until it becomes clear there will be no recovery.a. If a deduction is taken because there is no such reasonable prospect

of recovery, yet recovery is unexpectedly received, the amount recovered is deemed to be taxable income in the year in which it is received.

4. Generally, business and personal casualty losses are measured in the same way (i.e., the amount of deductible loss is the same regardless of whether the loss is business or personal). There are some differences, however, which are set forth in Treas. Reg. 1.165-7(b).

Personal Use Property: Loss is not deductible; but gains are taxable Exception is casualty losses Losses due to fire, flood, shipwreck and theftFirepot losses of § 1231Losses from involuntary conversion or destruction are deductible but with huge limits$100 per occurrence haircut10% of AGI floor

Page 89: Beck Fed IncomeTax Outline

What if you casualty gains and losses from insurance?

Net them; make sure to subtract $100 for each occurrence

if there is a net gain, it is capital gainif there is a net loss, then apply the 10% floor and deduct the rest below the line case law:loss must be sudden – house collapse due to termites is not casualty but flood would bethere must be actual damage – Pulver case: burden of proof is on taxpayer – Mary allen case: missing brooch, claimed to be stolen

but she couldn’t prove it, no deduction

amount of the loss? Measure of loss is the difference between value before loss and after loss, limited by basis

fmv before is 100fmv after is 10then loss is 90but if your basis is only 50, then that is your loss

#27 Chap 23 C Casualty Losses, Chap 24 A Installment Sales, Chap 25 Losses (skim only)

§ 453 Installment Saleif you sell property with payments due in future years, there is a liquidity problem

because you will owe taxes on the total gain§ 453 matches the tax on the gain with the payments due in the current year FMV 1000AB 100

Sell it with down payment 200 and 8 future payments of 100This is taxed by calculating profit if all the payments are made – in eg profit would be

900Profit ratio is 900/1000or 90%Multiple each installment by the profit ratio so each payment would be taxable 90% and

10% return of basis Down payment is subject to same rule – 90% x 200 = 180 of gain and 20 return of capital

Total exempts = basis Total taxable = gain

Purchaser gets full basis in year one – he is borrowing money from the seller

Wrinkles:

Page 90: Beck Fed IncomeTax Outline

Cannot defer § 1245 gain – overrides 453 and will not let you defer the gain It gets taxed immediately and you must reduce the expected profit and adjust the profit

ratio

Can’t defer gain for publicly listed securities – taxed on gain on year in which you make sale, not in year in which payment is received

Interest:Separately dealt with If you have 10% interest on an installment sale, the interest is income to the seller

Seller would prefer to be taxed at the capital rate. Buyer would like the entire purchase price to go into basis. So they both want to hide interest.

If interest is not provided for, interest will be imputed.

Pg 876§ 691 & 1014what happens to income items which are owed to decedents?They are ordinary income to whoever collects them; an exception to step-up basis rule.

LossesIf you have property you are holding at a lossFmv 10Basis 100If you sell it you will have a loss (not a personal use asset)

§267: if you sell to a related party, you lose the loss §267(d): but if the property goes back up in value, you will not be taxed on the gain (to

the extent of the disallowed loss)

Consistent with the carry over basis in the case of gifts and the case of no man’s land of non recognition of transaction for tax purposes where the sale is above FMV but below basis

This applies to direct sales and indirect sales

Moral: to get a loss deduction, you have to get rid of the property

Wash sales rule § 1091: if you have a stock loss and want to get the loss, but want to re-buy the stock, you must wait 31 days in order to maintain the loss

Page 91: Beck Fed IncomeTax Outline

12/4/02#28

Chap 26 A – D Non-recognition

I. Introduction to Non-recognition principlesA. Gain or loss has no economic or tax significance unless it is both

realized (obtained or incurred) and recognized.1. Not all realized gains are subject to immediate consideration under

the IRC.a. If gain or loss is not subject to immediate consideration, it

is not generally subject to tax.b. Provisions that allow for nonrecognition or deferred

reporting usually recognize the existence of a gain or loss, but allow it to go unrecognized for tax purposes, at least for the time being.

2. A threshold question in tax liability is always whether a realized gain or loss is recognized under the Code.

3. Some Code provisions are merely timing devices to determine when to recognize gain or loss, but others are outright disallowances of recognition.

4. Nonrecognition provisions are predicated on the notion that gain or loss is sensibly deferred when the taxpayer has essentially retained an investment that is similar to the original investment.

5. § 1041, encountered in an earlier chapter, provides for the nonrecognition of gain or loss on transfers between spouses or incident to a divorce.a. § 1041, along with other nonrecognition provisions carries

with it an adjusted basis concept.6. Most nonrecognition provisions basically provide only a

postponement of tax on gain or deduction of loss that initially goes unrecognized by utilizing adjusted basis concepts.

B. Non-Recognition in like kind exchanges §§ 1001(c); 1031; 1223(1) Reg 1.1031(a)-1, -2(a); 1.1031(b)-1(b) ex 1; 1.1031(d)-1

1. Like kind exchanges are exchanges of property held for productive use in a trade or business or for investment.a. An exchange of property that does not fit within these categories

does not qualify for like kind exchange treatment.b. The transaction must be an exchange rather than a sale for Section

1031 to apply.c. Exchanges must be of like kind property as defined in the statute.

Page 92: Beck Fed IncomeTax Outline

2. A sale is a transfer of property for money or its equivalent3. Generally, an exchange is property for property, but the presence of a

small amount of cash in an exchange of property will not automatically disqualify the transfer for like kind treatment. Bloomington Coca-Cola Bottling Co. v. Commissioner.

4. An exchange of real property for real property is a like kind exchange, regardless of the dissimilarities in grade or nature between the properties.a. The rule is that like kind property refers to the nature or character

of the property, i.e. real or personal, and not to its grade or quality.b. Like kind is interpreted broadly when applied to real estate

transactions.c. Like kind is given a more narrow interpretation when personal

property is exchanged.

(1) For example, the IRS has held that an exchange of gold bullion for silver bullion is not an exchange of like kind property because the metals are primarily used in different ways.

(2) Section 1031 and the regulations promulgated thereunder classify personal property into "General Asset Classes" or "Product Classes," among others.(a) In general, personal property that is exchanged with

property in the same class is like kind property.5. The IRS and Courts have interpreted sale-leaseback transactions

differently.a. The Second and Third Circuits have interpreted sale-leaseback

arrangements with lease terms of 30+ years as constituting sales. Leslie Co. v. Commissioner.(1) An exchange is a reciprocal transfer of properties while a

sale is the transfer of property for money consideration only. Leslie Co. v. Commissioner.

(2) The initial issue to be resolved is the character of the transaction. Leslie Co. v. Commissioner.

(3) To determine whether money was the sole consideration for a transfer of property, it is necessary to value the properties involved in the transaction. Leslie Co. v. Commissioner.

b. The Eight Circuit and the IRS take the position that a sale-leaseback arrangement providing for a lease term of 30 + years is a like kind exchange and subject to nonrecognition. Century Electric Co. v. Commissioner.

c. The IRS has taken the position that a sale and leaseback in which the fee title is exchanged for a long-term leasehold interest (30 or more years) is a like kind exchange and any gain or loss on the transaction is not recognized.

Page 93: Beck Fed IncomeTax Outline

d. Occasionally, the IRS uses the familiar substance over form argument in attempts to invoke the non-recognition of losses in transactions involving the exchange of property

C. Three-cornered exchanges are sometimes used by taxpayers to avoid adverse tax consequences.

1. Section 1031 does not apply if property is sold for money and the proceeds are invested in property of a like kind.

2. The IRS recognizes the use of three-cornered exchanges, which use a combination of Section 1031 and other tax neutralizing factors to effectuate a transaction. See Revenue Ruling 77-297.

D. Non-simultaneous exchanges have been scrutinized by Congress and provisions have been made to narrow the availability of 1031 treatment in cases of non-simultaneous exchanges.

1. Section 1031(a)(3) allows an outright transfer but limits the taxpayer to relatively short periods of time within which to identify and receive like kind property. a. The identification period is generally 45 days after the taxpayer

transfers the property that is relinquished.b. The exchange period is generally 180 days after the transfer is

made.

E. Issues arising under Section 1031

1. Section 1031 applies to both losses and gains and is non-elective.a. If the requirements of 1031 are met the Section applies

automatically.2. Section 1031 is accompanied by an exchanged basis provision and a

tacked holding period.3. Section 1031(a) applies only to exchanges for like kind property.4. Section 1031 (b) or (c) may apply to exchanges for like kind property

involving cash or non like kind property in the form of boot.5. If the property exchanged is subject to a liability, the amount of the

liability is treated as cash boot received by the transferor in the transaction.a. If both properties in the exchange are encumbered, the amount of

liabilities on both are offset against one another and any excess relief is treated as boot to the receiver.

Page 94: Beck Fed IncomeTax Outline

II. Involuntary Conversions

A. Section 1033 provides for the nonrecognition of gain in certain circumstances where property has been involuntarily converted.

B. An involuntary conversion may occur as a result of total destruction, theft, seizure, reclamation, requisition or condemnation, or by the sale of the property as a result of the threat or imminence of requisition or condemnation.

C. Section 1033 is an elective provision that allows the taxpayer to recognize gain on an involuntary conversion of property only to the extent that the amount realized exceeds the cost of replacement property.1. The section allows the taxpayer to take the monetary proceeds of

the involuntary conversion and buy replacement property with those proceeds. The taxpayer is only taxed on a gain if the cost of the replacement property is lower than the amount received for the involuntary conversion.

D. The taxpayer must purchase replacement property that is "similar or related in use" to the converted property and must do so within the time period mandated by the section in order to qualify for nonrecognition of gain.1. When two pieces of property are used together and constitute one

economic unit, and one piece is involuntarily converted, the IRS has considered the other piece to have been involuntarily converted also, and replacement property bought with the proceeds of the involuntary conversion plus the sale of the other property have qualified for nonrecognition under Section 1033. Harry G. Masser.

2. The test for whether property is similar or related in use to the converted property centers on an inquiry as to the relationship of the taxpayer to the properties involved. Clifton Investment Co. v. Commissioner.a. If the properties require the same sorts of management and

services to be rendered by the owner and have similar business risks associated with them, they will ordinarily be considered properties similar or related in use for purposes of Section 1031. Clifton Inv. Co. v. Commissioner.

b. If there is a material alteration in the relationship of the taxpayer to the replacement property versus the original property, the properties will not generally be considered similar or related in use.

3. The courts and the IRS have disagreed as to the degree of consideration that should be given to the investment character of the properties involved.

4. The IRS will examine the physical properties and the end uses to which the properties are put, in addition to other factors, to decide if the properties are similar or related in use. Revenue Ruling 76-319.

Page 95: Beck Fed IncomeTax Outline

5. The IRS also considers the investment character of the properties involved to be pertinent to the determination of whether the properties are similar or related in use, however, the character is not controlling. Revenue Ruling 71-41.

III. There are several other nonrecognition provisions in the Code.

A. Corporate and business nonrecognition examples:1. A transfer of property to a new corporation in exchange for its

shares, or a transfer of property to a partnership in exchange for a partnership interest is generally subject to nonrecognition.

2. Some partnership distributions and some liquidation distributions are subject to nonrecognition.

3. There are several different types of corporate reorganizations involving transfers of stock that result in nonrecognition for both the corporation and the shareholders involved.

4. With respect to most of these nonrecognition provisions, there is and a tacked holding period usually applies.

B. Non-corporate and non-business nonrecognition provisions:1. Some exchanges of life insurance or annuity contracts qualify for

non-recognition.2. Exchanges of some United States obligations for other such

obligations can be accorded nonrecognition in the proper circumstances.

3. Certain investments in specialized small business investment companies qualify for nonrecognition under Section 1044 of the IRC.

Non-Recognition: § 1031 & 1033

Like Kind Exchanges: Permits exchange of appreciated property for other property with deferment of tax

AB 10FMV 100

Exchanged for like property with same FMV – carry forward old basis of 10 – gain lurks until you sell the second property

Page 96: Beck Fed IncomeTax Outline

- Requires an actual barter/ exchange – can’t just sell property A for cash and then buy property B

What is like kind?

Property has to be held for business or investment (both properties)

AND

Like kind is broad when considering land – hotel in NY is like kind to mineral rights in CO if mineral rights in CO are considered an interest in real property (Lease of 30 years or longer is also an interest in land which would qualify for a like-kind exchange)

For other tangible property, like kind is stricter, narrower – o Gold ingots and silver ingots are not like kind o Livestock of different sexes are not like kind o Computer is not like kind to a desk (but may be for a printer)o Car exchanged for a truck does not qualify

- Heavy construction equipment exchanges likely will qualify

Intangibles: Securities can never be like kind exchanges (Stock for Stock)

Mechanics of like kind exchange:Property A ab 10fmv 100

Property Bab 10fmv 125

Exchange thru C

Cash is called boot – if you receive cash, you are taxed on the gainA has 25 more of basis on property B after the exchange – 35 of basis

If A gets cash, because B is only worth 80:New Basis = carry over + gain recognized – boot rec’d 10 = 10 + 20 – 20

Page 97: Beck Fed IncomeTax Outline

One more problem: Delayed transfer

- A wants to exchange to defer gain; so looks for buyer C who wants to buy immediately but A hasn’t found B property yet

- A transfers to C and receives only from C, his promise/contract to buy B once A finds it and exchanging with A

- There are time limits: A has to identify property B within 90 days and the exchange must occur within 180 days of the first transfer

Property Which is condemned or destroyed: § 1033:

- If tax gain is due to condemnation or destruction, tax can be delayed provided you replace the property within 2 years (even if you get cash) – must be similar or related in service or use (construed extremely narrowly) rulings in book: pool parlor replaced by bowling alley, no good – office building replaced by hotel, no good –

- Other Rulings: Looks not at end use to see if it is similar or related in service or use but to the relationship of the owner to the property – owner was not owner of the property but leased out the property to tenant/mgr who really runs the business – owner just collects lease check – if pool parlor burns down and owner rebuilds and leases to another type of business, it will be allowed because the owner’s relationship to the property is the same (passive lessor)

question 5 on examquestion 1 on exam: 2million grandfathered debt + another 100,000 of home equity

indebtedness